You are on page 1of 74

1

G.R. No. L-22595 November 1, 1927


Testate Estate of Joseph G. Brimo, JUAN MICIANO, administrator, petitioner-appellee,
vs.
ANDRE BRIMO, opponent-appellant.
Ross, Lawrence and Selph for appellant.
Camus and Delgado for appellee.

ROMUALDEZ, J .:
The partition of the estate left by the deceased Joseph G. Brimo is in question in this case.
The judicial administrator of this estate filed a scheme of partition. Andre Brimo, one of the brothers of the deceased, opposed it. The court, however,
approved it.
The errors which the oppositor-appellant assigns are:
(1) The approval of said scheme of partition; (2) denial of his participation in the inheritance; (3) the denial of the motion for reconsideration of the order
approving the partition; (4) the approval of the purchase made by the Pietro Lana of the deceased's business and the deed of transfer of said business;
and (5) the declaration that the Turkish laws are impertinent to this cause, and the failure not to postpone the approval of the scheme of partition and the
delivery of the deceased's business to Pietro Lanza until the receipt of the depositions requested in reference to the Turkish laws.
The appellant's opposition is based on the fact that the partition in question puts into effect the provisions of Joseph G. Brimo's will which are not in
accordance with the laws of his Turkish nationality, for which reason they are void as being in violation or article 10 of the Civil Code which, among other
things, provides the following:
Nevertheless, legal and testamentary successions, in respect to the order of succession as well as to the amount of the successional rights
and the intrinsic validity of their provisions, shall be regulated by the national law of the person whose succession is in question, whatever may
be the nature of the property or the country in which it may be situated.
But the fact is that the oppositor did not prove that said testimentary dispositions are not in accordance with the Turkish laws, inasmuch as he did not
present any evidence showing what the Turkish laws are on the matter, and in the absence of evidence on such laws, they are presumed to be the same
as those of the Philippines. (Lim and Lim vs. Collector of Customs, 36 Phil., 472.)
It has not been proved in these proceedings what the Turkish laws are. He, himself, acknowledges it when he desires to be given an opportunity to
present evidence on this point; so much so that he assigns as an error of the court in not having deferred the approval of the scheme of partition until the
receipt of certain testimony requested regarding the Turkish laws on the matter.
The refusal to give the oppositor another opportunity to prove such laws does not constitute an error. It is discretionary with the trial court, and, taking
into consideration that the oppositor was granted ample opportunity to introduce competent evidence, we find no abuse of discretion on the part of the
court in this particular. There is, therefore, no evidence in the record that the national law of the testator Joseph G. Brimo was violated in the
testamentary dispositions in question which, not being contrary to our laws in force, must be complied with and executed. lawphil.net
Therefore, the approval of the scheme of partition in this respect was not erroneous.
In regard to the first assignment of error which deals with the exclusion of the herein appellant as a legatee, inasmuch as he is one of the persons
designated as such in will, it must be taken into consideration that such exclusion is based on the last part of the second clause of the will, which says:
Second. I like desire to state that although by law, I am a Turkish citizen, this citizenship having been conferred upon me by conquest and not
by free choice, nor by nationality and, on the other hand, having resided for a considerable length of time in the Philippine Islands where I
succeeded in acquiring all of the property that I now possess, it is my wish that the distribution of my property and everything in connection
with this, my will, be made and disposed of in accordance with the laws in force in the Philippine islands, requesting all of my relatives to
respect this wish, otherwise, I annul and cancel beforehand whatever disposition found in this will favorable to the person or persons who fail
to comply with this request.
The institution of legatees in this will is conditional, and the condition is that the instituted legatees must respect the testator's will to distribute his
property, not in accordance with the laws of his nationality, but in accordance with the laws of the Philippines.
If this condition as it is expressed were legal and valid, any legatee who fails to comply with it, as the herein oppositor who, by his attitude in these
proceedings has not respected the will of the testator, as expressed, is prevented from receiving his legacy.
The fact is, however, that the said condition is void, being contrary to law, for article 792 of the civil Code provides the following:

2
Impossible conditions and those contrary to law or good morals shall be considered as not imposed and shall not prejudice the heir or legatee
in any manner whatsoever, even should the testator otherwise provide.
And said condition is contrary to law because it expressly ignores the testator's national law when, according to article 10 of the civil Code above quoted,
such national law of the testator is the one to govern his testamentary dispositions.
Said condition then, in the light of the legal provisions above cited, is considered unwritten, and the institution of legatees in said will is unconditional and
consequently valid and effective even as to the herein oppositor.
It results from all this that the second clause of the will regarding the law which shall govern it, and to the condition imposed upon the legatees, is null
and void, being contrary to law.
All of the remaining clauses of said will with all their dispositions and requests are perfectly valid and effective it not appearing that said clauses are
contrary to the testator's national law.
Therefore, the orders appealed from are modified and it is directed that the distribution of this estate be made in such a manner as to include the herein
appellant Andre Brimo as one of the legatees, and the scheme of partition submitted by the judicial administrator is approved in all other respects,
without any pronouncement as to costs.
So ordered.
Street, Malcolm, Avancea, Villamor and Ostrand, JJ., concur.

Republic of the Philippines
SUPREME COURT
Manila
EN BANC
G.R. No. L-16749 January 31, 1963
IN THE MATTER OF THE TESTATE ESTATE OF EDWARD E. CHRISTENSEN, DECEASED.
ADOLFO C. AZNAR, Executor and LUCY CHRISTENSEN, Heir of the deceased, Executor and Heir-appellees,
vs.
HELEN CHRISTENSEN GARCIA, oppositor-appellant.
M. R. Sotelo for executor and heir-appellees.
Leopoldo M. Abellera and Jovito Salonga for oppositor-appellant.
LABRADOR, J .:
This is an appeal from a decision of the Court of First Instance of Davao, Hon. Vicente N. Cusi, Jr., presiding, in Special Proceeding No. 622 of said
court, dated September 14, 1949, approving among things the final accounts of the executor, directing the executor to reimburse Maria Lucy
Christensen the amount of P3,600 paid by her to Helen Christensen Garcia as her legacy, and declaring Maria Lucy Christensen entitled to the residue
of the property to be enjoyed during her lifetime, and in case of death without issue, one-half of said residue to be payable to Mrs. Carrie Louise C.
Borton, etc., in accordance with the provisions of the will of the testator Edward E. Christensen. The will was executed in Manila on March 5, 1951 and
contains the following provisions:
3. I declare ... that I have but ONE (1) child, named MARIA LUCY CHRISTENSEN (now Mrs. Bernard Daney), who was born in the
Philippines about twenty-eight years ago, and who is now residing at No. 665 Rodger Young Village, Los Angeles, California, U.S.A.
4. I further declare that I now have no living ascendants, and no descendants except my above named daughter, MARIA LUCY
CHRISTENSEN DANEY.
x x x x x x x x x
7. I give, devise and bequeath unto MARIA HELEN CHRISTENSEN, now married to Eduardo Garcia, about eighteen years of age
and who, notwithstanding the fact that she was baptized Christensen, is not in any way related to me, nor has she been at any time
adopted by me, and who, from all information I have now resides in Egpit, Digos, Davao, Philippines, the sum of THREE
THOUSAND SIX HUNDRED PESOS (P3,600.00), Philippine Currency the same to be deposited in trust for the said Maria Helen
Christensen with the Davao Branch of the Philippine National Bank, and paid to her at the rate of One Hundred Pesos (P100.00),
Philippine Currency per month until the principal thereof as well as any interest which may have accrued thereon, is exhausted..
x x x x x x x x x

3
12. I hereby give, devise and bequeath, unto my well-beloved daughter, the said MARIA LUCY CHRISTENSEN DANEY (Mrs.
Bernard Daney), now residing as aforesaid at No. 665 Rodger Young Village, Los Angeles, California, U.S.A., all the income from
the rest, remainder, and residue of my property and estate, real, personal and/or mixed, of whatsoever kind or character, and
wheresoever situated, of which I may be possessed at my death and which may have come to me from any source whatsoever,
during her lifetime: ....
It is in accordance with the above-quoted provisions that the executor in his final account and project of partition ratified the payment of only P3,600 to
Helen Christensen Garcia and proposed that the residue of the estate be transferred to his daughter, Maria Lucy Christensen.
Opposition to the approval of the project of partition was filed by Helen Christensen Garcia, insofar as it deprives her (Helen) of her legitime as an
acknowledged natural child, she having been declared by Us in G.R. Nos. L-11483-84 an acknowledged natural child of the deceased Edward E.
Christensen. The legal grounds of opposition are (a) that the distribution should be governed by the laws of the Philippines, and (b) that said order of
distribution is contrary thereto insofar as it denies to Helen Christensen, one of two acknowledged natural children, one-half of the estate in full
ownership. In amplification of the above grounds it was alleged that the law that should govern the estate of the deceased Christensen should not be the
internal law of California alone, but the entire law thereof because several foreign elements are involved, that the forum is the Philippines and even if the
case were decided in California, Section 946 of the California Civil Code, which requires that the domicile of the decedent should apply, should be
applicable. It was also alleged that Maria Helen Christensen having been declared an acknowledged natural child of the decedent, she is deemed for all
purposes legitimate from the time of her birth.
The court below ruled that as Edward E. Christensen was a citizen of the United States and of the State of California at the time of his death, the
successional rights and intrinsic validity of the provisions in his will are to be governed by the law of California, in accordance with which a testator has
the right to dispose of his property in the way he desires, because the right of absolute dominion over his property is sacred and inviolable (In re
McDaniel's Estate, 77 Cal. Appl. 2d 877, 176 P. 2d 952, and In re Kaufman, 117 Cal. 286, 49 Pac. 192, cited in page 179, Record on Appeal). Oppositor
Maria Helen Christensen, through counsel, filed various motions for reconsideration, but these were denied. Hence, this appeal.
The most important assignments of error are as follows:
I
THE LOWER COURT ERRED IN IGNORING THE DECISION OF THE HONORABLE SUPREME COURT THAT HELEN IS THE ACKNOWLEDGED
NATURAL CHILD OF EDWARD E. CHRISTENSEN AND, CONSEQUENTLY, IN DEPRIVING HER OF HER JUST SHARE IN THE INHERITANCE.
II
THE LOWER COURT ERRED IN ENTIRELY IGNORING AND/OR FAILING TO RECOGNIZE THE EXISTENCE OF SEVERAL FACTORS, ELEMENTS
AND CIRCUMSTANCES CALLING FOR THE APPLICATION OF INTERNAL LAW.
III
THE LOWER COURT ERRED IN FAILING TO RECOGNIZE THAT UNDER INTERNATIONAL LAW, PARTICULARLY UNDER THE RENVOI
DOCTRINE, THE INTRINSIC VALIDITY OF THE TESTAMENTARY DISPOSITION OF THE DISTRIBUTION OF THE ESTATE OF THE DECEASED
EDWARD E. CHRISTENSEN SHOULD BE GOVERNED BY THE LAWS OF THE PHILIPPINES.
IV
THE LOWER COURT ERRED IN NOT DECLARING THAT THE SCHEDULE OF DISTRIBUTION SUBMITTED BY THE EXECUTOR IS CONTRARY
TO THE PHILIPPINE LAWS.
V
THE LOWER COURT ERRED IN NOT DECLARING THAT UNDER THE PHILIPPINE LAWS HELEN CHRISTENSEN GARCIA IS ENTITLED TO ONE-
HALF (1/2) OF THE ESTATE IN FULL OWNERSHIP.
There is no question that Edward E. Christensen was a citizen of the United States and of the State of California at the time of his death. But there is
also no question that at the time of his death he was domiciled in the Philippines, as witness the following facts admitted by the executor himself in
appellee's brief:
In the proceedings for admission of the will to probate, the facts of record show that the deceased Edward E. Christensen was born
on November 29, 1875 in New York City, N.Y., U.S.A.; his first arrival in the Philippines, as an appointed school teacher, was on
July 1, 1901, on board the U.S. Army Transport "Sheridan" with Port of Embarkation as the City of San Francisco, in the State of
California, U.S.A. He stayed in the Philippines until 1904.
In December, 1904, Mr. Christensen returned to the United States and stayed there for the following nine years until 1913, during
which time he resided in, and was teaching school in Sacramento, California.
Mr. Christensen's next arrival in the Philippines was in July of the year 1913. However, in 1928, he again departed the Philippines
for the United States and came back here the following year, 1929. Some nine years later, in 1938, he again returned to his own
country, and came back to the Philippines the following year, 1939.

4
Wherefore, the parties respectfully pray that the foregoing stipulation of facts be admitted and approved by this Honorable Court,
without prejudice to the parties adducing other evidence to prove their case not covered by this stipulation of facts. 1wph1.t
Being an American citizen, Mr. Christensen was interned by the Japanese Military Forces in the Philippines during World War II.
Upon liberation, in April 1945, he left for the United States but returned to the Philippines in December, 1945. Appellees Collective
Exhibits "6", CFI Davao, Sp. Proc. 622, as Exhibits "AA", "BB" and "CC-Daney"; Exhs. "MM", "MM-l", "MM-2-Daney" and p. 473,
t.s.n., July 21, 1953.)
In April, 1951, Edward E. Christensen returned once more to California shortly after the making of his last will and testament (now in
question herein) which he executed at his lawyers' offices in Manila on March 5, 1951. He died at the St. Luke's Hospital in the City
of Manila on April 30, 1953. (pp. 2-3)
In arriving at the conclusion that the domicile of the deceased is the Philippines, we are persuaded by the fact that he was born in New York, migrated to
California and resided there for nine years, and since he came to the Philippines in 1913 he returned to California very rarely and only for short visits
(perhaps to relatives), and considering that he appears never to have owned or acquired a home or properties in that state, which would indicate that he
would ultimately abandon the Philippines and make home in the State of California.
Sec. 16. Residence is a term used with many shades of meaning from mere temporary presence to the most permanent abode.
Generally, however, it is used to denote something more than mere physical presence. (Goodrich on Conflict of Laws, p. 29)
As to his citizenship, however, We find that the citizenship that he acquired in California when he resided in Sacramento, California from 1904 to 1913,
was never lost by his stay in the Philippines, for the latter was a territory of the United States (not a state) until 1946 and the deceased appears to have
considered himself as a citizen of California by the fact that when he executed his will in 1951 he declared that he was a citizen of that State; so that he
appears never to have intended to abandon his California citizenship by acquiring another. This conclusion is in accordance with the following principle
expounded by Goodrich in his Conflict of Laws.
The terms "'residence" and "domicile" might well be taken to mean the same thing, a place of permanent abode. But domicile, as
has been shown, has acquired a technical meaning. Thus one may be domiciled in a place where he has never been. And he may
reside in a place where he has no domicile. The man with two homes, between which he divides his time, certainly resides in each
one, while living in it. But if he went on business which would require his presence for several weeks or months, he might properly
be said to have sufficient connection with the place to be called a resident. It is clear, however, that, if he treated his settlement as
continuing only for the particular business in hand, not giving up his former "home," he could not be a domiciled New Yorker.
Acquisition of a domicile of choice requires the exercise of intention as well as physical presence. "Residence simply requires bodily
presence of an inhabitant in a given place, while domicile requires bodily presence in that place and also an intention to make it
one's domicile." Residence, however, is a term used with many shades of meaning, from the merest temporary presence to the
most permanent abode, and it is not safe to insist that any one use et the only proper one. (Goodrich, p. 29)
The law that governs the validity of his testamentary dispositions is defined in Article 16 of the Civil Code of the Philippines, which is as follows:
ART. 16. Real property as well as personal property is subject to the law of the country where it is situated.
However, intestate and testamentary successions, both with respect to the order of succession and to the amount of successional
rights and to the intrinsic validity of testamentary provisions, shall be regulated by the national law of the person whose succession
is under consideration, whatever may be the nature of the property and regardless of the country where said property may be found.
The application of this article in the case at bar requires the determination of the meaning of the term "national law" is used therein.
There is no single American law governing the validity of testamentary provisions in the United States, each state of the Union having its own private law
applicable to its citizens only and in force only within the state. The "national law" indicated in Article 16 of the Civil Code above quoted can not,
therefore, possibly mean or apply to any general American law. So it can refer to no other than the private law of the State of California.
The next question is: What is the law in California governing the disposition of personal property? The decision of the court below, sustains the
contention of the executor-appellee that under the California Probate Code, a testator may dispose of his property by will in the form and manner he
desires, citing the case of Estate of McDaniel, 77 Cal. Appl. 2d 877, 176 P. 2d 952. But appellant invokes the provisions of Article 946 of the Civil Code
of California, which is as follows:
If there is no law to the contrary, in the place where personal property is situated, it is deemed to follow the person of its owner, and
is governed by the law of his domicile.
The existence of this provision is alleged in appellant's opposition and is not denied. We have checked it in the California Civil Code and it is there.
Appellee, on the other hand, relies on the case cited in the decision and testified to by a witness. (Only the case of Kaufman is correctly cited.) It is
argued on executor's behalf that as the deceased Christensen was a citizen of the State of California, the internal law thereof, which is that given in the
abovecited case, should govern the determination of the validity of the testamentary provisions of Christensen's will, such law being in force in the State
of California of which Christensen was a citizen. Appellant, on the other hand, insists that Article 946 should be applicable, and in accordance therewith
and following the doctrine of therenvoi, the question of the validity of the testamentary provision in question should be referred back to the law of the
decedent's domicile, which is the Philippines.
The theory of doctrine of renvoi has been defined by various authors, thus:

5
The problem has been stated in this way: "When the Conflict of Laws rule of the forum refers a jural matter to a foreign law for
decision, is the reference to the purely internal rules of law of the foreign system; i.e., to the totality of the foreign law minus its
Conflict of Laws rules?"
On logic, the solution is not an easy one. The Michigan court chose to accept the renvoi, that is, applied the Conflict of Laws rule of
Illinois which referred the matter back to Michigan law. But once having determined the the Conflict of Laws principle is the rule
looked to, it is difficult to see why the reference back should not have been to Michigan Conflict of Laws. This would have resulted in
the "endless chain of references" which has so often been criticized be legal writers. The opponents of the renvoi would have looked
merely to the internal law of Illinois, thus rejecting the renvoi or the reference back. Yet there seems no compelling logical reason
why the original reference should be the internal law rather than to the Conflict of Laws rule. It is true that such a solution avoids
going on a merry-go-round, but those who have accepted the renvoi theory avoid this inextricabilis circulas by getting off at the
second reference and at that point applying internal law. Perhaps the opponents of the renvoi are a bit more consistent for they look
always to internal law as the rule of reference.
Strangely enough, both the advocates for and the objectors to the renvoi plead that greater uniformity will result from adoption of
their respective views. And still more strange is the fact that the only way to achieve uniformity in this choice-of-law problem is if in
the dispute the two states whose laws form the legal basis of the litigation disagree as to whether the renvoi should be accepted. If
both reject, or both accept the doctrine, the result of the litigation will vary with the choice of the forum. In the case stated above, had
the Michigan court rejected the renvoi, judgment would have been against the woman; if the suit had been brought in the Illinois
courts, and they too rejected the renvoi, judgment would be for the woman. The same result would happen, though the courts would
switch with respect to which would hold liability, if both courts accepted the renvoi.
The Restatement accepts the renvoi theory in two instances: where the title to land is in question, and where the validity of a decree
of divorce is challenged. In these cases the Conflict of Laws rule of the situs of the land, or the domicile of the parties in the divorce
case, is applied by the forum, but any further reference goes only to the internal law. Thus, a person's title to land, recognized by the
situs, will be recognized by every court; and every divorce, valid by the domicile of the parties, will be valid everywhere. (Goodrich,
Conflict of Laws, Sec. 7, pp. 13-14.)
X, a citizen of Massachusetts, dies intestate, domiciled in France, leaving movable property in Massachusetts, England, and France.
The question arises as to how this property is to be distributed among X's next of kin.
Assume (1) that this question arises in a Massachusetts court. There the rule of the conflict of laws as to intestate succession to
movables calls for an application of the law of the deceased's last domicile. Since by hypothesis X's last domicile was France, the
natural thing for the Massachusetts court to do would be to turn to French statute of distributions, or whatever corresponds thereto in
French law, and decree a distribution accordingly. An examination of French law, however, would show that if a French court were
called upon to determine how this property should be distributed, it would refer the distribution to the national law of the deceased,
thus applying the Massachusetts statute of distributions. So on the surface of things the Massachusetts court has open to it
alternative course of action: (a) either to apply the French law is to intestate succession, or (b) to resolve itself into a French court
and apply the Massachusetts statute of distributions, on the assumption that this is what a French court would do. If it accepts the
so-called renvoidoctrine, it will follow the latter course, thus applying its own law.
This is one type of renvoi. A jural matter is presented which the conflict-of-laws rule of the forum refers to a foreign law, the conflict-
of-laws rule of which, in turn, refers the matter back again to the law of the forum. This is renvoi in the narrower sense. The German
term for this judicial process is 'Ruckverweisung.'" (Harvard Law Review, Vol. 31, pp. 523-571.)
After a decision has been arrived at that a foreign law is to be resorted to as governing a particular case, the further question may
arise: Are the rules as to the conflict of laws contained in such foreign law also to be resorted to? This is a question which, while it
has been considered by the courts in but a few instances, has been the subject of frequent discussion by textwriters and essayists;
and the doctrine involved has been descriptively designated by them as the "Renvoyer" to send back, or the "Ruchversweisung", or
the "Weiterverweisung", since an affirmative answer to the question postulated and the operation of the adoption of the foreign law
in toto would in many cases result in returning the main controversy to be decided according to the law of the forum. ... (16 C.J.S.
872.)
Another theory, known as the "doctrine of renvoi", has been advanced. The theory of the doctrine of renvoiis that the court of the
forum, in determining the question before it, must take into account the whole law of the other jurisdiction, but also its rules as to
conflict of laws, and then apply the law to the actual question which the rules of the other jurisdiction prescribe. This may be the law
of the forum. The doctrine of therenvoi has generally been repudiated by the American authorities. (2 Am. Jur. 296)
The scope of the theory of renvoi has also been defined and the reasons for its application in a country explained by Prof. Lorenzen in an article in the
Yale Law Journal, Vol. 27, 1917-1918, pp. 529-531. The pertinent parts of the article are quoted herein below:
The recognition of the renvoi theory implies that the rules of the conflict of laws are to be understood as incorporating not only the
ordinary or internal law of the foreign state or country, but its rules of the conflict of laws as well. According to this theory 'the law of
a country' means the whole of its law.
x x x x x x x x x
Von Bar presented his views at the meeting of the Institute of International Law, at Neuchatel, in 1900, in the form of the following
theses:
(1) Every court shall observe the law of its country as regards the application of foreign laws.

6
(2) Provided that no express provision to the contrary exists, the court shall respect:
(a) The provisions of a foreign law which disclaims the right to bind its nationals abroad as regards their personal statute,
and desires that said personal statute shall be determined by the law of the domicile, or even by the law of the place
where the act in question occurred.
(b) The decision of two or more foreign systems of law, provided it be certain that one of them is necessarily competent,
which agree in attributing the determination of a question to the same system of law.
x x x x x x x x x
If, for example, the English law directs its judge to distribute the personal estate of an Englishman who has died domiciled in
Belgium in accordance with the law of his domicile, he must first inquire whether the law of Belgium would distribute personal
property upon death in accordance with the law of domicile, and if he finds that the Belgian law would make the distribution in
accordance with the law of nationality that is the English law he must accept this reference back to his own law.
We note that Article 946 of the California Civil Code is its conflict of laws rule, while the rule applied in In re Kaufman, Supra, its internal law. If the law on
succession and the conflict of laws rules of California are to be enforced jointly, each in its own intended and appropriate sphere, the principle cited In re
Kaufman should apply to citizens living in the State, but Article 946 should apply to such of its citizens as are not domiciled in California but in other
jurisdictions. The rule laid down of resorting to the law of the domicile in the determination of matters with foreign element involved is in accord with the
general principle of American law that the domiciliary law should govern in most matters or rights which follow the person of the owner.
When a man dies leaving personal property in one or more states, and leaves a will directing the manner of distribution of the
property, the law of the state where he was domiciled at the time of his death will be looked to in deciding legal questions about the
will, almost as completely as the law of situs is consulted in questions about the devise of land. It is logical that, since the domiciliary
rules control devolution of the personal estate in case of intestate succession, the same rules should determine the validity of an
attempted testamentary dispostion of the property. Here, also, it is not that the domiciliary has effect beyond the borders of the
domiciliary state. The rules of the domicile are recognized as controlling by the Conflict of Laws rules at the situs property, and the
reason for the recognition as in the case of intestate succession, is the general convenience of the doctrine. The New York court
has said on the point: 'The general principle that a dispostiton of a personal property, valid at the domicile of the owner, is valid
anywhere, is one of the universal application. It had its origin in that international comity which was one of the first fruits of
civilization, and it this age, when business intercourse and the process of accumulating property take but little notice of boundary
lines, the practical wisdom and justice of the rule is more apparent than ever. (Goodrich, Conflict of Laws, Sec. 164, pp. 442-443.)
Appellees argue that what Article 16 of the Civil Code of the Philippines pointed out as the national law is the internal law of California. But as above
explained the laws of California have prescribed two sets of laws for its citizens, one for residents therein and another for those domiciled in other
jurisdictions. Reason demands that We should enforce the California internal law prescribed for its citizens residing therein, and enforce the conflict of
laws rules for the citizens domiciled abroad. If we must enforce the law of California as in comity we are bound to go, as so declared in Article 16 of our
Civil Code, then we must enforce the law of California in accordance with the express mandate thereof and as above explained, i.e., apply the internal
law for residents therein, and its conflict-of-laws rule for those domiciled abroad.
It is argued on appellees' behalf that the clause "if there is no law to the contrary in the place where the property is situated" in Sec. 946 of the California
Civil Code refers to Article 16 of the Civil Code of the Philippines and that the law to the contrary in the Philippines is the provision in said Article 16 that
the national law of the deceased should govern. This contention can not be sustained. As explained in the various authorities cited above the national
law mentioned in Article 16 of our Civil Code is the law on conflict of laws in the California Civil Code, i.e., Article 946, which authorizes the reference or
return of the question to the law of the testator's domicile. The conflict of laws rule in California, Article 946, Civil Code, precisely refers back the case,
when a decedent is not domiciled in California, to the law of his domicile, the Philippines in the case at bar. The court of the domicile can not and should
not refer the case back to California; such action would leave the issue incapable of determination because the case will then be like a football, tossed
back and forth between the two states, between the country of which the decedent was a citizen and the country of his domicile. The Philippine court
must apply its own law as directed in the conflict of laws rule of the state of the decedent, if the question has to be decided, especially as the application
of the internal law of California provides no legitime for children while the Philippine law, Arts. 887(4) and 894, Civil Code of the Philippines, makes
natural children legally acknowledged forced heirs of the parent recognizing them.
The Philippine cases (In re Estate of Johnson, 39 Phil. 156; Riera vs. Palmaroli, 40 Phil. 105; Miciano vs. Brimo, 50 Phil. 867; Babcock Templeton vs.
Rider Babcock, 52 Phil. 130; and Gibbs vs. Government, 59 Phil. 293.) cited by appellees to support the decision can not possibly apply in the case at
bar, for two important reasons, i.e., the subject in each case does not appear to be a citizen of a state in the United States but with domicile in the
Philippines, and it does not appear in each case that there exists in the state of which the subject is a citizen, a law similar to or identical with Art. 946 of
the California Civil Code.
We therefore find that as the domicile of the deceased Christensen, a citizen of California, is the Philippines, the validity of the provisions of his will
depriving his acknowledged natural child, the appellant, should be governed by the Philippine Law, the domicile, pursuant to Art. 946 of the Civil Code of
California, not by the internal law of California..
WHEREFORE, the decision appealed from is hereby reversed and the case returned to the lower court with instructions that the partition be made as
the Philippine law on succession provides. Judgment reversed, with costs against appellees.
Padilla, Bautista Angelo, Concepcion, Reyes, Barrera, Paredes, Dizon, Regala and Makalintal, JJ., concur.
Bengzon, C.J., took no part.

The Lawphil Project - Arellano Law Foundation

7
G.R. No. L-23678 June 6, 1967
TESTATE ESTATE OF AMOS G. BELLIS, deceased.
PEOPLE'S BANK and TRUST COMPANY, executor.
MARIA CRISTINA BELLIS and MIRIAM PALMA BELLIS, oppositors-appellants,
vs.
EDWARD A. BELLIS, ET AL., heirs-appellees.
Vicente R. Macasaet and Jose D. Villena for oppositors appellants.
Paredes, Poblador, Cruz and Nazareno for heirs-appellees E. A. Bellis, et al.
Quijano and Arroyo for heirs-appellees W. S. Bellis, et al.
J. R. Balonkita for appellee People's Bank & Trust Company.
Ozaeta, Gibbs and Ozaeta for appellee A. B. Allsman.
BENGZON, J.P., J .:
This is a direct appeal to Us, upon a question purely of law, from an order of the Court of First Instance of Manila dated April 30, 1964, approving the
project of partition filed by the executor in Civil Case No. 37089 therein.1wph1.t
The facts of the case are as follows:
Amos G. Bellis, born in Texas, was "a citizen of the State of Texas and of the United States." By his first wife, Mary E. Mallen, whom he divorced, he had
five legitimate children: Edward A. Bellis, George Bellis (who pre-deceased him in infancy), Henry A. Bellis, Alexander Bellis and Anna Bellis Allsman; by
his second wife, Violet Kennedy, who survived him, he had three legitimate children: Edwin G. Bellis, Walter S. Bellis and Dorothy Bellis; and finally, he
had three illegitimate children: Amos Bellis, Jr., Maria Cristina Bellis and Miriam Palma Bellis.
On August 5, 1952, Amos G. Bellis executed a will in the Philippines, in which he directed that after all taxes, obligations, and expenses of administration
are paid for, his distributable estate should be divided, in trust, in the following order and manner: (a) $240,000.00 to his first wife, Mary E. Mallen; (b)
P120,000.00 to his three illegitimate children, Amos Bellis, Jr., Maria Cristina Bellis, Miriam Palma Bellis, or P40,000.00 each and (c) after the foregoing
two items have been satisfied, the remainder shall go to his seven surviving children by his first and second wives, namely: Edward A. Bellis, Henry A.
Bellis, Alexander Bellis and Anna Bellis Allsman, Edwin G. Bellis, Walter S. Bellis, and Dorothy E. Bellis, in equal shares.1wph1.t
Subsequently, or on July 8, 1958, Amos G. Bellis died a resident of San Antonio, Texas, U.S.A. His will was admitted to probate in the Court of First
Instance of Manila on September 15, 1958.
The People's Bank and Trust Company, as executor of the will, paid all the bequests therein including the amount of $240,000.00 in the form of shares
of stock to Mary E. Mallen and to the three (3) illegitimate children, Amos Bellis, Jr., Maria Cristina Bellis and Miriam Palma Bellis, various amounts
totalling P40,000.00 each in satisfaction of their respective legacies, or a total of P120,000.00, which it released from time to time according as the lower
court approved and allowed the various motions or petitions filed by the latter three requesting partial advances on account of their respective legacies.
On January 8, 1964, preparatory to closing its administration, the executor submitted and filed its "Executor's Final Account, Report of Administration
and Project of Partition" wherein it reported, inter alia, the satisfaction of the legacy of Mary E. Mallen by the delivery to her of shares of stock amounting
to $240,000.00, and the legacies of Amos Bellis, Jr., Maria Cristina Bellis and Miriam Palma Bellis in the amount of P40,000.00 each or a total of
P120,000.00. In the project of partition, the executor pursuant to the "Twelfth" clause of the testator's Last Will and Testament divided the residuary
estate into seven equal portions for the benefit of the testator's seven legitimate children by his first and second marriages.
On January 17, 1964, Maria Cristina Bellis and Miriam Palma Bellis filed their respective oppositions to the project of partition on the ground that they
were deprived of their legitimes as illegitimate children and, therefore, compulsory heirs of the deceased.
Amos Bellis, Jr. interposed no opposition despite notice to him, proof of service of which is evidenced by the registry receipt submitted on April 27, 1964
by the executor.
1

After the parties filed their respective memoranda and other pertinent pleadings, the lower court, on April 30, 1964, issued an order overruling the
oppositions and approving the executor's final account, report and administration and project of partition. Relying upon Art. 16 of the Civil Code, it
applied the national law of the decedent, which in this case is Texas law, which did not provide for legitimes.
Their respective motions for reconsideration having been denied by the lower court on June 11, 1964, oppositors-appellants appealed to this Court to
raise the issue of which law must apply Texas law or Philippine law.
In this regard, the parties do not submit the case on, nor even discuss, the doctrine of renvoi, applied by this Court in Aznar v. Christensen Garcia, L-
16749, January 31, 1963. Said doctrine is usually pertinent where the decedent is a national of one country, and a domicile of another. In the present
case, it is not disputed that the decedent was both a national of Texas and a domicile thereof at the time of his death.
2
So that even assuming Texas has
a conflict of law rule providing that the domiciliary system (law of the domicile) should govern, the same would not result in a reference back (renvoi) to
Philippine law, but would still refer to Texas law. Nonetheless, if Texas has a conflicts rule adopting the situs theory (lex rei sitae) calling for the
application of the law of the place where the properties are situated, renvoi would arise, since the properties here involved are found in the Philippines.
In the absence, however, of proof as to the conflict of law rule of Texas, it should not be presumed different from ours.
3
Appellants' position is therefore
not rested on the doctrine of renvoi. As stated, they never invoked nor even mentioned it in their arguments. Rather, they argue that their case falls
under the circumstances mentioned in the third paragraph of Article 17 in relation to Article 16 of the Civil Code.

8
Article 16, par. 2, and Art. 1039 of the Civil Code, render applicable the national law of the decedent, in intestate or testamentary successions, with
regard to four items: (a) the order of succession; (b) the amount of successional rights; (e) the intrinsic validity of the provisions of the will; and (d) the
capacity to succeed. They provide that
ART. 16. Real property as well as personal property is subject to the law of the country where it is situated.
However, intestate and testamentary successions, both with respect to the order of succession and to the amount of successional
rights and to the intrinsic validity of testamentary provisions, shall be regulated by the national law of the person whose succession
is under consideration, whatever may he the nature of the property and regardless of the country wherein said property may be
found.
ART. 1039. Capacity to succeed is governed by the law of the nation of the decedent.
Appellants would however counter that Art. 17, paragraph three, of the Civil Code, stating that
Prohibitive laws concerning persons, their acts or property, and those which have for their object public order, public policy and good
customs shall not be rendered ineffective by laws or judgments promulgated, or by determinations or conventions agreed upon in a
foreign country.
prevails as the exception to Art. 16, par. 2 of the Civil Code afore-quoted. This is not correct. Precisely, Congressdeleted the phrase, "notwithstanding
the provisions of this and the next preceding article" when they incorporated Art. 11 of the old Civil Code as Art. 17 of the new Civil Code, while
reproducing without substantial change the second paragraph of Art. 10 of the old Civil Code as Art. 16 in the new. It must have been their purpose to
make the second paragraph of Art. 16 a specific provision in itself which must be applied in testate and intestate succession. As further indication of this
legislative intent, Congress added a new provision, under Art. 1039, which decrees that capacity to succeed is to be governed by the national law of the
decedent.
It is therefore evident that whatever public policy or good customs may be involved in our System of legitimes, Congress has not intended to extend the
same to the succession of foreign nationals. For it has specifically chosen to leave, inter alia, the amount of successional rights, to the decedent's
national law. Specific provisions must prevail over general ones.
Appellants would also point out that the decedent executed two wills one to govern his Texas estate and the other his Philippine estate arguing
from this that he intended Philippine law to govern his Philippine estate. Assuming that such was the decedent's intention in executing a separate
Philippine will, it would not alter the law, for as this Court ruled in Miciano v. Brimo, 50 Phil. 867, 870, a provision in a foreigner's will to the effect that his
properties shall be distributed in accordance with Philippine law and not with his national law, is illegal and void, for his national law cannot be ignored in
regard to those matters that Article 10 now Article 16 of the Civil Code states said national law should govern.
The parties admit that the decedent, Amos G. Bellis, was a citizen of the State of Texas, U.S.A., and that under the laws of Texas, there are no forced
heirs or legitimes. Accordingly, since the intrinsic validity of the provision of the will and the amount of successional rights are to be determined under
Texas law, the Philippine law on legitimes cannot be applied to the testacy of Amos G. Bellis.
Wherefore, the order of the probate court is hereby affirmed in toto, with costs against appellants. So ordered.
Concepcion, C.J., Reyes, J.B.L., Dizon, Regala, Makalintal, Zaldivar, Sanchez and Castro, JJ., concur.

Footnotes
1
He later filed a motion praying that as a legal heir he be included in this case as one of the oppositors-appellants; to file or adopt
the opposition of his sisters to the project of partition; to submit his brief after paying his proportionate share in the expenses
incurred in the printing of the record on appeal; or to allow him to adopt the briefs filed by his sisters but this Court resolved to
deny the motion.
2
San Antonio, Texas was his legal residence.
3
Lim vs. Collector, 36 Phil. 472; In re Testate Estate of Suntay, 95 Phil. 500.

The Lawphil Project - Arellano Law Foundation


Republic of the Philippines
SUPREME COURT
Manila
EN BANC

9
G.R. No. L-14628 September 30, 1960
FRANCISCO HERMOSISIMA, petitioner,
vs.
THE HON. COURT OF APPEALS, ET AL., respondents.
Regino Hermosisima for petitioner.
F.P. Gabriel, Jr. for respondents.
CONCEPCION, J .:
An appeal by certiorari, taken by petitioner Francisco Hermosisima, from a decision of Court of Appeals modifying that of the Court of First Instance of
Cebu.
On October 4, 1954, Soledad Cagigas, hereinafter referred to as complaint, filed with said of her child, Chris Hermosisima, as natural child and moral
damages for alleged breach of promise. Petitioner admitted the paternity of child and expressed willingness to support the latter, but denied having ever
promised to marry the complainant. Upon her motion, said court ordered petitioner, on October 27, 1954, to pay, by way of alimony pendente lite,
P50.00 a month, which was, on February 16, 1955, reduced to P30.00 a month. In due course, later on, said court rendered a decision the dispositive
part of which reads:
WHEREFORE, judgment is hereby rendered, declaring the child, Chris Hermosisima, as the natural daughter of defendant, and
confirming the order pendente lite, ordering defendant to pay to the said child, through plaintiff, the sum of thirty pesos (P30.00),
payable on or before the fifth day of every month sentencing defendant to pay to plaintiff the sum of FOUR THOUSAND FIVE
HUNDRED PESOS (P4,500.00) for actual and compensatory damages; the sum of FIVE THOUSAND PESOS (P5,000.00) as moral
damages; and the further sum of FIVE HUNDRED PESOS (P500.00) as attorney's fees for plaintiff, with costs against defendant.
On appeal taken by petitioner, the Court of Appeals affirmed this decision, except as to the actual and compensatory damages and the moral damages,
which were increased to P5,614.25 and P7,000.00, respectively.
The main issue before us is whether moral damages are recoverable, under our laws, for breach of promise to marry. The pertinent facts are:
Complainant Soledad Cagigas, was born in July 1917. Since 1950, Soledad then a teacher in the Sibonga Provincial High School in Cebu, and
petitioner, who was almost ten (10) years younger than she, used to go around together and were regarded as engaged, although he had made no
promise of marriage prior thereto. In 1951, she gave up teaching and became a life insurance underwriter in the City of Cebu, where intimacy developed
among her and the petitioner, since one evening in 1953, when after coming from the movies, they had sexual intercourse in his cabin on board M/V
"Escao," to which he was then attached as apprentice pilot. In February 1954, Soledad advised petitioner that she was in the family way, whereupon he
promised to marry her. Their child, Chris Hermosisima, was born on June 17, 1954, in a private maternity and clinic. However, subsequently, or on July
24, 1954, defendant married one Romanita Perez. Hence, the present action, which was commenced on or about October 4, 1954.
Referring now to the issue above referred to, it will be noted that the Civil Code of Spain permitted the recovery of damages for breach to marry. Article
43 and 44 of said Code provides:
ART. 43. A mutual promise of marriage shall not give rise to an obligation to contract marriage. No court shall entertain any
complaint by which the enforcement of such promise is sought.
ART. 44. If the promise has been in a public or private instrument by an adult, or by a minor with the concurrence of the person
whose consent is necessary for the celebration of the marriage, or if the banns have been published, the one who without just cause
refuses to marry shall be obliged to reimburse the other for the expenses which he or she may have incurred by reason of the
promised marriage.
The action for reimbursement of expenses to which the foregoing article refers must be brought within one year, computed from the
day of the refusal to celebrate the marriage.
Inasmuch as these articles were never in force in the Philippines, this Court ruled in De Jesus vs. Syquia (58 Phil., 866), that "the action for breach of
promises to marry has no standing in the civil law, apart from the right to recover money or property advanced . . . upon the faith of such promise". The
Code Commission charged with the drafting of the Proposed Civil Code of the Philippines deem it best, however, to change the law thereon. We quote
from the report of the Code Commission on said Proposed Civil Code:
Articles 43 and 44 the Civil Code of 1889 refer to the promise of marriage. But these articles are not enforced in the Philippines. The
subject is regulated in the Proposed Civil Code not only as to the aspect treated of in said articles but also in other particulars. It is
advisable to furnish legislative solutions to some questions that might arise relative to betrothal. Among the provisions proposed are:
That authorizing the adjudication of moral damages, in case of breach of promise of marriage, and that creating liability for causing a
marriage engagement to be broken.1awphl.nt
Accordingly, the following provisions were inserted in said Proposed Civil Code, under Chapter I, Title III, Book I thereof:
Art. 56. A mutual promise to marry may be made expressly or impliedly.

10
Art. 57. An engagement to be married must be agreed directly by the future spouses.
Art. 58. A contract for a future marriage cannot, without the consent of the parent or guardian, be entered into by a male between
the ages of sixteen and twenty years or by a female between the ages of sixteen and eighteen years. Without such consent of the
parents or guardian, the engagement to marry cannot be the basis of a civil action for damages in case of breach of the promise.
Art. 59. A promise to marry when made by a female under the age of fourteen years is not civilly actionable, even though approved
by the parent or guardian.
Art. 60. In cases referred to in the proceeding articles, the criminal and civil responsibility of a male for seduction shall not be
affected.
Art. 61. No action for specific performance of a mutual promise to marry may be brought.
Art. 62. An action for breach of promise to marry may be brought by the aggrieved party even though a minor without the assistance
of his parent or guardian. Should the minor refuse to bring suit, the parent or guardian may institute the action.
Art. 63. Damages for breach of promise to marry shall include not only material and pecuniary losses but also compensation for
mental and moral suffering.
Art. 64. Any person, other than a rival, the parents, guardians and grandparents, of the affianced parties, who cause a marriage
engagement to be broken shall be liable for damages, both material and moral, to the engaged person who is rejected.
Art. 65. In case of breach of promise to marry, the party breaking the engagement shall be obliged to return what he or she has
received from the other as gift on account of the promise of the marriage.
These article were, however, eliminated in Congress. The reason therefor are set forth in the report of the corresponding Senate Committee, from which
we quote:
The elimination of this Chapter is proposed. That breach of promise to marry is not actionable has been definitely decide in the case of De Jesus vs.
Syquia, 58 Phil., 866. The history of breach of promise suit in the United States and in England has shown that no other action lends itself more readily
to abuse by designing women and unscrupulous men. It is this experience which has led to the abolition of the rights of action in the so-called Balm suit
in many of the American States.
See statutes of:
Florida 1945 pp. 1342 1344
Maryland 1945 pp. 1759 1762
Nevada 1943 p. 75
Maine 1941 pp. 140 141
New Hampshire 1941 p. 223
California 1939 p. 1245
Massachusetts 1938 p. 326
Indiana 1936 p. 1009
Michigan 1935 p. 201
New York 1935
Pennsylvania p. 450
The Commission perhaps though that it has followed the more progression trend in legislation when it provided for breach of
promise to marry suits. But it is clear that the creation of such causes of action at a time when so many States, in consequence of
years of experience are doing away with them, may well prove to be a step in the wrong direction. (Congressional Record, Vol. IV,
No. 79, Thursday, May 19, 1949, p. 2352.)
The views thus expressed were accepted by both houses of Congress. In the light of the clear and manifest intent of our law making body not to
sanction actions for breach of promise to marry, the award of moral damages made by the lower courts is, accordingly, untenable. The Court of Appeals
said award:
Moreover, it appearing that because of defendant-appellant's seduction power, plaintiff-appellee, overwhelmed by her love for him
finally yielded to his sexual desires in spite of her age and self-control, she being a woman after all, we hold that said defendant-
appellant is liable for seduction and, therefore, moral damages may be recovered from him under the provision of Article 2219,
paragraph 3, of the new Civil Code.
Apart from the fact that the general tenor of said Article 2219, particularly the paragraphs preceding and those following the one cited by the Court of
Appeals, and the language used in said paragraph strongly indicates that the "seduction" therein contemplated is the crime punished as such in Article
as such in Article 337 and 338 of the Revised Penal Code, which admittedly does not exist in the present case, we find ourselves unable to say that
petitioner is morally guilty of seduction, not only because he is approximately ten (10) years younger than the complainant who around thirty-six (36)
years of age, and as highly enlightened as a former high school teacher and a life insurance agent are supposed to be when she became intimate
with petitioner, then a mere apprentice pilot, but, also, because, the court of first instance found that, complainant "surrendered herself" to petitioner
because, "overwhelmed by her love" for him, she "wanted to bind" "by having a fruit of their engagement even before they had the benefit of clergy."

11
The court of first instance sentenced petitioner to pay the following: (1) a monthly pension of P30.00 for the support of the child: (2) P4,500, representing
the income that complainant had allegedly failed to earn during her pregnancy and shortly after the birth of the child, as actual and compensation
damages; (3) P5,000, as moral damages; and (4) P500.00, as attorney's fees. The Court of Appeals added to the second item the sum of P1,114.25
consisting of P144.20, for hospitalization and medical attendance, in connection with the parturiation, and the balance representing expenses incurred to
support the child and increased the moral damages to P7,000.00.
With the elimination of this award for damages, the decision of the Court of Appeals is hereby affirmed, therefore, in all other respects, without special
pronouncement as to cost in this instance. It is so ordered.
Paras, C.J., Bengzon, Padilla, Bautista Angelo, Labrador, Reyes, J.B.L., Barrera, Gutierrez David, Paredes and Dizon, JJ., concur.

G.R. No. L-20089 December 26, 1964
BEATRIZ P. WASSMER, plaintiff-appellee,
vs.
FRANCISCO X. VELEZ, defendant-appellant.
Jalandoni & Jamir for defendant-appellant.
Samson S. Alcantara for plaintiff-appellee.
BENGZON, J.P., J .:
The facts that culminated in this case started with dreams and hopes, followed by appropriate planning and serious endeavors, but terminated in
frustration and, what is worse, complete public humiliation.
Francisco X. Velez and Beatriz P. Wassmer, following their mutual promise of love, decided to get married and set September 4, 1954 as the big day.
On September 2, 1954 Velez left this note for his bride-to-be:
Dear Bet
Will have to postpone wedding My mother opposes it. Am leaving on the Convair today.
Please do not ask too many people about the reason why That would only create a scandal.
Paquing
But the next day, September 3, he sent her the following telegram:
NOTHING CHANGED REST ASSURED RETURNING VERY SOON APOLOGIZE MAMA PAPA LOVE .
PAKING
Thereafter Velez did not appear nor was he heard from again.
Sued by Beatriz for damages, Velez filed no answer and was declared in default. Plaintiff adduced evidence before the clerk of court as commissioner,
and on April 29, 1955, judgment was rendered ordering defendant to pay plaintiff P2,000.00 as actual damages; P25,000.00 as moral and exemplary
damages; P2,500.00 as attorney's fees; and the costs.
On June 21, 1955 defendant filed a "petition for relief from orders, judgment and proceedings and motion for new trial and reconsideration." Plaintiff
moved to strike it cut. But the court, on August 2, 1955, ordered the parties and their attorneys to appear before it on August 23, 1955 "to explore at this
stage of the proceedings the possibility of arriving at an amicable settlement." It added that should any of them fail to appear "the petition for relief and
the opposition thereto will be deemed submitted for resolution."
On August 23, 1955 defendant failed to appear before court. Instead, on the following day his counsel filed a motion to defer for two weeks the resolution
on defendants petition for relief. The counsel stated that he would confer with defendant in Cagayan de Oro City the latter's residence on the
possibility of an amicable element. The court granted two weeks counted from August 25, 1955.
Plaintiff manifested on June 15, 1956 that the two weeks given by the court had expired on September 8, 1955 but that defendant and his counsel had
failed to appear.
Another chance for amicable settlement was given by the court in its order of July 6, 1956 calling the parties and their attorneys to appear on July 13,
1956. This time. however, defendant's counsel informed the court that chances of settling the case amicably were nil.

12
On July 20, 1956 the court issued an order denying defendant's aforesaid petition. Defendant has appealed to this Court. In his petition of June 21, 1955
in the court a quo defendant alleged excusable negligence as ground to set aside the judgment by default. Specifically, it was stated that defendant filed
no answer in the belief that an amicable settlement was being negotiated.
A petition for relief from judgment on grounds of fraud, accident, mistake or excusable negligence, must be duly supported by an affidavit of merits
stating facts constituting a valid defense. (Sec. 3, Rule 38, Rules of Court.) Defendant's affidavit of merits attached to his petition of June 21, 1955
stated: "That he has a good and valid defense against plaintiff's cause of action, his failure to marry the plaintiff as scheduled having been due to
fortuitous event and/or circumstances beyond his control." An affidavit of merits like this stating mere conclusions or opinions instead of facts is not valid.
(Cortes vs. Co Bun Kim, L-3926, Oct. 10, 1951; Vaswani vs. P. Tarrachand Bros., L-15800, December 29, 1960.)
Defendant, however, would contend that the affidavit of merits was in fact unnecessary, or a mere surplusage, because the judgment sought to be set
aside was null and void, it having been based on evidence adduced before the clerk of court. In Province of Pangasinan vs. Palisoc, L-16519, October
30, 1962, this Court pointed out that the procedure of designating the clerk of court as commissioner to receive evidence is sanctioned by Rule 34 (now
Rule 33) of the Rules of Court. Now as to defendant's consent to said procedure, the same did not have to be obtained for he was declared in default
and thus had no standing in court (Velez vs. Ramas, 40 Phil. 787; Alano vs. Court of First Instance, L-14557, October 30, 1959).
In support of his "motion for new trial and reconsideration," defendant asserts that the judgment is contrary to law. The reason given is that "there is no
provision of the Civil Code authorizing" an action for breach of promise to marry. Indeed, our ruling in Hermosisima vs. Court of Appeals (L-14628, Sept.
30, 1960), as reiterated in Estopa vs. Biansay (L-14733, Sept. 30, 1960), is that "mere breach of a promise to marry" is not an actionable wrong. We
pointed out that Congress deliberately eliminated from the draft of the new Civil Code the provisions that would have it so.
It must not be overlooked, however, that the extent to which acts not contrary to law may be perpetrated with impunity, is not limitless for Article 21 of
said Code provides that "any person who wilfully causes loss or injury to another in a manner that is contrary to morals, good customs or public policy
shall compensate the latter for the damage."
The record reveals that on August 23, 1954 plaintiff and defendant applied for a license to contract marriage, which was subsequently issued (Exhs. A,
A-1). Their wedding was set for September 4, 1954. Invitations were printed and distributed to relatives, friends and acquaintances (Tsn., 5; Exh. C). The
bride-to-be's trousseau, party drsrses and other apparel for the important occasion were purchased (Tsn., 7-8). Dresses for the maid of honor and the
flower girl were prepared. A matrimonial bed, with accessories, was bought. Bridal showers were given and gifts received (Tsn., 6; Exh. E). And then,
with but two days before the wedding, defendant, who was then 28 years old,: simply left a note for plaintiff stating: "Will have to postpone wedding
My mother opposes it ... " He enplaned to his home city in Mindanao, and the next day, the day before the wedding, he wired plaintiff: "Nothing changed
rest assured returning soon." But he never returned and was never heard from again.
Surely this is not a case of mere breach of promise to marry. As stated, mere breach of promise to marry is not an actionable wrong. But to formally set
a wedding and go through all the above-described preparation and publicity, only to walk out of it when the matrimony is about to be solemnized, is quite
different. This is palpably and unjustifiably contrary to good customs for which defendant must be held answerable in damages in accordance with Article
21 aforesaid.
Defendant urges in his afore-stated petition that the damages awarded were excessive. No question is raised as to the award of actual damages. What
defendant would really assert hereunder is that the award of moral and exemplary damages, in the amount of P25,000.00, should be totally eliminated.
Per express provision of Article 2219 (10) of the New Civil Code, moral damages are recoverable in the cases mentioned in Article 21 of said Code. As
to exemplary damages, defendant contends that the same could not be adjudged against him because under Article 2232 of the New Civil Code the
condition precedent is that "the defendant acted in a wanton, fraudulent, reckless, oppressive, or malevolent manner." The argument is devoid of merit
as under the above-narrated circumstances of this case defendant clearly acted in a "wanton ... , reckless [and] oppressive manner." This Court's
opinion, however, is that considering the particular circumstances of this case, P15,000.00 as moral and exemplary damages is deemed to be a
reasonable award.
PREMISES CONSIDERED, with the above-indicated modification, the lower court's judgment is hereby affirmed, with costs.
Bengzon, C.J., Bautista Angelo, Reyes, J.B.L., Barrera, Paredes, Dizon, Regala, Makalintal, and Zaldivar, JJ.,concur.

The Lawphil Project - Arellano Law Foundation



Republic of the Philippines
SUPREME COURT
Manila
THIRD DIVISION

G.R. No. 97336 February 19, 1993

13
GASHEM SHOOKAT BAKSH, petitioner,
vs.
HON. COURT OF APPEALS and MARILOU T. GONZALES, respondents.
Public Attorney's Office for petitioner.
Corleto R. Castro for private respondent.

DAVIDE, JR., J .:
This is an appeal by certiorari under Rule 45 of the Rules of Court seeking to review and set aside the Decision
1
of the respondent Court of Appeals in
CA-G.R. CV No. 24256 which affirmed in toto the 16 October 1939 Decision of Branch 38 (Lingayen) of the Regional Trial Court (RTC) of Pangasinan in
Civil Case No. 16503. Presented is the issue of whether or not damages may be recovered for a breach of promise to marry on the basis of Article 21 of
the Civil Code of the Philippines.
The antecedents of this case are not complicated:
On 27 October 1987, private respondent, without the assistance of counsel, filed with the aforesaid trial court a complaint
2
for damages against the
petitioner for the alleged violation of their agreement to get married. She alleges in said complaint that: she is twenty-two (22) years old, single, Filipino
and a pretty lass of good moral character and reputation duly respected in her community; petitioner, on the other hand, is an Iranian citizen residing at
the Lozano Apartments, Guilig, Dagupan City, and is an exchange student taking a medical course at the Lyceum Northwestern Colleges in Dagupan
City; before 20 August 1987, the latter courted and proposed to marry her; she accepted his love on the condition that they would get married; they
therefore agreed to get married after the end of the school semester, which was in October of that year; petitioner then visited the private respondent's
parents in Baaga, Bugallon, Pangasinan to secure their approval to the marriage; sometime in 20 August 1987, the petitioner forced her to live with him
in the Lozano Apartments; she was a virgin before she began living with him; a week before the filing of the complaint, petitioner's attitude towards her
started to change; he maltreated and threatened to kill her; as a result of such maltreatment, she sustained injuries; during a confrontation with a
representative of the barangay captain of Guilig a day before the filing of the complaint, petitioner repudiated their marriage agreement and asked her
not to live with him anymore and; the petitioner is already married to someone living in Bacolod City. Private respondent then prayed for judgment
ordering the petitioner to pay her damages in the amount of not less than P45,000.00, reimbursement for actual expenses amounting to P600.00,
attorney's fees and costs, and granting her such other relief and remedies as may be just and equitable. The complaint was docketed as Civil Case No.
16503.
In his Answer with Counterclaim,
3
petitioner admitted only the personal circumstances of the parties as averred in the complaint and denied the rest of
the allegations either for lack of knowledge or information sufficient to form a belief as to the truth thereof or because the true facts are those alleged as
his Special and Affirmative Defenses. He thus claimed that he never proposed marriage to or agreed to be married with the private respondent; he
neither sought the consent and approval of her parents nor forced her to live in his apartment; he did not maltreat her, but only told her to stop coming to
his place because he discovered that she had deceived him by stealing his money and passport; and finally, no confrontation took place with a
representative of the barangay captain. Insisting, in his Counterclaim, that the complaint is baseless and unfounded and that as a result thereof, he was
unnecessarily dragged into court and compelled to incur expenses, and has suffered mental anxiety and a besmirched reputation, he prayed for an
award of P5,000.00 for miscellaneous expenses and P25,000.00 as moral damages.
After conducting a pre-trial on 25 January 1988, the trial court issued a Pre-Trial Order
4
embodying the stipulated facts which the parties had agreed
upon, to wit:
1. That the plaintiff is single and resident (sic) of Baaga, Bugallon, Pangasinan, while the defendant is single, Iranian
citizen and resident (sic) of Lozano Apartment, Guilig, Dagupan City since September 1, 1987 up to the present;
2. That the defendant is presently studying at Lyceum Northwestern, Dagupan City, College of Medicine, second year
medicine proper;
3. That the plaintiff is (sic) an employee at Mabuhay Luncheonette , Fernandez Avenue, Dagupan City since July, 1986 up
to the present and a (sic) high school graduate;
4. That the parties happened to know each other when the manager of the Mabuhay Luncheonette, Johhny Rabino
introduced the defendant to the plaintiff on August 3, 1986.
After trial on the merits, the lower court, applying Article 21 of the Civil Code, rendered on 16 October 1989 a decision
5
favoring the private respondent.
The petitioner was thus ordered to pay the latter damages and attorney's fees; the dispositive portion of the decision reads:
IN THE LIGHT of the foregoing consideration, judgment is hereby rendered in favor of the plaintiff and against the
defendant.
1. Condemning (sic) the defendant to pay the plaintiff the sum of twenty thousand (P20,000.00) pesos as moral damages.
2. Condemning further the defendant to play the plaintiff the sum of three thousand (P3,000.00) pesos as atty's fees and
two thousand (P2,000.00) pesos at (sic) litigation expenses and to pay the costs.

14
3. All other claims are denied.
6

The decision is anchored on the trial court's findings and conclusions that (a) petitioner and private respondent were lovers, (b) private respondent is not
a woman of loose morals or questionable virtue who readily submits to sexual advances, (c) petitioner, through machinations, deceit and false
pretenses, promised to marry private respondent, d) because of his persuasive promise to marry her, she allowed herself to be deflowered by him, (e) by
reason of that deceitful promise, private respondent and her parents in accordance with Filipino customs and traditions made some preparations
for the wedding that was to be held at the end of October 1987 by looking for pigs and chickens, inviting friends and relatives and contracting sponsors,
(f) petitioner did not fulfill his promise to marry her and (g) such acts of the petitioner, who is a foreigner and who has abused Philippine hospitality, have
offended our sense of morality, good customs, culture and traditions. The trial court gave full credit to the private respondent's testimony because, inter
alia, she would not have had the temerity and courage to come to court and expose her honor and reputation to public scrutiny and ridicule if her claim
was false.
7

The above findings and conclusions were culled from the detailed summary of the evidence for the private respondent in the foregoing decision,
digested by the respondent Court as follows:
According to plaintiff, who claimed that she was a virgin at the time and that she never had a boyfriend before, defendant
started courting her just a few days after they first met. He later proposed marriage to her several times and she accepted
his love as well as his proposal of marriage on August 20, 1987, on which same day he went with her to her hometown of
Baaga, Bugallon, Pangasinan, as he wanted to meet her parents and inform them of their relationship and their intention
to get married. The photographs Exhs. "A" to "E" (and their submarkings) of defendant with members of plaintiff's family or
with plaintiff, were taken that day. Also on that occasion, defendant told plaintiffs parents and brothers and sisters that he
intended to marry her during the semestral break in October, 1987, and because plaintiff's parents thought he was good
and trusted him, they agreed to his proposal for him to marry their daughter, and they likewise allowed him to stay in their
house and sleep with plaintiff during the few days that they were in Bugallon. When plaintiff and defendant later returned
to Dagupan City, they continued to live together in defendant's apartment. However, in the early days of October, 1987,
defendant would tie plaintiff's hands and feet while he went to school, and he even gave her medicine at 4 o'clock in the
morning that made her sleep the whole day and night until the following day. As a result of this live-in relationship, plaintiff
became pregnant, but defendant gave her some medicine to abort the fetus. Still plaintiff continued to live with defendant
and kept reminding him of his promise to marry her until he told her that he could not do so because he was already
married to a girl in Bacolod City. That was the time plaintiff left defendant, went home to her parents, and thereafter
consulted a lawyer who accompanied her to the barangay captain in Dagupan City. Plaintiff, her lawyer, her godmother,
and a barangay tanod sent by the barangay captain went to talk to defendant to still convince him to marry plaintiff, but
defendant insisted that he could not do so because he was already married to a girl in Bacolod City, although the truth, as
stipulated by the parties at the pre-trial, is that defendant is still single.
Plaintiff's father, a tricycle driver, also claimed that after defendant had informed them of his desire to marry Marilou, he
already looked for sponsors for the wedding, started preparing for the reception by looking for pigs and chickens, and
even already invited many relatives and friends to the forthcoming wedding.
8

Petitioner appealed the trial court's decision to the respondent Court of Appeals which docketed the case as CA-G.R. CV No. 24256. In his Brief,
9
he
contended that the trial court erred (a) in not dismissing the case for lack of factual and legal basis and (b) in ordering him to pay moral damages,
attorney's fees, litigation expenses and costs.
On 18 February 1991, respondent Court promulgated the challenged decision
10
affirming in toto the trial court's ruling of 16 October 1989. In sustaining
the trial court's findings of fact, respondent Court made the following analysis:
First of all, plaintiff, then only 21 years old when she met defendant who was already 29 years old at the time, does not
appear to be a girl of loose morals. It is uncontradicted that she was a virgin prior to her unfortunate experience with
defendant and never had boyfriend. She is, as described by the lower court, a barrio lass "not used and accustomed to
trend of modern urban life", and certainly would (sic) not have allowed
"herself to be deflowered by the defendant if there was no persuasive promise made by the defendant to marry her." In
fact, we agree with the lower court that plaintiff and defendant must have been sweethearts or so the plaintiff must have
thought because of the deception of defendant, for otherwise, she would not have allowed herself to be photographed
with defendant in public in so (sic) loving and tender poses as those depicted in the pictures Exhs. "D" and "E". We cannot
believe, therefore, defendant's pretense that plaintiff was a nobody to him except a waitress at the restaurant where he
usually ate. Defendant in fact admitted that he went to plaintiff's hometown of Baaga, Bugallon, Pangasinan, at least
thrice; at (sic) the town fiesta on February 27, 1987 (p. 54, tsn May 18, 1988), at (sic) a beach party together with the
manager and employees of the Mabuhay Luncheonette on March 3, 1987 (p. 50, tsn id.), and on April 1, 1987 when he
allegedly talked to plaintiff's mother who told him to marry her daughter (pp. 55-56, tsn id.). Would defendant have left
Dagupan City where he was involved in the serious study of medicine to go to plaintiff's hometown in Baaga, Bugallon,
unless there was (sic) some kind of special relationship between them? And this special relationship must indeed have led
to defendant's insincere proposal of marriage to plaintiff, communicated not only to her but also to her parents, and (sic)
Marites Rabino, the owner of the restaurant where plaintiff was working and where defendant first proposed marriage to
her, also knew of this love affair and defendant's proposal of marriage to plaintiff, which she declared was the reason why
plaintiff resigned from her job at the restaurant after she had accepted defendant's proposal (pp. 6-7, tsn March 7, 1988).
Upon the other hand, appellant does not appear to be a man of good moral character and must think so low and have so
little respect and regard for Filipino women that he openly admitted that when he studied in Bacolod City for several years
where he finished his B.S. Biology before he came to Dagupan City to study medicine, he had a common-law wife in
Bacolod City. In other words, he also lived with another woman in Bacolod City but did not marry that woman, just like
what he did to plaintiff. It is not surprising, then, that he felt so little compunction or remorse in pretending to love and
promising to marry plaintiff, a young, innocent, trustful country girl, in order to satisfy his lust on her.
11


15
and then concluded:
In sum, we are strongly convinced and so hold that it was defendant-appellant's fraudulent and deceptive protestations of
love for and promise to marry plaintiff that made her surrender her virtue and womanhood to him and to live with him on
the honest and sincere belief that he would keep said promise, and it was likewise these (sic) fraud and deception on
appellant's part that made plaintiff's parents agree to their daughter's living-in with him preparatory to their supposed
marriage. And as these acts of appellant are palpably and undoubtedly against morals, good customs, and public policy,
and are even gravely and deeply derogatory and insulting to our women, coming as they do from a foreigner who has
been enjoying the hospitality of our people and taking advantage of the opportunity to study in one of our institutions of
learning, defendant-appellant should indeed be made, under Art. 21 of the Civil Code of the Philippines, to compensate
for the moral damages and injury that he had caused plaintiff, as the lower court ordered him to do in its decision in this
case.
12

Unfazed by his second defeat, petitioner filed the instant petition on 26 March 1991; he raises therein the single issue of whether or not Article 21 of the
Civil Code applies to the case at bar.
13

It is petitioner's thesis that said Article 21 is not applicable because he had not committed any moral wrong or injury or violated any good custom or
public policy; he has not professed love or proposed marriage to the private respondent; and he has never maltreated her. He criticizes the trial court for
liberally invoking Filipino customs, traditions and culture, and ignoring the fact that since he is a foreigner, he is not conversant with such Filipino
customs, traditions and culture. As an Iranian Moslem, he is not familiar with Catholic and Christian ways. He stresses that even if he had made a
promise to marry, the subsequent failure to fulfill the same is excusable or tolerable because of his Moslem upbringing; he then alludes to the Muslim
Code which purportedly allows a Muslim to take four (4) wives and concludes that on the basis thereof, the trial court erred in ruling that he does not
posses good moral character. Moreover, his controversial "common law life" is now his legal wife as their marriage had been solemnized in civil
ceremonies in the Iranian Embassy. As to his unlawful cohabitation with the private respondent, petitioner claims that even if responsibility could be
pinned on him for the live-in relationship, the private respondent should also be faulted for consenting to an illicit arrangement. Finally, petitioner
asseverates that even if it was to be assumed arguendo that he had professed his love to the private respondent and had also promised to marry her,
such acts would not be actionable in view of the special circumstances of the case. The mere breach of promise is not actionable.
14

On 26 August 1991, after the private respondent had filed her Comment to the petition and the petitioner had filed his Reply thereto, this Court gave due
course to the petition and required the parties to submit their respective Memoranda, which they subsequently complied with.
As may be gleaned from the foregoing summation of the petitioner's arguments in support of his thesis, it is clear that questions of fact, which boil down
to the issue of the credibility of witnesses, are also raised. It is the rule in this jurisdiction that appellate courts will not disturb the trial court's findings as
to the credibility of witnesses, the latter court having heard the witnesses and having had the opportunity to observe closely their deportment and
manner of testifying, unless the trial court had plainly overlooked facts of substance or value which, if considered, might affect the result of the case.
15

Petitioner has miserably failed to convince Us that both the appellate and trial courts had overlooked any fact of substance or values which could alter
the result of the case.
Equally settled is the rule that only questions of law may be raised in a petition for review on certiorari under Rule 45 of the Rules of Court. It is not the
function of this Court to analyze or weigh all over again the evidence introduced by the parties before the lower court. There are, however, recognized
exceptions to this rule. Thus, inMedina vs. Asistio, Jr.,
16
this Court took the time, again, to enumerate these exceptions:
xxx xxx xxx
(1) When the conclusion is a finding grounded entirely on speculation, surmises or conjectures (Joaquin v. Navarro, 93
Phil. 257 [1953]); (2) When the inference made is manifestly mistaken, absurb or impossible (Luna v. Linatok, 74 Phil. 15
[1942]); (3) Where there is a grave abuse of discretion (Buyco v. People, 95 Phil. 453 [1955]); (4) When the judgment is
based on a misapprehension of facts (Cruz v. Sosing,
L-4875, Nov. 27, 1953); (5) When the findings of fact are conflicting (Casica v. Villaseca, L-9590 Ap. 30, 1957; unrep.) (6)
When the Court of Appeals, in making its findings, went beyond the issues of the case and the same is contrary to the
admissions of both appellate and appellee (Evangelista v. Alto Surety and Insurance Co., 103 Phil. 401 [1958]);
(7) The findings of the Court of Appeals are contrary to those of the trial court (Garcia v. Court of Appeals, 33 SCRA 622
[1970]; Sacay v. Sandiganbayan, 142 SCRA 593 [1986]); (8) When the findings of fact are conclusions without citation of
specific evidence on which they are based (Ibid.,); (9) When the facts set forth in the petition as well as in the petitioners
main and reply briefs are not disputed by the respondents (Ibid.,); and (10) The finding of fact of the Court of Appeals is
premised on the supposed absence of evidence and is contradicted by the evidence on record (Salazar v. Gutierrez, 33
SCRA 242 [1970]).
Petitioner has not endeavored to joint out to Us the existence of any of the above quoted exceptions in this case. Consequently, the factual findings of
the trial and appellate courts must be respected.
And now to the legal issue.
The existing rule is that a breach of promise to marry per se is not an actionable wrong.
17
Congress deliberately eliminated from the draft of the New
Civil Code the provisions that would have made it so. The reason therefor is set forth in the report of the Senate Committees on the Proposed Civil
Code, from which We quote:
The elimination of this chapter is proposed. That breach of promise to marry is not actionable has been definitely decided
in the case of De Jesus vs. Syquia.
18
The history of breach of promise suits in the United States and in England has
shown that no other action lends itself more readily to abuse by designing women and unscrupulous men. It is this

16
experience which has led to the abolition of rights of action in the so-called Heart Balm suits in many of the American
states. . . .
19

This notwithstanding, the said Code contains a provision, Article 21, which is designed to expand the concept of torts or quasi-delict in this jurisdiction by
granting adequate legal remedy for the untold number of moral wrongs which is impossible for human foresight to specifically enumerate and punish in
the statute books.
20

As the Code Commission itself stated in its Report:
But the Code Commission had gone farther than the sphere of wrongs defined or determined by positive law. Fully
sensible that there are countless gaps in the statutes, which leave so many victims of moral wrongs helpless, even though
they have actually suffered material and moral injury, the Commission has deemed it necessary, in the interest of justice,
to incorporate in the proposed Civil Code the following rule:
Art. 23. Any person who wilfully causes loss or injury to another in a manner that is contrary to
morals, good customs or public policy shall compensate the latter for the damage.
An example will illustrate the purview of the foregoing norm: "A" seduces the nineteen-year old daughter of "X". A promise
of marriage either has not been made, or can not be proved. The girl becomes pregnant. Under the present laws, there is
no crime, as the girl is above nineteen years of age. Neither can any civil action for breach of promise of marriage be filed.
Therefore, though the grievous moral wrong has been committed, and though the girl and family have suffered
incalculable moral damage, she and her parents cannot bring action for damages. But under the proposed article, she and
her parents would have such a right of action.
Thus at one stroke, the legislator, if the forgoing rule is approved, would vouchsafe adequate legal remedy for that untold
number of moral wrongs which it is impossible for human foresight to provide for specifically in the statutes.
21

Article 2176 of the Civil Code, which defines a quasi-delict thus:
Whoever by act or omission causes damage to another, there being fault or negligence, is obliged to pay for the damage
done. Such fault or negligence, if there is no pre-existing contractual relation between the parties, is called a quasi-
delict and is governed by the provisions of this Chapter.
is limited to negligent acts or omissions and excludes the notion of willfulness or intent. Quasi-delict, known in Spanish legal
treatises as culpa aquiliana, is a civil law concept while torts is an Anglo-American or common law concept. Torts is much broader
than culpa aquiliana because it includes not only negligence, but international criminal acts as well such as assault and battery,
false imprisonment and deceit. In the general scheme of the Philippine legal system envisioned by the Commission responsible for
drafting the New Civil Code, intentional and malicious acts, with certain exceptions, are to be governed by the Revised Penal Code
while negligent acts or omissions are to be covered by Article 2176 of the Civil Code.
22
In between these opposite spectrums are
injurious acts which, in the absence of Article 21, would have been beyond redress. Thus, Article 21 fills that vacuum. It is even
postulated that together with Articles 19 and 20 of the Civil Code, Article 21 has greatly broadened the scope of the law on civil
wrongs; it has become much more supple and adaptable than the Anglo-American law on torts.
23

In the light of the above laudable purpose of Article 21, We are of the opinion, and so hold, that where a man's promise to marry is in fact the proximate
cause of the acceptance of his love by a woman and his representation to fulfill that promise thereafter becomes the proximate cause of the giving of
herself unto him in a sexual congress, proof that he had, in reality, no intention of marrying her and that the promise was only a subtle scheme or
deceptive device to entice or inveigle her to accept him and to obtain her consent to the sexual act, could justify the award of damages pursuant to
Article 21 not because of such promise to marry but because of the fraud and deceit behind it and the willful injury to her honor and reputation which
followed thereafter. It is essential, however, that such injury should have been committed in a manner contrary to morals, good customs or public policy.
In the instant case, respondent Court found that it was the petitioner's "fraudulent and deceptive protestations of love for and promise to marry plaintiff
that made her surrender her virtue and womanhood to him and to live with him on the honest and sincere belief that he would keep said promise, and it
was likewise these fraud and deception on appellant's part that made plaintiff's parents agree to their daughter's living-in with him preparatory to their
supposed marriage."
24
In short, the private respondent surrendered her virginity, the cherished possession of every single Filipina, not because of lust
but because of moral seduction the kind illustrated by the Code Commission in its example earlier adverted to. The petitioner could not be held liable
for criminal seduction punished under either Article 337 or Article 338 of the Revised Penal Code because the private respondent was above eighteen
(18) years of age at the time of the seduction.
Prior decisions of this Court clearly suggest that Article 21 may be applied in a breach of promise to marry where the woman is a victim of moral
seduction. Thus, in Hermosisima vs. Court of Appeals,
25
this Court denied recovery of damages to the woman because:
. . . we find ourselves unable to say that petitioner is morally guilty of seduction, not only because he is approximately ten
(10) years younger than the complainant who was around thirty-six (36) years of age, and as highly enlightened as a
former high school teacher and a life insurance agent are supposed to be when she became intimate with petitioner,
then a mere apprentice pilot, but, also, because the court of first instance found that, complainant "surrendered herself" to
petitioner because, "overwhelmed by her love" for him, she "wanted to bind" him by having a fruit of their engagement
even before they had the benefit of clergy.
In Tanjanco vs. Court of Appeals,
26
while this Court likewise hinted at possible recovery if there had been moral seduction, recovery was eventually
denied because We were not convinced that such seduction existed. The following enlightening disquisition and conclusion were made in the said case:

17
The Court of Appeals seem to have overlooked that the example set forth in the Code Commission's memorandum refers
to a tort upon a minor who had been seduced. The essential feature is seduction, that in law is more than mere sexual
intercourse, or a breach of a promise of marriage; it connotes essentially the idea of deceit, enticement, superior power or
abuse of confidence on the part of the seducer to which the woman has yielded (U.S. vs. Buenaventura, 27 Phil. 121;
U.S. vs. Arlante, 9 Phil. 595).
It has been ruled in the Buenaventura case (supra) that
To constitute seduction there must in all cases be some sufficient promise or inducementand the
woman must yield because of the promise or other inducement. If she consents merely from carnal
lust and the intercourse is from mutual desire, there is no seduction (43 Cent. Dig. tit. Seduction, par.
56) She must be induced to depart from the path of virtue by the use of some species of arts,
persuasions and wiles, which are calculated to have and do have that effect, and which result in her
person to ultimately submitting her person to the sexual embraces of her seducer (27 Phil. 123).
And in American Jurisprudence we find:
On the other hand, in an action by the woman, the enticement, persuasion or deception is the
essence of the injury; and a mere proof of intercourse is insufficient to warrant a recovery.
Accordingly it is not seduction where the willingness arises out of sexual desire of curiosity of the
female, and the defendant merely affords her the needed opportunity for the commission of the act. It
has been emphasized that to allow a recovery in all such cases would tend to the demoralization of
the female sex, and would be a reward for unchastity by which a class of adventuresses would be
swift to profit. (47 Am. Jur. 662)
xxx xxx xxx
Over and above the partisan allegations, the fact stand out that for one whole year, from 1958 to 1959, the plaintiff-
appellee, a woman of adult age, maintain intimate sexual relations with appellant, with repeated acts of intercourse. Such
conduct is incompatible with the idea of seduction. Plainly there is here voluntariness and mutual passion; for had the
appellant been deceived, had she surrendered exclusively because of the deceit, artful persuasions and wiles of the
defendant, she would not have again yielded to his embraces, much less for one year, without exacting early fulfillment of
the alleged promises of marriage, and would have cut short all sexual relations upon finding that defendant did not intend
to fulfill his defendant did not intend to fulfill his promise. Hence, we conclude that no case is made under article 21 of the
Civil Code, and no other cause of action being alleged, no error was committed by the Court of First Instance in
dismissing the complaint.
27

In his annotations on the Civil Code,
28
Associate Justice Edgardo L. Paras, who recently retired from this Court, opined that in a breach of promise to
marry where there had been carnal knowledge, moral damages may be recovered:
. . . if there be criminal or moral seduction, but not if the intercourse was due to mutual lust. (Hermosisima vs. Court of
Appeals,
L-14628, Sept. 30, 1960; Estopa vs. Piansay, Jr., L-14733, Sept. 30, 1960; Batarra vs. Marcos, 7 Phil. 56 (sic); Beatriz
Galang vs. Court of Appeals, et al., L-17248, Jan. 29, 1962). (In other words, if the CAUSE be the promise to marry, and
the EFFECT be the carnal knowledge, there is a chance that there was criminal or moral seduction, hence recovery of
moral damages will prosper. If it be the other way around, there can be no recovery of moral damages, because here
mutual lust has intervened). . . .
together with "ACTUAL damages, should there be any, such as the expenses for the wedding presentations (See Domalagon v.
Bolifer, 33 Phil. 471).
Senator Arturo M. Tolentino
29
is also of the same persuasion:
It is submitted that the rule in Batarra vs. Marcos,
30
still subsists, notwithstanding the incorporation of the present
article
31
in the Code. The example given by the Code Commission is correct, if there was seduction, not necessarily in the
legal sense, but in the vulgar sense of deception. But when the sexual act is accomplished without any deceit or qualifying
circumstance of abuse of authority or influence, but the woman, already of age, has knowingly given herself to a man, it
cannot be said that there is an injury which can be the basis for indemnity.
But so long as there is fraud, which is characterized by willfulness (sic), the action lies. The court, however, must weigh
the degree of fraud, if it is sufficient to deceive the woman under the circumstances, because an act which would deceive
a girl sixteen years of age may not constitute deceit as to an experienced woman thirty years of age. But so long as there
is a wrongful act and a resulting injury, there should be civil liability, even if the act is not punishable under the criminal law
and there should have been an acquittal or dismissal of the criminal case for that reason.
We are unable to agree with the petitioner's alternative proposition to the effect that granting, for argument's sake, that he did promise to marry the
private respondent, the latter is nevertheless also at fault. According to him, both parties are in pari delicto; hence, pursuant to Article 1412(1) of the Civil
Code and the doctrine laid down inBatarra vs. Marcos,
32
the private respondent cannot recover damages from the petitioner. The latter even goes as far
as stating that if the private respondent had "sustained any injury or damage in their relationship, it is primarily because of her own doing,
33
for:

18
. . . She is also interested in the petitioner as the latter will become a doctor sooner or later. Take notice that she is a plain
high school graduate and a mere employee . . . (Annex "C") or a waitress (TSN, p. 51, January 25, 1988) in a
luncheonette and without doubt, is in need of a man who can give her economic security. Her family is in dire need of
financial assistance. (TSN, pp. 51-53, May 18, 1988). And this predicament prompted her to accept a proposition that may
have been offered by the petitioner.
34

These statements reveal the true character and motive of the petitioner. It is clear that he harbors a condescending, if not sarcastic, regard for the
private respondent on account of the latter's ignoble birth, inferior educational background, poverty and, as perceived by him, dishonorable employment.
Obviously then, from the very beginning, he was not at all moved by good faith and an honest motive. Marrying with a woman so circumstances could
not have even remotely occurred to him. Thus, his profession of love and promise to marry were empty words directly intended to fool, dupe, entice,
beguile and deceive the poor woman into believing that indeed, he loved her and would want her to be his life's partner. His was nothing but pure lust
which he wanted satisfied by a Filipina who honestly believed that by accepting his proffer of love and proposal of marriage, she would be able to enjoy
a life of ease and security. Petitioner clearly violated the Filipino's concept of morality and brazenly defied the traditional respect Filipinos have for their
women. It can even be said that the petitioner committed such deplorable acts in blatant disregard of Article 19 of the Civil Code which directs every
person to act with justice, give everyone his due and observe honesty and good faith in the exercise of his rights and in the performance of his
obligations.
No foreigner must be allowed to make a mockery of our laws, customs and traditions.
The pari delicto rule does not apply in this case for while indeed, the private respondent may not have been impelled by the purest of intentions, she
eventually submitted to the petitioner in sexual congress not out of lust, but because of moral seduction. In fact, it is apparent that she had qualms of
conscience about the entire episode for as soon as she found out that the petitioner was not going to marry her after all, she left him. She is not,
therefore, in pari delicto with the petitioner. Pari delicto means "in equal fault; in a similar offense or crime; equal in guilt or in legal fault."
35
At most, it
could be conceded that she is merely in delicto.
Equity often interferes for the relief of the less guilty of the parties, where his transgression has been brought about by the
imposition of undue influence of the party on whom the burden of the original wrong principally rests, or where his consent
to the transaction was itself procured by
fraud.
36

In Mangayao vs. Lasud,
37
We declared:
Appellants likewise stress that both parties being at fault, there should be no action by one against the other (Art. 1412,
New Civil Code). This rule, however, has been interpreted as applicable only where the fault on both sides is, more or
less, equivalent. It does not apply where one party is literate or intelligent and the other one is not. (c.f. Bough vs.
Cantiveros, 40 Phil. 209).
We should stress, however, that while We find for the private respondent, let it not be said that this Court condones the deplorable behavior of her
parents in letting her and the petitioner stay together in the same room in their house after giving approval to their marriage. It is the solemn duty of
parents to protect the honor of their daughters and infuse upon them the higher values of morality and dignity.
WHEREFORE, finding no reversible error in the challenged decision, the instant petition is hereby DENIED, with costs against the petitioner.
SO ORDERED.
Feliciano, Bidin, Romero and Melo, JJ., concur.
Gutierrez, Jr., J., is on leave.

[G.R. No. 140420. February 15 , 2001]
SERGIO AMONOY, petitioner, vs. Spouses JOSE GUTIERREZ and ANGELA FORNILDA, respondents.
D E C I S I O N
PANGANIBAN, J .:
Damnum absque injuria. Under this principle, the legitimate exercise of a persons rights, even if it causes loss to another, does not automatically
result in an actionable injury. The law does not prescribe a remedy for the loss. This principle does not, however, apply when there is an abuse of a
persons right, or when the exercise of this right is suspended or extinguished pursuant to a court order. Indeed, in the availment of ones rights, one
must act with justice, give others their due, and observe honesty and good faith.
The Case


19
Before us is a Petition for Review under Rule 45 of the Rules of Court, assailing the April 21, 1999 Decision
[1]
of the Court of Appeals (CA) in CA-
GR CV No. 41451, which set aside the judgment
[2]
of the Regional Trial Court (RTC) of Tanay, Rizal. The RTC had earlier dismissed the Complaint for
damages filed by herein respondents against petitioner. The dispositive portion of the challenged CA Decision reads as follows:
WHEREFORE, the appealed Decision is SET ASIDE, and in its stead judgment is rendered ordering the defendant-appellee Sergio Amonoy to pay the
plaintiffs-appellants Bruno and Bernardina Gutierrez as actual damages the sum of [t]wo [h]undred [f]ifty [t]housand [p]esos (P250,000.00).
[3]

Likewise assailed is the October 19, 1999 CA Resolution,
[4]
which denied the Motion for Reconsideration.
The Facts

The appellate court narrated the factual antecedents of this case as follows:
This case had its roots in Special Proceedings No. 3103 of Branch I of the CFI of Pasig, Rizal, for the settlement of the estate of the deceased Julio
Cantolos, involving six (6) parcels of land situated in Tanay, Rizal. Amonoy was the counsel of therein Francisca Catolos, Agnes Catolos, Asuncion
Pasamba and Alfonso Formilda. On 12 January 1965, the Project of Partition submitted was approved and x x x two (2) of the said lots were
adjudicated to Asuncion Pasamba and Alfonso Formilda. The attorneys fees charged by Amonoy was P27,600.00 and on 20 January 1965 Asuncion
Pasamba and Alfonso Formilda executed a deed of real estate mortgage on the said two (2) lots adjudicated to them, in favor of Amonoy to secure the
payment of his attorneys fees. But it was only on 6 August 1969 after the taxes had been paid, the claims settled and the properties adjudicated, that
the estate was declared closed and terminated.
Asuncion Pasamba died on 24 February 1969 while Alfonso Fornilda passed away on 2 July 1969. Among the heirs of the latter was his daughter,
plaintiff-appellant Angela Gutierrez.
Because his attorneys fees thus secured by the two lots were not paid, on 21 January 1970 Amonoy filed for their foreclosure in Civil Case No. 12726
entitled Sergio Amonoy vs. Heirs of Asuncion Pasamba and Heirs of Alfonso Fornilda before the CFI of Pasig, Rizal, and this was assigned to Branch
VIII. The heirs opposed, contending that the attorneys fees charged [were] unconscionable and that the agreed sum was only P11,695.92. But on 28
September 1972 judgment was rendered in favor of Amonoy requiring the heirs to pay within 90 days the P27,600.00 secured by the mortgage,
P11,880.00 as value of the harvests, and P9,645.00 as another round of attorneys fees. Failing in that, the two (2) lots would be sold at public auction.
They failed to pay. On 6 February 1973, the said lots were foreclosed and on 23 March 1973 the auction sale was held where Amonoy was the highest
bidder at P23,760.00. On 2 May 1973 his bid was judicially confirmed. A deficiency was claimed and to satisfy it another execution sale was conducted,
and again the highest bidder was Amonoy at P12,137.50.
Included in those sold was the lot on which the Gutierrez spouses had their house.
More than a year after the Decision in Civil Case No. 12726 was rendered, the said decedents heirs filed on 19 December 1973 before the CFI of
Pasig, Rizal[,] Civil Case No. 18731 entitled Maria Penano, et al vs. Sergio Amonoy, et al, a suit for the annulment thereof. The case was dismissed by
the CFI on 7 November 1977, and this was affirmed by the Court of Appeals on 22 July 1981.
Thereafter, the CFI on 25 July 1985 issued a Writ of Possession and pursuant to which a notice to vacate was made on 26 August 1985. On Amonoys
motion of 24 April 1986, the Orders of 25 April 1986 and 6 May 1986 were issued for the demolition of structures in the said lots, including the house of
the Gutierrez spouses.
On 27 September 1985 the petition entitled David Fornilda, et al vs Branch 164 RTC IVth Pasig, Deputy Sheriff Joaquin Antonil and Atty. Sergio
Amonoy, G.R. No. L-72306, was filed before the Supreme Court. Among the petitioners was the plaintiff-appellant Angela Gutierrez. On a
twin Musiyun (Mahigpit na Musiyon Para Papanagutin Kaugnay ng Paglalapastangan, and Musiyung Makahingi ng Utos sa Pagpapapigil ng
Pagpapagiba at Pananagutin sa Paglalapastangan) with full titles as fanciful and elongated as their Petisyung (Petisyung Makapagsuri Taglay and
Pagpigil ng Utos), a temporary restraining order was granted on 2 June 1986 enjoining the demolition of the petitioners houses.
Then on 5 October 1988 a Decision was rendered in the said G.R. No. L-72306 disposing that:
WHEREFORE, Certiorari is granted; the Order of respondent Trial Court, dated 25 July 1985, granting a Writ of Possession, as well as its Orders, dated
25 April 1986 and 16 May 1986, directing and authorizing respondent Sheriff to demolish the houses of petitioners Angela and Leocadia Fornilda are
hereby set aside, and the Temporary Restraining Order heretofore issued, is made permanent. The six (6) parcels of land herein controverted are
hereby ordered returned to petitioners unless some of them have been conveyed to innocent third persons.
[5]

But by the time the Supreme Court promulgated the above-mentioned Decision, respondents house had already been destroyed, supposedly in
accordance with a Writ of Demolition ordered by the lower court.
Thus, a Complaint for damages in connection with the destruction of their house was filed by respondents against petitioner before the RTC on
December 15, 1989.
In its January 27, 1993 Decision, the RTC dismissed respondents suit. On appeal, the CA set aside the lower courts ruling and ordered petitioner
to pay respondents P250,000 as actual damages. Petitioner then filed a Motion for Reconsideration, which was also denied.
Hence, this recourse.
[6]


20
The Issue

In his Memorandum,
[7]
petitioner submits this lone issue for our consideration:
Whether or not the Court of Appeals was correct in deciding that the petitioner [was] liable to the respondents for damages
[8]

The Courts Ruling

The Petition has no merit.
Main Issue: Petitioners Liability

Well-settled is the maxim that damage resulting from the legitimate exercise of a persons rights is a loss without injury -- damnum absque injuria -
- for which the law gives no remedy.
[9]
In other words, one who merely exercises ones rights does no actionable injury and cannot be held liable for
damages.
Petitioner invokes this legal precept in arguing that he is not liable for the demolition of respondents house. He maintains that he was merely
acting in accordance with the Writ of Demolition ordered by the RTC.
We reject this submission. Damnum absque injuria finds no application to this case.
True, petitioner commenced the demolition of respondents house on May 30, 1986 under the authority of a Writ of Demolition issued by the
RTC. But the records show that a Temporary Restraining Order (TRO), enjoining the demolition of respondents house, was issued by the Supreme
Court on June 2, 1986. The CA also found, based on the Certificate of Service of the Supreme Court process server, that a copy of the TRO was served
on petitioner himself on June 4, 1986.
Petitioner, however, did not heed the TRO of this Court. We agree with the CA that he unlawfully pursued the demolition of respondents house
well until the middle of 1987. This is clear from Respondent Angela Gutierrezs testimony. The appellate court quoted the following pertinent portion
thereof:
[10]

Q. On May 30, 1986, were they able to destroy your house?
A. Not all, a certain portion only
x x x x x x x x x
Q. Was your house completely demolished?
A. No, sir.
Q. How about the following day?
A. It was completely demolished
x x x x x x x x x
Q. Until when[,] Mrs. Witness?
A. Until 1987.
Q. About what month of 1987?
A. Middle of the year.
Q. Can you tell the Honorable Court who completed the demolition?
A. The men of Fiscal Amonoy.
[11]

The foregoing disproves the claim of petitioner that the demolition, which allegedly commenced only on May 30, 1986, was completed the
following day. It likewise belies his allegation that the demolitions had already ceased when he received notice of the TRO.
Although the acts of petitioner may have been legally justified at the outset, their continuation after the issuance of the TRO amounted to an
insidious abuse of his right. Indubitably, his actions were tainted with bad faith. Had he not insisted on completing the demolition, respondents would
not have suffered the loss that engendered the suit before the RTC. Verily, his acts constituted not only an abuse of a right, but an invalid exercise of a
right that had been suspended when he received the TRO from this Court on June 4, 1986. By then, he was no longer entitled to proceed with the
demolition.
A commentator on this topic explains:
The exercise of a right ends when the right disappears, and it disappears when it is abused, especially to the prejudice of others. The mask of a right
without the spirit of justice which gives it life, is repugnant to the modern concept of social law. It cannot be said that a person exercises a right when he
unnecessarily prejudices another x x x. Over and above the specific precepts of positive law are the supreme norms of justice x x x; and he who violates
them violates the law. For this reason, it is not permissible to abuse our rights to prejudice others.
[12]


21
Likewise, in Albenson Enterprises Corp. v. CA,
[13]
the Court discussed the concept of abuse of rights as follows:
Article 19, known to contain what is commonly referred to as the principle of abuse of rights, sets certain standards which may be observed not only in
the exercise of ones rights but also in the performance of ones duties. These standards are the following: to act with justice; to give everyone his due;
and to observe honesty and good faith. The law, therefore, recognizes the primordial limitation on all rights: that in their exercise, the norms of human
conduct set forth in Article 19 must be observed. A right, though by itself legal because recognized or granted by law as such, may nevertheless
become the source of some illegality. When a right is exercised in a manner which does not conform with norms enshrined in Article 19 and results in
damage to another, a legal wrong is thereby committed for which the wrongdoer must be held responsible x x x.
Clearly then, the demolition of respondents house by petitioner, despite his receipt of the TRO, was not only an abuse but also an unlawful
exercise of such right. In insisting on his alleged right, he wantonly violated this Courts Order and wittingly caused the destruction of respondents
house.
Obviously, petitioner cannot invoke damnum absque injuria, a principle premised on the valid exercise of a right.
[14]
Anything less or beyond such
exercise will not give rise to the legal protection that the principle accords. And when damage or prejudice to another is occasioned thereby, liability
cannot be obscured, much less abated.
In the ultimate analysis, petitioners liability is premised on the obligation to repair or to make whole the damage caused to another by reason of
ones act or omission, whether done intentionally or negligently and whether or not punishable by law.
[15]

WHEREFORE, the Petition is DENIED and the appealed Decision AFFIRMED. Costs against petitioner.
SO ORDERED.
Melo, (Chairman), Vitug, Gonzaga-Reyes, and Sandoval-Gutierrez, JJ., concur.

SECOND DIVISION
G.R. No. 154259 February 28, 2005
NIKKO HOTEL MANILA GARDEN and RUBY LIM, petitioners,
vs.
ROBERTO REYES, a.k.a. "AMAY BISAYA," respondent.
D E C I S I O N
CHICO-NAZARIO, J .:
In this petition for review on certiorari, petitioners Nikko Hotel Manila Garden (Hotel Nikko)
1
and Ruby Lim assail the Decision
2
of the Court of Appeals
dated 26 November 2001 reversing the Decision
3
of the Regional Trial Court (RTC) of Quezon City, Branch 104, as well as the Resolution
4
of the Court
of Appeals dated 09 July 2002 which denied petitioners motion for reconsideration.
The cause of action before the trial court was one for damages brought under the human relations provisions of the New Civil Code. Plaintiff thereat
(respondent herein) Roberto Reyes, more popularly known by the screen name "Amay Bisaya," alleged that at around 6:00 oclock in the evening of 13
October 1994, while he was having coffee at the lobby of Hotel Nikko,
5
he was spotted by his friend of several years, Dr. Violeta Filart, who then
approached him.
6
Mrs. Filart invited him to join her in a party at the hotels penthouse in celebration of the natal day of the hotels manager, Mr.
Masakazu Tsuruoka.
7
Mr. Reyes asked if she could vouch for him for which she replied: "of course."
8
Mr. Reyes then went up with the party of Dr. Filart
carrying the basket of fruits which was the latters present for the celebrant.
9
At the penthouse, they first had their picture taken with the celebrant after
which Mr. Reyes sat with the party of Dr. Filart.
10
After a couple of hours, when the buffet dinner was ready, Mr. Reyes lined-up at the buffet table but, to
his great shock, shame and embarrassment, he was stopped by petitioner herein, Ruby Lim, who claimed to speak for Hotel Nikko as Executive
Secretary thereof.
11
In a loud voice and within the presence and hearing of the other guests who were making a queue at the buffet table, Ruby Lim told
him to leave the party ("huwag ka nang kumain, hindi ka imbitado, bumaba ka na lang").
12
Mr. Reyes tried to explain that he was invited by Dr.
Filart.
13
Dr. Filart, who was within hearing distance, however, completely ignored him thus adding to his shame and humiliation.
14
Not long after, while he
was still recovering from the traumatic experience, a Makati policeman approached and asked him to step out of the hotel.
15
Like a common criminal, he
was escorted out of the party by the policeman.
16
Claiming damages, Mr. Reyes asked for One Million Pesos actual damages, One Million Pesos moral
and/or exemplary damages and Two Hundred Thousand Pesos attorneys fees.
17

Ruby Lim, for her part, admitted having asked Mr. Reyes to leave the party but not under the ignominious circumstance painted by the latter. Ms. Lim
narrated that she was the Hotels Executive Secretary for the past twenty (20) years.
18
One of her functions included organizing the birthday party of the
hotels former General Manager, Mr. Tsuruoka.
19
The year 1994 was no different. For Mr. Tsuruokas party, Ms. Lim generated an exclusive guest list
and extended invitations accordingly.
20
The guest list was limited to approximately sixty (60) of Mr. Tsuruokas closest friends and some hotel employees
and that Mr. Reyes was not one of those invited.
21
At the party, Ms. Lim first noticed Mr. Reyes at the bar counter ordering a drink.
22
Mindful of Mr.
Tsuruokas wishes to keep the party intimate, Ms. Lim approached Mr. Boy Miller, the "captain waiter," to inquire as to the presence of Mr. Reyes who
was not invited.
23
Mr. Miller replied that he saw Mr. Reyes with the group of Dr. Filart.
24
As Dr. Filart was engaged in conversation with another guest and
as Ms. Lim did not want to interrupt, she inquired instead from the sister of Dr. Filart, Ms. Zenaida Fruto, who told her that Dr. Filart did not invite Mr.
Reyes.
25
Ms. Lim then requested Ms. Fruto to tell Mr. Reyes to leave the party as he was not invited.
26
Mr. Reyes, however, lingered prompting Ms. Lim
to inquire from Ms. Fruto who said that Mr. Reyes did not want to leave.
27
When Ms. Lim turned around, she saw Mr. Reyes conversing with a Captain
Batung whom she later approached.
28
Believing that Captain Batung and Mr. Reyes knew each other, Ms. Lim requested from him the same favor from
Ms. Fruto,i.e., for Captain Batung to tell Mr. Reyes to leave the party as he was not invited.
29
Still, Mr. Reyes lingered. When Ms. Lim spotted Mr. Reyes
by the buffet table, she decided to speak to him herself as there were no other guests in the immediate vicinity.
30
However, as Mr. Reyes was already
helping himself to the food, she decided to wait.
31
When Mr. Reyes went to a corner and started to eat, Ms. Lim approached him and said: "alam ninyo,
hindo ho kayo dapat nandito. Pero total nakakuha na ho kayo ng pagkain, ubusin na lang ninyo at pagkatapos kung pwede lang po umalis na

22
kayo."
32
She then turned around trusting that Mr. Reyes would show enough decency to leave, but to her surprise, he began screaming and making a
big scene, and even threatened to dump food on her.
33
1awphi1.nt
Dr. Violeta Filart, the third defendant in the complaint before the lower court, also gave her version of the story to the effect that she never invited Mr.
Reyes to the party.
34
According to her, it was Mr. Reyes who volunteered to carry the basket of fruits intended for the celebrant as he was likewise going
to take the elevator, not to the penthouse but to Altitude 49.
35
When they reached the penthouse, she reminded Mr. Reyes to go down as he was not
properly dressed and was not invited.
36
All the while, she thought that Mr. Reyes already left the place, but she later saw him at the bar talking to Col.
Batung.
37
Then there was a commotion and she saw Mr. Reyes shouting.
38
She ignored Mr. Reyes.
39
She was embarrassed and did not want the
celebrant to think that she invited him.
40

After trial on the merits, the court a quo dismissed the complaint,
41
giving more credence to the testimony of Ms. Lim that she was discreet in asking Mr.
Reyes to leave the party. The trial court likewise ratiocinated that Mr. Reyes assumed the risk of being thrown out of the party as he was uninvited:
Plaintiff had no business being at the party because he was not a guest of Mr. Tsuruoka, the birthday celebrant. He assumed the risk of being asked to
leave for attending a party to which he was not invited by the host. Damages are pecuniary consequences which the law imposes for the breach of some
duty or the violation of some right. Thus, no recovery can be had against defendants Nikko Hotel and Ruby Lim because he himself was at fault
(Garciano v. Court of Appeals, 212 SCRA 436). He knew that it was not the party of defendant Violeta Filart even if she allowed him to join her and took
responsibility for his attendance at the party. His action against defendants Nikko Hotel and Ruby Lim must therefore fail.
42

On appeal, the Court of Appeals reversed the ruling of the trial court as it found more commanding of belief the testimony of Mr. Reyes that Ms. Lim
ordered him to leave in a loud voice within hearing distance of several guests:
In putting appellant in a very embarrassing situation, telling him that he should not finish his food and to leave the place within the hearing distance of
other guests is an act which is contrary to morals, good customs . . ., for which appellees should compensate the appellant f or the damage suffered by
the latter as a consequence therefore (Art. 21, New Civil Code). The liability arises from the acts which are in themselves legal or not prohibited, but
contrary to morals or good customs. Conversely, even in the exercise of a formal right, [one] cannot with impunity intentionally cause damage to another
in a manner contrary to morals or good customs.
43

The Court of Appeals likewise ruled that the actuation of Ms. Lim in approaching several people to inquire into the presence of Mr. Reyes exposed the
latter to ridicule and was uncalled for as she should have approached Dr. Filart first and both of them should have talked to Mr. Reyes in private:
Said acts of appellee Lim are uncalled for. What should have been done by appellee Lim was to approach appellee Mrs. Filart and together they should
have told appellant Reyes in private that the latter should leave the party as the celebrant only wanted close friends around. It is necessary that Mrs.
Filart be the one to approach appellant because it was she who invited appellant in that occasion. Were it not for Mrs. Filarts invitation, appellant could
not have suffered such humiliation. For that, appellee Filart is equally liable.
. . .
The acts of [appellee] Lim are causes of action which are predicated upon mere rudeness or lack of consideration of one person, which calls not only
protection of human dignity but respect of such dignity. Under Article 20 of the Civil Code, every person who violates this duty becomes liable for
damages, especially if said acts were attended by malice or bad faith. Bad faith does not simply connote bad judgment or simple negligence. It imports a
dishonest purpose or some moral obliquity and conscious doing of a wrong, a breach of a known duty to some motive or interest or ill-will that partakes
of the nature of fraud (Cojuangco, Jr. v. CA, et al., 309 SCRA 603).
44

Consequently, the Court of Appeals imposed upon Hotel Nikko, Ruby Lim and Dr. Violeta Filart the solidary obligation to pay Mr. Reyes (1) exemplary
damages in the amount of Two Hundred Thousand Pesos (P200,000); (2) moral damages in the amount of Two Hundred Thousand Pesos (P200,000);
and (3) attorneys fees in the amount of Ten Thousand Pesos (P10,000).
45
On motion for reconsideration, the Court of Appeals affirmed its earlier
decision as the argument raised in the motion had "been amply discussed and passed upon in the decision sought to be reconsidered."
46

Thus, the instant petition for review. Hotel Nikko and Ruby Lim contend that the Court of Appeals seriously erred in
I.
NOT APPLYING THE DOCTRINE OF VOLENTI NON FIT INJURIA CONSIDERING THAT BY ITS OWN FINDINGS, AMAY BISAYA WAS A GATE-
CRASHER
II.
HOLDING HOTEL NIKKO AND RUBY LIM JOINTLY AND SEVERALLY LIABLE WITH DR. FILART FOR DAMAGES SINCE BY ITS OWN RULING,
AMAY BISAYA "COULD NOT HAVE SUFFERED SUCH HUMILIATION," "WERE IT NOT FOR DR. FILARTS INVITATION"
III.
DEPARTING FROM THE FINDINGS OF FACT OF THE TRIAL COURT AS REGARDS THE CIRCUMSTANCES THAT ALLEGEDLY CAUSED THE
HUMILIATION OF AMAY BISAYA
IV.

23
IN CONCLUDING THAT AMAY BISAYA WAS TREATED UNJUSTLY BECAUSE OF HIS POVERTY, CONSIDERING THAT THIS WAS NEVER AN
ISSUE AND NO EVIDENCE WAS PRESENTED IN THIS REGARD
V.
IN FAILING TO PASS UPON THE ISSUE ON THE DEFECTS OF THE APPELLANTS BRIEF, THEREBY DEPARTING FROM THE ACCEPTED
AND USUAL COURSE OF JUDICIAL PROCEEDINGS
Petitioners Lim and Hotel Nikko contend that pursuant to the doctrine of volenti non fit injuria, they cannot be made liable for damages as respondent
Reyes assumed the risk of being asked to leave (and being embarrassed and humiliated in the process) as he was a "gate-crasher."
The doctrine of volenti non fit injuria ("to which a person assents is not esteemed in law as injury"
47
) refers to self-inflicted injury
48
or to the consent to
injury
49
which precludes the recovery of damages by one who has knowingly and voluntarily exposed himself to danger, even if he is not negligent in
doing so.
50
As formulated by petitioners, however, this doctrine does not find application to the case at bar because even if respondent Reyes assumed
the risk of being asked to leave the party, petitioners, under Articles 19 and 21 of the New Civil Code, were still under obligation to treat him fairly in
order not to expose him to unnecessary ridicule and shame.
Thus, the threshold issue is whether or not Ruby Lim acted abusively in asking Roberto Reyes, a.k.a. "Amay Bisaya," to l eave the party where he was
not invited by the celebrant thereof thereby becoming liable under Articles 19 and 21 of the Civil Code. Parenthetically, and if Ruby Lim were so liable,
whether or not Hotel Nikko, as her employer, is solidarily liable with her.
As the trial court and the appellate court reached divergent and irreconcilable conclusions concerning the same facts and evi dence of the case, this
Court is left without choice but to use its latent power to review such findings of facts. Indeed, the general rule is that we are not a trier of facts as our
jurisdiction is limited to reviewing and revising errors of law.
51
One of the exceptions to this general rule, however, obtains herein as the findings of the
Court of Appeals are contrary to those of the trial court.
52
The lower court ruled that Ms. Lim did not abuse her right to ask Mr. Reyes to leave the party
as she talked to him politely and discreetly. The appellate court, on the other hand, held that Ms. Lim is liable for damages as she needlessly
embarrassed Mr. Reyes by telling him not to finish his food and to leave the place within hearing distance of the other guests. Both courts, however,
were in agreement that it was Dr. Filarts invitation that brought Mr. Reyes to the party.
The consequential question then is: Which version is credible?
From an in depth review of the evidence, we find more credible the lower courts findings of fact.
First, let us put things in the proper perspective.
We are dealing with a formal party in a posh, five-star hotel,
53
for-invitation-only, thrown for the hotels former Manager, a Japanese national. Then came
a person who was clearly uninvited (by the celebrant)
54
and who could not just disappear into the crowd as his face is known by many, being an actor.
While he was already spotted by the organizer of the party, Ms. Lim, the very person who generated the guest list, it did not yet appear that the celebrant
was aware of his presence. Ms. Lim, mindful of the celebrants instruction to keep the party intimate, would naturally want to get rid of the "gate-crasher"
in the most hush-hush manner in order not to call attention to a glitch in an otherwise seamless affair and, in the process, risk the displeasure of the
celebrant, her former boss. To unnecessarily call attention to the presence of Mr. Reyes would certainly reflect badly on Ms. Lims ability to follow the
instructions of the celebrant to invite only his close friends and some of the hotels personnel. Mr. Reyes, upon whom the burden rests to prove that
indeed Ms. Lim loudly and rudely ordered him to leave, could not offer any satisfactory explanation why Ms. Lim would do that and risk ruining a formal
and intimate affair. On the contrary, Mr. Reyes, on cross-examination, had unwittingly sealed his fate by admitting that when Ms. Lim talked to him, she
was very close. Close enough for him to kiss:
Q: And, Mr. Reyes, you testified that Miss Lim approached you while you were at the buffet table? How close was she when she
approached you?
A: Very close because we nearly kissed each other.
Q: And yet, she shouted for you to go down? She was that close and she shouted?
A: Yes. She said, "wag kang kumain, hindi ka imbitado dito, bumaba ka na lang."
Q: So, you are testifying that she did this in a loud voice?
. . .
A: Yes. If it is not loud, it will not be heard by many.
55

In the absence of any proof of motive on the part of Ms. Lim to humiliate Mr. Reyes and expose him to ridicule and shame, it is highly unlikely that she
would shout at him from a very close distance. Ms. Lim having been in the hotel business for twenty years wherein being polite and discreet are virtues
to be emulated, the testimony of Mr. Reyes that she acted to the contrary does not inspire belief and is indeed incredible. Thus, the lower court was
correct in observing that

24
Considering the closeness of defendant Lim to plaintiff when the request for the latter to leave the party was made such that they nearly kissed each
other, the request was meant to be heard by him only and there could have been no intention on her part to cause embarrassment to him. It was
plaintiffs reaction to the request that must have made the other guests aware of what transpired between them. . .
Had plaintiff simply left the party as requested, there was no need for the police to take him out.
56

Moreover, another problem with Mr. Reyess version of the story is that it is unsupported. It is a basic rule in civil cases that he who alleges proves. Mr.
Reyes, however, had not presented any witness to back his story up. All his witnesses Danny Rodinas, Pepito Guerrero and Alexander Silva - proved
only that it was Dr. Filart who invited him to the party.
57

Ms. Lim, not having abused her right to ask Mr. Reyes to leave the party to which he was not invited, cannot be made liable to pay for damages under
Articles 19 and 21 of the Civil Code. Necessarily, neither can her employer, Hotel Nikko, be held liable as its liability springs from that of its employee.
58

Article 19, known to contain what is commonly referred to as the principle of abuse of rights,
59
is not a panacea for all human hurts and social
grievances. Article 19 states:
Art. 19. Every person must, in the exercise of his rights and in the performance of his duties, act with justice, give everyone his due, and observe
honesty and good faith.1awphi1.nt
Elsewhere, we explained that when "a right is exercised in a manner which does not conform with the norms enshrined in Article 19 and results in
damage to another, a legal wrong is thereby committed for which the wrongdoer must be responsible."
60
The object of this article, therefore, is to set
certain standards which must be observed not only in the exercise of ones rights but also in the performance of ones duties.
61
These standards are the
following: act with justice, give everyone his due and observe honesty and good faith.
62
Its antithesis, necessarily, is any act evincing bad faith or intent
to injure. Its elements are the following: (1) There is a legal right or duty; (2) which is exercised in bad faith; (3) for the sole intent of prejudicing or injuring
another.
63
When Article 19 is violated, an action for damages is proper under Articles 20 or 21 of the Civil Code. Article 20 pertains to damages arising
from a violation of law
64
which does not obtain herein as Ms. Lim was perfectly within her right to ask Mr. Reyes to leave. Article 21, on the other hand,
states:
Art. 21. Any person who willfully causes loss or injury to another in a manner that is contrary to morals, good customs or public policy shall compensate
the latter for the damage.
Article 21
65
refers to acts contra bonus mores and has the following elements: (1) There is an act which is legal; (2) but which is contrary to morals, good
custom, public order, or public policy; and (3) it is done with intent to injure.
66

A common theme runs through Articles 19 and 21,
67
and that is, the act complained of must be intentional.
68

As applied to herein case and as earlier discussed, Mr. Reyes has not shown that Ms. Lim was driven by animosity against him. These two people did
not know each other personally before the evening of 13 October 1994, thus, Mr. Reyes had nothing to offer for an explanation for Ms. Lims alleged
abusive conduct except the statement that Ms. Lim, being "single at 44 years old," had a "very strong bias and prejudice agai nst (Mr. Reyes) possibly
influenced by her associates in her work at the hotel with foreign businessmen."
69
The lameness of this argument need not be belabored. Suffice it to
say that a complaint based on Articles 19 and 21 of the Civil Code must necessarily fail if it has nothing to recommend it but innuendos and conjectures.
Parenthetically, the manner by which Ms. Lim asked Mr. Reyes to leave was likewise acceptable and humane under the circumstances. In this regard,
we cannot put our imprimatur on the appellate courts declaration that Ms. Lims act of personally approaching Mr. Reyes (without first verifying from
Mrs. Filart if indeed she invited Mr. Reyes) gave rise to a cause of action "predicated upon mere rudeness or lack of consideration of one person, which
calls not only protection of human dignity but respect of such dignity."
70
Without proof of any ill-motive on her part, Ms. Lims act of by-passing Mrs. Filart
cannot amount to abusive conduct especially because she did inquire from Mrs. Filarts companion who told her that Mrs. Filart did not invite Mr.
Reyes.
71
If at all, Ms. Lim is guilty only of bad judgment which, if done with good intentions, cannot amount to bad faith.
Not being liable for both actual and moral damages, neither can petitioners Lim and Hotel Nikko be made answerable for exemplary
damages
72
especially for the reason stated by the Court of Appeals. The Court of Appeals held
Not a few of the rich people treat the poor with contempt because of the latters lowly station in life.l^vvphi1.net This has to be limited somewhere. In a
democracy, such a limit must be established. Social equality is not sought by the legal provisions under consideration, but due regard for decency and
propriety (Code Commission, pp. 33-34). And by way of example or correction for public good and to avert further commission of such acts, exemplary
damages should be imposed upon appellees.
73

The fundamental fallacy in the above-quoted findings is that it runs counter with the very facts of the case and the evidence on hand.l^vvphi1.net It is not
disputed that at the time of the incident in question, Mr. Reyes was "an actor of long standing; a co-host of a radio program over DZRH; a Board Member
of the Music Singer Composer (MUSICO) chaired by popular singer Imelda Papin; a showbiz Coordinator of Citizen Crime Watch; and 1992 official
candidate of the KBL Party for Governor of Bohol; and an awardee of a number of humanitarian organizations of the Philippines."
74
During his direct
examination on rebuttal, Mr. Reyes stressed that he had income
75
and nowhere did he say otherwise. On the other hand, the records are bereft of any
information as to the social and economic standing of petitioner Ruby Lim. Consequently, the conclusion reached by the appellate court cannot
withstand scrutiny as it is without basis.
All told, and as far as Ms. Lim and Hotel Nikko are concerned, any damage which Mr. Reyes might have suffered through Ms. Lims exercise of a
legitimate right done within the bounds of propriety and good faith, must be his to bear alone.

25
WHEREFORE, premises considered, the petition filed by Ruby Lim and Nikko Hotel Manila Garden is GRANTED. The Decision of the Court of Appeals
dated 26 November 2001 and its Resolution dated 09 July 2002 are hereby REVERSED and SET ASIDE. The Decision of the Regional Trial Court of
Quezon City, Branch 104, dated 26 April 1999 is hereby AFFIRMED. No costs.
SO ORDERED.

G.R. No. L-17396 May 30, 1962
CECILIO PE, ET AL., plaintiffs-appellants,
vs.
ALFONSO PE, defendant-appellee.
Cecilio L. Pe for and in his own behalf as plaintiff-appellant.
Leodegario L. Mogol for defendant-appellee.
BAUTISTA ANGELO, J .:
Plaintiffs brought this action before the Court of First Instance of Manila to recover moral, compensatory, exemplary and corrective damages in the
amount of P94,000.00 exclusive of attorney's fees and expenses of litigation.
Defendant, after denying some allegations contained in the complaint, set up as a defense that the facts alleged therein, even if true, do not constitute a
valid cause of action.
After trial, the lower court, after finding that defendant had carried on a love affair with one Lolita Pe, an unmarried woman, being a married man himself,
declared that defendant cannot be held liable for moral damages it appearing that plaintiffs failed to prove that defendant, being aware of his marital
status, deliberately and in bad faith tried to win Lolita's affection. So it rendered decision dismissing the complaint.1wph1.t
Plaintiffs brought this case on appeal before this Court on the ground that the issues involved are purely of law.
The facts as found by the trial court are: Plaintiffs are the parents, brothers and sisters of one Lolita Pe. At the time of her disappearance on April 14,
1957, Lolita was 24 years old and unmarried. Defendant is a married man and works as agent of the La Perla Cigar and Cigarette Factory. He used to
stay in the town of Gasan, Marinduque, in connection with his aforesaid occupation. Lolita was staying with her parents in the same town. Defendant
was an adopted son of a Chinaman named Pe Beco, a collateral relative of Lolita's father. Because of such fact and the similarity in their family name,
defendant became close to the plaintiffs who regarded him as a member of their family. Sometime in 1952, defendant frequented the house of Lolita on
the pretext that he wanted her to teach him how to pray the rosary. The two eventually fell in love with each other and conducted clandestine trysts not
only in the town of Gasan but also in Boac where Lolita used to teach in a barrio school. They exchanged love notes with each other the contents of
which reveal not only their infatuation for each other but also the extent to which they had carried their relationship. The rumors about their love affairs
reached the ears of Lolita's parents sometime, in 1955, and since then defendant was forbidden from going to their house and from further seeing Lolita.
The plaintiffs even filed deportation proceedings against defendant who is a Chinese national. The affair between defendant and Lolita continued
nonetheless.
Sometime in April, 1957, Lolita was staying with her brothers and sisters at their residence at 54-B Espaa Extension, Quezon City. On April 14, 1957,
Lolita disappeared from said house. After she left, her brothers and sisters checked up her thing and found that Lolita's clothes were gone. However,
plaintiffs found a note on a crumpled piece of paper inside Lolita's aparador. Said note, written on a small slip of paper approximately 4" by 3" in size,
was in a handwriting recognized to be that of defendant's. In English it reads:
Honey, suppose I leave here on Sunday night, and that's 13th of this month and we will have a date on the 14th, that's Monday
morning at 10 a.m.
Reply
Love
The disappearance of Lolita was reported to the police authorities and the NBI but up to the present there is no news or trace of her whereabouts.
The present action is based on Article 21 of the New Civil Code which provides:
Any person who wilfully causes loss or injury to another in a manner which is contrary to morals, good customs or public policy shall
compensate the latter for the damage.
There is no doubt that the claim of plaintiffs for damages is based on the fact that defendant, being a married man, carried on a love affair with Lolita Pe
thereby causing plaintiffs injury in a manner contrary to morals, good customs and public policy. But in spite of the fact that plaintiffs have clearly
established that in illicit affair was carried on between defendant and Lolita which caused great damage to the name and reputation of plaintiffs who are
her parents, brothers and sisters, the trial court considered their complaint not actionable for the reason that they failed to prove that defendant
deliberately and in bad faith tried to win Lolita's affection Thus, the trial court said: "In the absence of proof on this point, the court may not presume that
it was the defendant who deliberately induced such relationship. We cannot be unmindful of the uncertainties and sometimes inexplicable mysteries of

26
the human emotions. It is a possibility that the defendant and Lolita simply fell in love with each other, not only without any desire on their part, but also
against their better judgment and in full consciousness of what it will bring to both of them. This is specially so with respect to Lolita, being an unmarried
woman, falling in love with defendant who is a married man."
We disagree with this view. The circumstances under which defendant tried to win Lolita's affection cannot lead, to any other conclusion than that it was
he who, thru an ingenious scheme or trickery, seduced the latter to the extent of making her fall in love with him. This is shown by the fact that defendant
frequented the house of Lolita on the pretext that he wanted her to teach him how to pray the rosary. Because of the frequency of his visits to the latter's
family who was allowed free access because he was a collateral relative and was considered as a member of her family, the two eventually fell in love
with each other and conducted clandestine love affairs not only in Gasan but also in Boac where Lolita used to teach in a barrio school. When the
rumors about their illicit affairs reached the knowledge of her parents, defendant was forbidden from going to their house and even from seeing Lolita.
Plaintiffs even filed deportation proceedings against defendant who is a Chinese national. Nevertheless, defendant continued his love affairs with Lolita
until she disappeared from the parental home. Indeed, no other conclusion can be drawn from this chain of events than that defendant not only
deliberately, but through a clever strategy, succeeded in winning the affection and love of Lolita to the extent of having illicit relations with her. The wrong
he has caused her and her family is indeed immeasurable considering the fact that he is a married man. Verily, he has committ ed an injury to Lolita's
family in a manner contrary to morals, good customs and public policy as contemplated in Article 21 of the new Civil Code.
WHEREFORE, the decision appealed from is reversed. Defendant is hereby sentenced to pay the plaintiffs the sum of P5,000.00 as damages and
P2,000.00 as attorney's fees and expenses of litigations. Costs against appellee.
Padilla, Labrador, Concepcion, Reyes, J.B.L., Barrera, Paredes and Dizon, JJ., concur.

The Lawphil Project - Arellano Law Foundation

PHILIPPINE JURISPRUDENCE - FULL TEXT
The Lawphil Project - Arellano Law Foundation
G.R. No. 101749 July 10, 1992
CONRADO BUNAG, JR. vs. HON. COURT OF APPEALS, ET AL.


Republic of the Philippines
SUPREME COURT
Manila
SECOND DIVISION

G.R. No. 101749 July 10, 1992
CONRADO BUNAG, JR., petitioner,
vs.
HON. COURT OF APPEALS, First Division, and ZENAIDA B. CIRILO, respondents.

REGALADO, J .:
Petitioner appeals for the reversal of the decision
1
of respondent Court of Appeals promulgated on May 17, 1991 in CA-G.R. CV No. 07054, entitled
"Zenaida B. Cirilo vs. Conrado Bunag, Sr. and Conrado Bunag, Jr.," which affirmed in toto the decision of the Regional Trial Court, Branch XI at Bacoor,
Cavite, and, implicitly, respondent court's resolution of September 3, 1991
2
denying petitioner's motion for reconsideration.
Respondent court having assiduously discussed the salient antecedents of this case, vis-a-vis the factual findings of the court below, the evidence of
record and the contentions of the parties, it is appropriate that its findings, which we approve and adopt, be extensively reproduced hereunder:
Based on the evidence on record, the following facts are considered indisputable: On the afternoon of September 8, 1973,
defendant-appellant Bunag, Jr. brought plaintiff-appellant to a motel or hotel where they had sexual intercourse. Later that evening,
said defendant-appellant brought plaintiff-appellant to the house of his grandmother Juana de Leon in Pamplona, Las Pias, Metro
Manila, where they lived together as husband and wife for 21 days, or until September 29, 1973. On September 10, 1973,
defendant-appellant Bunag, Jr. and plaintiff-appellant filed their respective applications for a marriage license with the Office of the
Local Civil Registrar of Bacoor, Cavite. On October 1, 1973, after leaving plaintiff-appellant, defendant-appellant Bunag, Jr. filed an
affidavit withdrawing his application for a marriage license.
Plaintiff-appellant contends that on the afternoon of September 8, 1973, defendant-appellant Bunag, Jr., together with an
unidentified male companion, abducted her in the vicinity of the San Juan de Dios Hospital in Pasay City and brought her to a motel
where she was raped. The court a quo, which adopted her evidence, summarized the same which we paraphrased as follows:
Plaintiff was 26 years old on November 5, 1974 when she testified, single and had finished a college course in
Commerce (t.s.n., p. 4, Nov. 5, 1974). It appears that on September 8, 1973, at about 4:00 o'clock in the

27
afternoon, while she was walking along Figueras Street, Pasay City on her way to the San Juan de Dios
Canteen to take her snack, defendant, Conrado Bunag, Jr., came riding in a car driven by a male companion.
Plaintiff and defendant Bunag, Jr. were sweethearts, but two weeks before September 8, 1973, they had a
quarrel, and Bunag, Jr. wanted to talk matters over with plaintiff, so that he invited her to take their merienda at
the Aristocrat Restaurant in Manila instead of at the San Juan de Dios Canteen, to which plaintiff obliged, as
she believed in his sincerity (t.s.n., pp. 8-10, Nov. 5, 1974).
Plaintiff rode in the car and took the front seat beside the driver while Bunag, Jr. seated himself by her right
side. The car travelled north on its way to the Aristocrat Restaurant but upon reaching San Juan Street in Pasay
City, it turned abruptly to the right, to which plaintiff protested, but which the duo ignored and instead threatened
her not to make any noise as they were ready to die and would bump the car against the post if she persisted.
Frightened and silenced, the car travelled its course thru F.B. Harrison Boulevard until they reached a motel.
Plaintiff was then pulled and dragged from the car against her will, and amidst her cries and pleas. In spite of
her struggle she was no match to the joint strength of the two male combatants because of her natural
weakness being a woman and her small stature. Eventually, she was brought inside the hotel where the
defendant Bunag, Jr. deflowered her against her will and consent. She could not fight back and repel the attack
because after Bunag, Jr. had forced her to lie down and embraced her, his companion held her two feet,
removed her panty, after which he left. Bunag, Jr. threatened her that he would ask his companion to come
back and hold her feet if she did not surrender her womanhood to him, thus he succeeded in feasting on her
virginity. Plaintiff described the pains she felt and how blood came out of her private parts after her vagina was
penetrated by the penis of the defendant Bunag, Jr. (t.s.n. pp. 17-24, Nov. 5, 1974).
After that outrage on her virginity, plaintiff asked Bunag, Jr. once more to allow her to go home but the latter
would not consent and stated that he would only let her go after they were married as he intended to marry her,
so much so that she promised not to make any scandal and to marry him. Thereafter, they took a taxi together
after the car that they used had already gone, and proceeded to the house of Juana de Leon, Bunag, Jr.'s
grandmother in Pamplona, Las Pias, Metro Manila where they arrived at 9:30 o'clock in the evening (t.s.n., p.
26, Nov. 5, 1974). At about ten (10) o'clock that same evening, defendant Conrado Bunag, Sr., father of Bunag,
Jr. arrived and assured plaintiff that the following day which was a Monday, she and Bunag, Jr. would go to
Bacoor, to apply for a marriage license, which they did. They filed their applications for marriage license
(Exhibits "A" and "C") and after that plaintiff and defendant Bunag, Jr. returned to the house of Juana de Leon
and lived there as husband and wife from September 8, 1973 to September 29, 1973.
On September 29, 1973 defendant Bunag, Jr. left and never returned, humiliating plaintiff and compelled her to
go back to her parents on October 3, 1973. Plaintiff was ashamed when she went home and could not sleep
and eat because of the deception done against her by defendants-appellants (t.s.n., p. 35, Nov. 5, 1974).
The testimony of plaintiff was corroborated in toto by her uncle, Vivencio Bansagan who declared that on
September 8, 1973 when plaintiff failed to arrive home at 9:00 o'clock in the evening, his sister who is the
mother of plaintiff asked him to look for her but his efforts proved futile, and he told his sister that plaintiff might
have married (baka nag-asawa, t.s.n., pp. 5-6, March 18, 1976). However, in the afternoon of the next day
(Sunday), his sister told him that Francisco Cabrera, accompanied by barrio captain Jacinto Manalili of Ligas,
Bacoor, Cavite, informed her that plaintiff and Bunag, Jr. were in Cabrera's house, so that her sister requested
him to go and see the plaintiff, which he did, and at the house of Mrs. Juana de Leon in Pamplona, Las Pias,
Metro Manila he met defendant Conrado Bunag, Sr., who told him, "Pare, the children are here already. Let us
settle the matter and have them married."
He conferred with plaintiff who told him that as she had already lost her honor, she would bear her sufferings as Boy Bunag, Jr. and
his father promised they would be married.
Defendants-appellants, on the other hand, deny that defendant-appellant Conrado Bunag, Jr. abducted and raped plaintiff-appellant
on September 8, 1973. On the contrary, plaintiff-appellant and defendant-appellant Bunag, Jr. eloped on that date because of the
opposition of the latter's father to their relationship.
Defendant-appellants claim that defendant-appellant Bunag, Jr. and plaintiff-appellant had earlier made plans to elope and get
married, and this fact was known to their friends, among them, Architect Chito Rodriguez. The couple made good their plans to
elope on the afternoon of September 8, 1973, when defendant-appellant Bunag, Jr., accompanied by his friend Guillermo Ramos,
Jr., met plaintiff-appellant and her officemate named Lydia in the vicinity of the San Juan de Dios Hospital. The foursome then
proceeded to (the) aforesaid hospital's canteen where they had some snacks. Later, Guillermo Ramos, Jr. took Lydia to Quirino
Avenue where she could get a ride home, thereby leaving the defendant-appellant Bunag, Jr. and plaintiff-appellant alone.
According to defendant-appellant Bunag, Jr., after Guillermo Ramos, Jr. and Lydia left, he and plaintiff-appellant took a taxi to the
Golden Gate and Flamingo Hotels where they tried to get a room, but these were full. They finally got a room at the Holiday Hotel,
where defendant-appellant registered using his real name and residence certificate number. Three hours later, the couple check out
of the hotel and proceeded to the house of Juana de Leon at Pamplona, Las Pias, where they stayed until September 19, 1873.
Defendant-appellant claims that bitter disagreements with the plaintiff-appellant over money and the threats made to his life
prompted him to break off their plan to get married.
During this period, defendant-appellant Bunag, Sr. denied having gone to the house of Juan de Leon and telling plaintiff-appellant
that she would be wed to defendant-appellant Bunag, Jr. In fact, he phoned Atty. Conrado Adreneda, member of the board of
directors of Mandala Corporation, defendant-appellant Bunag, Jr.'s employer, three times between the evening of September 8,
1973 and September 9, 1973 inquiring as to the whereabouts of his son. He came to know about his son's whereabouts when he
was told of the couple's elopement late in the afternoon of September 9, 1973 by his mother Candida Gawaran. He likewise denied
having met relatives and emissaries of plaintiff-appellant and agreeing to her marriage to his son.
3


28
A complaint for damages for alleged breach of promise to marry was filed by herein private respondent Zenaida B. Cirilo against petitioner Conrado
Bunag, Jr. and his father, Conrado Bunag, Sr., as Civil Case No. N-2028 of the Regional Trial Court, Branch XIX at Bacoor, Cavite. On August 20, 1983,
on a finding, inter alia, that petitioner had forcibly abducted and raped private respondent, the trial court rendered a decision
4
ordering petitioner Bunag,
Jr. to pay private respondent P80,000.00 as moral damages, P20,000.00 as exemplary damages, P20,000.00 by way of temperate damages, and
P10,000.00 for and as attorney's fees, as well as the costs of suit. Defendant Conrado Bunag, Sr. was absolved from any and all liability.
Private respondent appealed that portion of the lower court's decision disculpating Conrado Bunag, Sr. from civil liability in this case. On the other hand,
the Bunags, as defendants-appellants, assigned in their appeal several errors allegedly committed by trial court, which were summarized by respondent
court as follows: (1) in finding that defendant-appellant Conrado Bunag, Jr. forcibly abducted and raped plaintiff-appellant; (2) in finding that defendants-
appellants promised plaintiff-appellant that she would be wed to defendant-appellant Conrado Bunag, Jr.; and (3) in awarding plaintiff-appellant
damages for the breach of defendants-appellants' promise of marriage.
5

As stated at the outset, on May 17, 1991 respondent Court of Appeals rendered judgment dismissing both appeals and affirming in toto the decision of
the trial court. His motion for reconsideration having been denied, petitioner Bunag, Jr. is before us on a petition for review, contending that (1)
respondent court failed to consider vital exhibits, testimonies and incidents for petitioner's defense, resulting in the misapprehensions of facts and
violative of the law on preparation of judgment; and (2) it erred in the application of the proper law and jurisprudence by holding that there was forcible
abduction with rape, not just a simple elopement and an agreement to marry, and in the award of excessive damages.
6

Petitioner Bunag, Jr. first contends that both the trial and appellate courts failed to take into consideration the alleged fact that he and private respondent
had agreed to marry, and that there was no case of forcible abduction with rape, but one of simple elopement and agreement to marry. It is averred that
the agreement to marry has been sufficiently proven by the testimonies of the witnesses for both parties and the exhibits presented in court.
This submission, therefore, clearly hinges on the credibility of the witnesses and evidence presented by the parties and the weight accorded thereto in
the factual findings of the trial court and the Court of Appeals. In effect, what petitioner would want this Court to do is to evaluate and analyze anew the
evidence, both testimonial and documentary, presented before and calibrated by the trial court, and as further meticulously reviewed and discussed by
respondent court.
The issue raised primarily and ineluctably involves questions of fact. We are, therefore, once again constrained to stress the well-entrenched statutory
and jurisprudential mandate that findings of fact of the Court of Appeals are, as a rule, conclusive upon this Court. Only questions of law, distinctly set
forth, may be raised in a petition for review on certiorari under Rule 45 of the Rules of Court, subject to clearly settled exceptions in case law.
Our jurisdiction in cases brought to us from the Court of Appeals is limited to reviewing and revising the errors of law imputed to the latter, its findings of
fact being conclusive. This Court has emphatically declared that it is not its function to analyze or weigh such evidence all over again, its jurisdiction
being limited to reviewing errors of law that might have been committed by the lower court. Barring, therefore, a showing that the findings complained of
are totally devoid of support in the record, or that they are so glaringly erroneous as to constitute serious abuse of discretion, such findings must stand,
for this Court is not expected or required to examine or contrast the oral and documentary evidence submitted by the parties.
7
Neither does the instant
case reveal any feature falling within, any of the exceptions which under our decisional rules may warrant a review of the factual findings of the Court of
Appeals. On the foregoing considerations and our review of the records, we sustain the holding of respondent court in favor of private respondent.
Petitioner likewise asserts that since action involves a breach of promise to marry, the trial court erred in awarding damages.
It is true that in this jurisdiction, we adhere to the time-honored rule that an action for breach of promise to marry has no standing in the civil law, apart
from the right to recover money or property advanced by the plaintiff upon the faith of such promise. 8 Generally, therefore, a breach of promise to
marry per se is not actionable, except where the plaintiff has actually incurred expenses for the wedding and the necessary incidents thereof.
However, the award of moral damages is allowed in cases specified in or analogous to those provided in Article 2219 of the Civil Code. Correlatively,
under Article 21 of said Code, in relation to paragraph 10 of said Article 2219, any person who wilfully causes loss or injury to another in a manner that is
contrary to morals, good customs or public policy shall compensate the latter for moral damages.
9
Article 21 was adopted to remedy the countless gaps
in the statutes which leave so many victims of moral wrongs helpless even though they have actually suffered material and moral injury, and is intended
to vouchsafe adequate legal remedy for that untold number of moral wrongs which is impossible for human foresight to specifically provide for in the
statutes.
10

Under the circumstances obtaining in the case at bar, the acts of petitioner in forcibly abducting private respondent and having carnal knowledge with
her against her will, and thereafter promising to marry her in order to escape criminal liability, only to thereafter renege on such promise after cohabiting
with her for twenty-one days, irremissibly constitute acts contrary to morals and good customs. These are grossly insensate and reprehensible
transgressions which indisputably warrant and abundantly justify the award of moral and exemplary damages, pursuant to Article 21 in relation to
paragraphs 3 and 10, Article 2219, and Article 2229 and 2234 of Civil Code.
Petitioner would, however, belabor the fact that said damages were awarded by the trial court on the basis of a finding that he is guilty of forcible
abduction with rape, despite the prior dismissal of the complaint therefor filed by private respondent with the Pasay City Fiscal's Office.
Generally, the basis of civil liability from crime is the fundamental postulate of our law that every person criminally liable for a felony is also civilly liable.
In other words, criminal liability will give rise to civil liability ex delicto only if the same felonious act or omission results in damage or injury to another and
is the direct and proximate cause thereof.
11
Hence, extinction of the penal action does not carry with it the extinction of civil liability unless the extinction
proceeds from a declaration in a final judgment that the fact from which the civil might arise did not exist.
12

In the instant case, the dismissal of the complaint for forcible abduction with rape was by mere resolution of the fiscal at the preliminary investigation
stage. There is no declaration in a final judgment that the fact from which the civil case might arise did not exist. Consequently, the dismissal did not in
any way affect the right of herein private respondent to institute a civil action arising from the offense because such preliminary dismissal of the penal
action did not carry with it the extinction of the civil action.

29
The reason most often given for this holding is that the two proceedings involved are not between the same parties. Furthermore, it has long been
emphasized, with continuing validity up to now, that there are different rules as to the competency of witnesses and the quantum of evidence in criminal
and civil proceedings. In a criminal action, the State must prove its case by evidence which shows the guilt of the accused beyond reasonable doubt,
while in a civil action it is sufficient for the plaintiff to sustain his cause by preponderance of evidence only.
13
Thus, in Rillon, et al. vs. Rillon,
14
we
stressed that it is not now necessary that a criminal prosecution for rape be first instituted and prosecuted to final judgment before a civil action based on
said offense in favor of the offended woman can likewise be instituted and prosecuted to final judgment.
WHEREFORE, the petition is hereby DENIED for lack of merit, and the assailed judgment and resolution are hereby AFFIRMED.
SO ORDERED.
Narvasa, C.J. and Padilla, J., concur.
Nocon, J., took no part.

G.R. No. 119976 September 18, 1995
IMELDA ROMUALDEZ-MARCOS, petitioner,
vs.
COMMISSION ON ELECTIONS and CIRILO ROY MONTEJO, respondents.

KAPUNAN, J .:
A constitutional provision should be construed as to give it effective operation and suppress the mischief at which it is aimed.
1
The 1987 Constitution
mandates that an aspirant for election to the House of Representatives be "a registered voter in the district in which he shall be elected, and a resident
thereof for a period of not less than one year immediately preceding the election."
2
The mischief which this provision reproduced verbatim from the
1973 Constitution seeks to prevent is the possibility of a "stranger or newcomer unacquainted with the conditions and needs of a community and not
identified with the latter, from an elective office to serve that community."
3

Petitioner Imelda Romualdez-Marcos filed her Certificate of Candidacy for the position of Representative of the First District of Leyte with the Provincial
Election Supervisor on March 8, 1995, providing the following information in item no. 8:
4

RESIDENCE IN THE CONSTITUENCY WHERE I SEEK TO BE ELECTED IMMEDIATELY PRECEDING THE
ELECTION: __________ Years and seven Months.
On March 23, 1995, private respondent Cirilo Roy Montejo, the incumbent Representative of the First District of Leyte and a candidate for the same
position, filed a "Petition for Cancellation and Disqualification"
5
with the Commission on Elections alleging that petitioner did not meet the constitutional
requirement for residency. In his petition, private respondent contended that Mrs. Marcos lacked the Constitution's one year residency requirement for
candidates for the House of Representatives on the evidence of declarations made by her in Voter Registration Record 94-No. 3349772
6
and in her
Certificate of Candidacy. He prayed that "an order be issued declaring (petitioner) disqualified and canceling the certificate of candidacy."
7

On March 29, 1995, petitioner filed an Amended/Corrected Certificate of Candidacy, changing the entry "seven" months to "since childhood" in item no.
8 of the amended certificate.
8
On the same day, the Provincial Election Supervisor of Leyte informed petitioner that:
[T]his office cannot receive or accept the aforementioned Certificate of Candidacy on the ground that it is filed out of time,
the deadline for the filing of the same having already lapsed on March 20, 1995. The Corrected/Amended Certificate of
Candidacy should have been filed on or before the March 20, 1995 deadline.
9

Consequently, petitioner filed the Amended/Corrected Certificate of Candidacy with the COMELEC's Head Office in Intramuros, Manila on
March 31, 1995. Her Answer to private respondent's petition in SPA No. 95-009 was likewise filed with the head office on the same day. In said Answer,
petitioner averred that the entry of the word "seven" in her original Certificate of Candidacy was the result of an "honest misinterpretation"
10
which she
sought to rectify by adding the words "since childhood" in her Amended/Corrected Certificate of Candidacy and that "she has always maintained
Tacloban City as her domicile or residence.
11
Impugning respondent's motive in filing the petition seeking her disqualification, she noted that:
When respondent (petitioner herein) announced that she was intending to register as a voter in Tacloban City and run for
Congress in the First District of Leyte, petitioner immediately opposed her intended registration by writing a letter stating
that "she is not a resident of said city but of Barangay Olot, Tolosa, Leyte. After respondent had registered as a voter in
Tolosa following completion of her six month actual residence therein, petitioner filed a petition with the COMELEC to
transfer the town of Tolosa from the First District to the Second District and pursued such a move up to the Supreme
Court, his purpose being to remove respondent as petitioner's opponent in the congressional election in the First District.
He also filed a bill, along with other Leyte Congressmen, seeking the creation of another legislative district to remove the
town of Tolosa out of the First District, to achieve his purpose. However, such bill did not pass the Senate. Having failed
on such moves, petitioner now filed the instant petition for the same objective, as it is obvious that he is afraid to submit
along with respondent for the judgment and verdict of the electorate of the First District of Leyte in an honest, orderly,
peaceful, free and clean elections on May 8, 1995.
12


30
On April 24, 1995, the Second Division of the Commission on Elections (COMELEC), by a vote of 2 to 1,
13
came up with a Resolution 1) finding private
respondent's Petition for Disqualification in SPA 95-009 meritorious; 2) striking off petitioner's Corrected/Amended Certificate of Candidacy of March 31,
1995; and 3) canceling her original Certificate of Candidacy.
14
Dealing with two primary issues, namely, the validity of amending the original Certificate
of Candidacy after the lapse of the deadline for filing certificates of candidacy, and petitioner's compliance with the one year residency requirement, the
Second Division held:
Respondent raised the affirmative defense in her Answer that the printed word "Seven" (months) was a result of an
"honest misinterpretation or honest mistake" on her part and, therefore, an amendment should subsequently be allowed.
She averred that she thought that what was asked was her "actual and physical" presence in Tolosa and not residence of
origin or domicile in the First Legislative District, to which she could have responded "since childhood." In an
accompanying affidavit, she stated that her domicile is Tacloban City, a component of the First District, to which she
always intended to return whenever absent and which she has never abandoned. Furthermore, in her memorandum, she
tried to discredit petitioner's theory of disqualification by alleging that she has been a resident of the First Legislative
District of Leyte since childhood, although she only became a resident of the Municipality of Tolosa for seven months. She
asserts that she has always been a resident of Tacloban City, a component of the First District, before coming to the
Municipality of Tolosa.
Along this point, it is interesting to note that prior to her registration in Tolosa, respondent announced that she would be
registering in Tacloban City so that she can be a candidate for the District. However, this intention was rebuffed when
petitioner wrote the Election Officer of Tacloban not to allow respondent since she is a resident of Tolosa and not
Tacloban. She never disputed this claim and instead implicitly acceded to it by registering in Tolosa.
This incident belies respondent's claim of "honest misinterpretation or honest mistake." Besides, the Certificate of
Candidacy only asks for RESIDENCE. Since on the basis of her Answer, she was quite aware of "residence of origin"
which she interprets to be Tacloban City, it is curious why she did not cite Tacloban City in her Certificate of Candidacy.
Her explanation that she thought what was asked was her actual and physical presence in Tolosa is not easy to believe
because there is none in the question that insinuates about Tolosa. In fact, item no. 8 in the Certificate of Candidacy
speaks clearly of "Residency in the CONSTITUENCY where I seek to be elected immediately preceding the election."
Thus, the explanation of respondent fails to be persuasive.
From the foregoing, respondent's defense of an honest mistake or misinterpretation, therefore, is devoid of merit.
To further buttress respondent's contention that an amendment may be made, she cited the case ofAlialy v. COMELEC (2
SCRA 957). The reliance of respondent on the case of Alialy is misplaced. The case only applies to the "inconsequential
deviations which cannot affect the result of the election, or deviations from provisions intended primarily to secure timely
and orderly conduct of elections." The Supreme Court in that case considered the amendment only as a matter of form.
But in the instant case, the amendment cannot be considered as a matter of form or an inconsequential deviation. The
change in the number of years of residence in the place where respondent seeks to be elected is a substantial matter
which determines her qualification as a candidacy, specially those intended to suppress, accurate material representation
in the original certificate which adversely affects the filer. To admit the amended certificate is to condone the evils brought
by the shifting minds of manipulating candidate, of the detriment of the integrity of the election.
Moreover, to allow respondent to change the seven (7) month period of her residency in order to prolong it by claiming it
was "since childhood" is to allow an untruthfulness to be committed before this Commission. The arithmetical accuracy of
the 7 months residency the respondent indicated in her certificate of candidacy can be gleaned from her entry in her
Voter's Registration Record accomplished on January 28, 1995 which reflects that she is a resident of Brgy. Olot, Tolosa,
Leyte for 6 months at the time of the said registration (Annex A, Petition). Said accuracy is further buttressed by her letter
to the election officer of San Juan, Metro Manila, dated August 24, 1994, requesting for the cancellation of her registration
in the Permanent List of Voters thereat so that she can be re-registered or transferred to Brgy. Olot, Tolosa, Leyte. The
dates of these three (3) different documents show the respondent's consistent conviction that she has transferred her
residence to Olot, Tolosa, Leyte from Metro Manila only for such limited period of time, starting in the last week of August
1994 which on March 8, 1995 will only sum up to 7 months. The Commission, therefore, cannot be persuaded to believe
in the respondent's contention that it was an error.
xxx xxx xxx
Based on these reasons the Amended/Corrected Certificate of Candidacy cannot be admitted by this Commission.
xxx xxx xxx
Anent the second issue, and based on the foregoing discussion, it is clear that respondent has not complied with the one
year residency requirement of the Constitution.
In election cases, the term "residence" has always been considered as synonymous with "domicile" which imports not
only the intention to reside in a fixed place but also personal presence in-that place, coupled with conduct indicative of
such intention. Domicile denotes a fixed permanent residence to which when absent for business or pleasure, or for like
reasons, one intends to return. (Perfecto Faypon vs. Eliseo Quirino, 96 Phil 294; Romualdez vs. RTC-Tacloban, 226
SCRA 408). In respondent's case, when she returned to the Philippines in 1991, the residence she chose was not
Tacloban but San Juan, Metro Manila. Thus, her animus revertendi is pointed to Metro Manila and not Tacloban.
This Division is aware that her claim that she has been a resident of the First District since childhood is nothing more than
to give her a color of qualification where she is otherwise constitutionally disqualified. It cannot hold ground in the face of

31
the facts admitted by the respondent in her affidavit. Except for the time that she studied and worked for some years after
graduation in Tacloban City, she continuously lived in Manila. In 1959, after her husband was elected Senator, she lived
and resided in San Juan, Metro Manila where she was a registered voter. In 1965, she lived in San Miguel, Manila where
she was again a registered voter. In 1978, she served as member of the Batasang Pambansa as the representative of the
City of Manila and later on served as the Governor of Metro Manila. She could not have served these positions if she had
not been a resident of the City of Manila. Furthermore, when she filed her certificate of candidacy for the office of the
President in 1992, she claimed to be a resident of San Juan, Metro Manila. As a matter of fact on August 24, 1994,
respondent wrote a letter with the election officer of San Juan, Metro Manila requesting for the cancellation of her
registration in the permanent list of voters that she may be re-registered or transferred to Barangay Olot, Tolosa, Leyte.
These facts manifest that she could not have been a resident of Tacloban City since childhood up to the time she filed her
certificate of candidacy because she became a resident of many places, including Metro Manila. This debunks her claim
that prior to her residence in Tolosa, Leyte, she was a resident of the First Legislative District of Leyte since childhood.
In this case, respondent's conduct reveals her lack of intention to make Tacloban her domicile. She registered as a voter
in different places and on several occasions declared that she was a resident of Manila. Although she spent her school
days in Tacloban, she is considered to have abandoned such place when she chose to stay and reside in other different
places. In the case of Romualdez vs. RTC(226 SCRA 408) the Court explained how one acquires a new domicile by
choice. There must concur: (1) residence or bodily presence in the new locality; (2) intention to remain there; and (3)
intention to abandon the old domicile. In other words there must basically be animus manendi with animus non revertendi.
When respondent chose to stay in Ilocos and later on in Manila, coupled with her intention to stay there by registering as a
voter there and expressly declaring that she is a resident of that place, she is deemed to have abandoned Tacloban City,
where she spent her childhood and school days, as her place of domicile.
Pure intention to reside in that place is not sufficient, there must likewise be conduct indicative of such intention.
Respondent's statements to the effect that she has always intended to return to Tacloban, without the accompanying
conduct to prove that intention, is not conclusive of her choice of residence. Respondent has not presented any evidence
to show that her conduct, one year prior the election, showed intention to reside in Tacloban. Worse, what was evident
was that prior to her residence in Tolosa, she had been a resident of Manila.
It is evident from these circumstances that she was not a resident of the First District of Leyte "since childhood."
To further support the assertion that she could have not been a resident of the First District of Leyte for more than one
year, petitioner correctly pointed out that on January 28, 1995 respondent registered as a voter at precinct No. 18-A of
Olot, Tolosa, Leyte. In doing so, she placed in her Voter Registration Record that she resided in the municipality of Tolosa
for a period of six months. This may be inconsequential as argued by the respondent since it refers only to her residence
in Tolosa, Leyte. But her failure to prove that she was a resident of the First District of Leyte prior to her residence in
Tolosa leaves nothing but a convincing proof that she had been a resident of the district for six months only.
15

In a Resolution promulgated a day before the May 8, 1995 elections, the COMELEC en banc denied petitioner's Motion for Reconsideration
16
of the
April 24, 1995 Resolution declaring her not qualified to run for the position of Member of the House of Representatives for the First Legislative District of
Leyte.
17
The Resolution tersely stated:
After deliberating on the Motion for Reconsideration, the Commission RESOLVED to DENY it, no new substantial matters
having been raised therein to warrant re-examination of the resolution granting the petition for disqualification.
18

On May 11, 1995, the COMELEC issued a Resolution allowing petitioner's proclamation should the results of the canvass show that she obtained the
highest number of votes in the congressional elections in the First District of Leyte. On the same day, however, the COMELEC reversed itself and issued
a second Resolution directing that the proclamation of petitioner be suspended in the event that she obtains the highest number of votes.
19

In a Supplemental Petition dated 25 May 1995, petitioner averred that she was the overwhelming winner of the elections for the congressional seat in
the First District of Leyte held May 8, 1995 based on the canvass completed by the Provincial Board of Canvassers on May 14, 1995. Petitioner alleged
that the canvass showed that she obtained a total of 70,471 votes compared to the 36,833 votes received by Respondent Montejo. A copy of said
Certificate of Canvass was annexed to the Supplemental Petition.
On account of the Resolutions disqualifying petitioner from running for the congressional seat of the First District of Leyte and the public respondent's
Resolution suspending her proclamation, petitioner comes to this court for relief.
Petitioner raises several issues in her Original and Supplemental Petitions. The principal issues may be classified into two general areas:
I. The issue of Petitioner's qualifications
Whether or not petitioner was a resident, for election purposes, of the First District of Leyte for a period of one year at the
time of the May 9, 1995 elections.
II. The Jurisdictional Issue
a) Prior to the elections
Whether or not the COMELEC properly exercised its jurisdiction in disqualifying petitioner outside the period mandated by
the Omnibus Election Code for disqualification cases under Article 78 of the said Code.

32
b) After the Elections
Whether or not the House of Representatives Electoral Tribunal assumed exclusive jurisdiction over the question of
petitioner's qualifications after the May 8, 1995 elections.
I. Petitioner's qualification
A perusal of the Resolution of the COMELEC's Second Division reveals a startling confusion in the application of settled concepts of "Domicile" and
"Residence" in election law. While the COMELEC seems to be in agreement with the general proposition that for the purposes of election law, residence
is synonymous with domicile, the Resolution reveals a tendency to substitute or mistake the concept of domicile for actual residence, a conception not
intended for the purpose of determining a candidate's qualifications for election to the House of Representatives as required by the 1987 Constitution. As
it were, residence, for the purpose of meeting the qualification for an elective position, has a settled meaning in our jurisdiction.
Article 50 of the Civil Code decrees that "[f]or the exercise of civil rights and the fulfillment of civil obligations, the domicile of natural persons is their
place of habitual residence." In Ong vs. Republic
20
this court took the concept of domicile to mean an individual's "permanent home", "a place to which,
whenever absent for business or for pleasure, one intends to return, and depends on facts and circumstances in the sense that they disclose
intent."
21
Based on the foregoing, domicile includes the twin elements of "the fact of residing or physical presence in a fixed place" and animus manendi,
or the intention of returning there permanently.
Residence, in its ordinary conception, implies the factual relationship of an individual to a certain place. It is the physical presence of a person in a given
area, community or country. The essential distinction between residence and domicile in law is that residence involves the intent to leave when the
purpose for which the resident has taken up his abode ends. One may seek a place for purposes such as pleasure, business, or health. If a person's
intent be to remain, it becomes his domicile; if his intent is to leave as soon as his purpose is established it is residence.
22
It is thus, quite perfectly
normal for an individual to have different residences in various places. However, a person can only have a single domicile, unless, for various reasons,
he successfully abandons his domicile in favor of another domicile of choice. In Uytengsu vs. Republic,
23
we laid this distinction quite clearly:
There is a difference between domicile and residence. "Residence" is used to indicate a place of abode, whether
permanent or temporary; "domicile" denotes a fixed permanent residence to which, when absent, one has the intention of
returning. A man may have a residence in one place and a domicile in another. Residence is not domicile, but domicile is
residence coupled with the intention to remain for an unlimited time. A man can have but one domicile for the same
purpose at any time, but he may have numerous places of residence. His place of residence is generally his place of
domicile, but it is not by any means necessarily so since no length of residence without intention of remaining will
constitute domicile.
For political purposes the concepts of residence and domicile are dictated by the peculiar criteria of political laws. As these concepts have evolved in our
election law, what has clearly and unequivocally emerged is the fact that residence for election purposes is used synonymously with domicile.
In Nuval vs. Guray,
24
the Court held that "the term residence. . . is synonymous with domicile which imports not only intention to reside in a fixed place,
but also personal presence in that place, coupled with conduct indicative of such intention."
25
Larena vs. Teves
26
reiterated the same doctrine in a case
involving the qualifications of the respondent therein to the post of Municipal President of Dumaguete, Negros Oriental. Faypon vs. Quirino,
27
held that
the absence from residence to pursue studies or practice a profession or registration as a voter other than in the place where one is elected does not
constitute loss of residence.
28
So settled is the concept (of domicile) in our election law that in these and other election law cases, this Court has stated
that the mere absence of an individual from his permanent residence without the intention to abandon it does not result in a loss or change of domicile.
The deliberations of the 1987 Constitution on the residence qualification for certain elective positions have placed beyond doubt the principle that when
the Constitution speaks of "residence" in election law, it actually means only "domicile" to wit:
Mr. Nolledo: With respect to Section 5, I remember that in the 1971 Constitutional Convention, there was an attempt to
require residence in the place not less than one year immediately preceding the day of the elections. So my question is:
What is the Committee's concept of residence of a candidate for the legislature? Is it actual residence or is it the concept
of domicile or constructive residence?
Mr. Davide: Madame President, insofar as the regular members of the National Assembly are concerned, the proposed
section merely provides, among others, "and a resident thereof", that is, in the district for a period of not less than one
year preceding the day of the election. This was in effect lifted from the 1973 Constitution, the interpretation given to it
was domicile.
29

xxx xxx xxx
Mrs. Rosario Braid: The next question is on Section 7, page 2. I think Commissioner Nolledo has raised the same point
that "resident" has been interpreted at times as a matter of intention rather than actual residence.
Mr. De los Reyes: Domicile.
Ms. Rosario Braid: Yes, So, would the gentleman consider at the proper time to go back to actual residence rather than
mere intention to reside?

33
Mr. De los Reyes: But we might encounter some difficulty especially considering that a provision in the Constitution in the
Article on Suffrage says that Filipinos living abroad may vote as enacted by law. So, we have to stick to the original
concept that it should be by domicile and not physical residence.
30

In Co vs. Electoral Tribunal of the House of Representatives,
31
this Court concluded that the framers of the 1987 Constitution obviously adhered to the
definition given to the term residence in election law, regarding it as having the same meaning as domicile.
32

In the light of the principles just discussed, has petitioner Imelda Romualdez Marcos satisfied the residency requirement mandated by Article VI, Sec. 6
of the 1987 Constitution? Of what significance is the questioned entry in petitioner's Certificate of Candidacy stating her residence in the First Legislative
District of Leyte as seven (7) months?
It is the fact of residence, not a statement in a certificate of candidacy which ought to be decisive in determining whether or not and individual has
satisfied the constitution's residency qualification requirement. The said statement becomes material only when there is or appears to be a deliberate
attempt to mislead, misinform, or hide a fact which would otherwise render a candidate ineligible. It would be plainly ridiculous for a candidate to
deliberately and knowingly make a statement in a certificate of candidacy which would lead to his or her disqualification.
It stands to reason therefore, that petitioner merely committed an honest mistake in jotting the word "seven" in the space provided for the residency
qualification requirement. The circumstances leading to her filing the questioned entry obviously resulted in the subsequent confusion which prompted
petitioner to write down the period of her actual stay in Tolosa, Leyte instead of her period of residence in the First district, which was "since childhood"
in the space provided. These circumstances and events are amply detailed in the COMELEC's Second Division's questioned resolution, albeit with a
different interpretation. For instance, when herein petitioner announced that she would be registering in Tacloban City to make her eligible to run in the
First District, private respondent Montejo opposed the same, claiming that petitioner was a resident of Tolosa, not Tacloban City. Petitioner then
registered in her place of actual residence in the First District, which is Tolosa, Leyte, a fact which she subsequently noted down in her Certificate of
Candidacy. A close look at said certificate would reveal the possible source of the confusion: the entry for residence (Item No. 7) is followed immediately
by the entry for residence in the constituency where a candidate seeks election thus:
7. RESIDENCE (complete Address): Brgy. Olot, Tolosa, Leyte
POST OFFICE ADDRESS FOR ELECTION PURPOSES: Brgy. Olot, Tolosa, Leyte
8. RESIDENCE IN THE CONSTITUENCY WHERE I SEEK TO
BE ELECTED IMMEDIATELY PRECEDING THE ELECTION:_________ Years and Seven Months.
Having been forced by private respondent to register in her place of actual residence in Leyte instead of petitioner's claimed domicile, it appears that
petitioner had jotted down her period of stay in her legal residence or domicile. The juxtaposition of entries in Item 7 and Item 8 the first requiring
actual residence and the second requiring domicile coupled with the circumstances surrounding petitioner's registration as a voter in Tolosa obviously
led to her writing down an unintended entry for which she could be disqualified. This honest mistake should not, however, be allowed to negate the fact
of residence in the First District if such fact were established by means more convincing than a mere entry on a piece of paper.
We now proceed to the matter of petitioner's domicile.
In support of its asseveration that petitioner's domicile could not possibly be in the First District of Leyte, the Second Division of the COMELEC, in its
assailed Resolution of April 24,1995 maintains that "except for the time when (petitioner) studied and worked for some years after graduation in
Tacloban City, she continuously lived in Manila." The Resolution additionally cites certain facts as indicative of the fact that petitioner's domicile ought to
be any place where she lived in the last few decades except Tacloban, Leyte. First, according to the Resolution, petitioner, in 1959, resided in San Juan,
Metro Manila where she was also registered voter. Then, in 1965, following the election of her husband to the Philippine presidency, she lived in San
Miguel, Manila where she as a voter. In 1978 and thereafter, she served as a member of the Batasang Pambansa and Governor of Metro Manila. "She
could not, have served these positions if she had not been a resident of Metro Manila," the COMELEC stressed. Here is where the confusion lies.
We have stated, many times in the past, that an individual does not lose his domicile even if he has lived and maintained residences in different places.
Residence, it bears repeating, implies a factual relationship to a given place for various purposes. The absence from legal residence or domicile to
pursue a profession, to study or to do other things of a temporary or semi-permanent nature does not constitute loss of residence. Thus, the assertion by
the COMELEC that "she could not have been a resident of Tacloban City since childhood up to the time she filed her certificate of candidacy because
she became a resident of many places" flies in the face of settled jurisprudence in which this Court carefully made distinctions between (actual)
residence and domicile for election law purposes. In Larena vs. Teves,
33
supra, we stressed:
[T]his court is of the opinion and so holds that a person who has his own house wherein he lives with his family in a
municipality without having ever had the intention of abandoning it, and without having lived either alone or with his family
in another municipality, has his residence in the former municipality, notwithstanding his having registered as an elector in
the other municipality in question and having been a candidate for various insular and provincial positions, stating every
time that he is a resident of the latter municipality.
More significantly, in Faypon vs. Quirino,
34
We explained that:
A citizen may leave the place of his birth to look for "greener pastures," as the saying goes, to improve his lot, and that, of
course includes study in other places, practice of his avocation, or engaging in business. When an election is to be held,
the citizen who left his birthplace to improve his lot may desire to return to his native town to cast his ballot but for
professional or business reasons, or for any other reason, he may not absent himself from his professional or business
activities; so there he registers himself as voter as he has the qualifications to be one and is not willing to give up or lose
the opportunity to choose the officials who are to run the government especially in national elections. Despite such

34
registration, the animus revertendi to his home, to his domicile or residence of origin has not forsaken him. This may be
the explanation why the registration of a voter in a place other than his residence of origin has not been deemed sufficient
to constitute abandonment or loss of such residence. It finds justification in the natural desire and longing of every person
to return to his place of birth. This strong feeling of attachment to the place of one's birth must be overcome by positive
proof of abandonment for another.
From the foregoing, it can be concluded that in its above-cited statements supporting its proposition that petitioner was ineligible to run for the position of
Representative of the First District of Leyte, the COMELEC was obviously referring to petitioner's various places of (actual) residence, not her domicile.
In doing so, it not only ignored settled jurisprudence on residence in election law and the deliberations of the constitutional commission but also the
provisions of the Omnibus Election Code (B.P. 881).
35

What is undeniable, however, are the following set of facts which establish the fact of petitioner's domicile, which we lift verbatim from the COMELEC's
Second Division's assailed Resolution:
36

In or about 1938 when respondent was a little over 8 years old, she established her domicile in Tacloban, Leyte (Tacloban
City). She studied in the Holy Infant Academy in Tacloban from 1938 to 1949 when she graduated from high school. She
pursued her college studies in St. Paul's College, now Divine Word University in Tacloban, where she earned her degree
in Education. Thereafter, she taught in the Leyte Chinese School, still in Tacloban City. In 1952 she went to Manila to
work with her cousin, the late speaker Daniel Z. Romualdez in his office in the House of Representatives. In 1954, she
married ex-President Ferdinand E. Marcos when he was still a congressman of Ilocos Norte and registered there as a
voter. When her husband was elected Senator of the Republic in 1959, she and her husband lived together in San Juan,
Rizal where she registered as a voter. In 1965, when her husband was elected President of the Republic of the
Philippines, she lived with him in Malacanang Palace and registered as a voter in San Miguel, Manila.
[I]n February 1986 (she claimed that) she and her family were abducted and kidnapped to Honolulu, Hawaii. In November
1991, she came home to Manila. In 1992, respondent ran for election as President of the Philippines and filed her
Certificate of Candidacy wherein she indicated that she is a resident and registered voter of San Juan, Metro Manila.
Applying the principles discussed to the facts found by COMELEC, what is inescapable is that petitioner held various residences for different purposes
during the last four decades. None of these purposes unequivocally point to an intention to abandon her domicile of origin in Tacloban, Leyte. Moreover,
while petitioner was born in Manila, as a minor she naturally followed the domicile of her parents. She grew up in Tacloban, reached her adulthood there
and eventually established residence in different parts of the country for various reasons. Even during her husband's presidency, at the height of the
Marcos Regime's powers, petitioner kept her close ties to her domicile of origin by establishing residences in Tacloban, celebrating her birthdays and
other important personal milestones in her home province, instituting well-publicized projects for the benefit of her province and hometown, and
establishing a political power base where her siblings and close relatives held positions of power either through the ballot or by appointment, always with
either her influence or consent. These well-publicized ties to her domicile of origin are part of the history and lore of the quarter century of Marcos power
in our country. Either they were entirely ignored in the COMELEC'S Resolutions, or the majority of the COMELEC did not know what the rest of the
country always knew: the fact of petitioner's domicile in Tacloban, Leyte.
Private respondent in his Comment, contends that Tacloban was not petitioner's domicile of origin because she did not live there until she was eight
years old. He avers that after leaving the place in 1952, she "abandoned her residency (sic) therein for many years and . . . (could not) re-establish her
domicile in said place by merely expressing her intention to live there again." We do not agree.
First, minor follows the domicile of his parents. As domicile, once acquired is retained until a new one is gained, it follows that in spite of the fact of
petitioner's being born in Manila, Tacloban, Leyte was her domicile of origin by operation of law. This domicile was not established only when her father
brought his family back to Leyte contrary to private respondent's averments.
Second, domicile of origin is not easily lost. To successfully effect a change of domicile, one must demonstrate:
37

1. An actual removal or an actual change of domicile;
2. A bona fide intention of abandoning the former place of residence and establishing a new one; and
3. Acts which correspond with the purpose.
In the absence of clear and positive proof based on these criteria, the residence of origin should be deemed to continue. Only with evidence showing
concurrence of all three requirements can the presumption of continuity or residence be rebutted, for a change of residence requires an actual and
deliberate abandonment, and one cannot have two legal residences at the same time.
38
In the case at bench, the evidence adduced by private
respondent plainly lacks the degree of persuasiveness required to convince this court that an abandonment of domicile of origin in favor of a domicile of
choice indeed occurred. To effect an abandonment requires the voluntary act of relinquishing petitioner's former domicile with an intent to supplant the
former domicile with one of her own choosing (domicilium voluntarium).
In this connection, it cannot be correctly argued that petitioner lost her domicile of origin by operation of law as a result of her marriage to the late
President Ferdinand E. Marcos in 1952. For there is a clearly established distinction between the Civil Code concepts of "domicile" and
"residence."
39
The presumption that the wife automatically gains the husband's domicile by operation of law upon marriage cannot be inferred from the
use of the term "residence" in Article 110 of the Civil Code because the Civil Code is one area where the two concepts are well delineated. Dr. Arturo
Tolentino, writing on this specific area explains:

35
In the Civil Code, there is an obvious difference between domicile and residence. Both terms imply relations between a
person and a place; but in residence, the relation is one of fact while in domicile it is legal or juridical, independent of the
necessity of physical presence.
40

Article 110 of the Civil Code provides:
Art. 110. The husband shall fix the residence of the family. But the court may exempt the wife from living with the
husband if he should live abroad unless in the service of the Republic.
A survey of jurisprudence relating to Article 110 or to the concepts of domicile or residence as they affect the female spouse upon marriage yields
nothing which would suggest that the female spouse automatically loses her domicile of origin in favor of the husband's choice of residence upon
marriage.
Article 110 is a virtual restatement of Article 58 of the Spanish Civil Code of 1889 which states:
La mujer esta obligada a seguir a su marido donde quiera que fije su residencia. Los Tribunales, sin embargo, podran con
justa causa eximirla de esta obligacion cuando el marido transende su residencia a ultramar o' a pais extranjero.
Note the use of the phrase "donde quiera su fije de residencia" in the aforequoted article, which means wherever (the husband) wishes to establish
residence. This part of the article clearly contemplates only actual residence because it refers to a positive act of fixing a family home or residence.
Moreover, this interpretation is further strengthened by the phrase "cuando el marido translade su residencia" in the same provision which means, "when
the husband shall transfer his residence," referring to another positive act of relocating the family to another home or place of actual residence. The
article obviously cannot be understood to refer to domicile which is a fixed,
fairly-permanent concept when it plainly connotes the possibility of transferring from one place to another not only once, but as often as the husband
may deem fit to move his family, a circumstance more consistent with the concept of actual residence.
The right of the husband to fix the actual residence is in harmony with the intention of the law to strengthen and unify the family, recognizing the fact that
the husband and the wife bring into the marriage different domiciles (of origin). This difference could, for the sake of family unity, be reconciled only by
allowing the husband to fix a single place of actual residence.
Very significantly, Article 110 of the Civil Code is found under Title V under the heading: RIGHTS AND OBLIGATIONS BETWEEN HUSBAND AND
WIFE. Immediately preceding Article 110 is Article 109 which obliges the husband and wife to live together, thus:
Art. 109. The husband and wife are obligated to live together, observe mutual respect and fidelity and render mutual
help and support.
The duty to live together can only be fulfilled if the husband and wife are physically together. This takes into account the situations where the couple has
many residences (as in the case of the petitioner). If the husband has to stay in or transfer to any one of their residences, the wife should necessarily be
with him in order that they may "live together." Hence, it is illogical to conclude that Art. 110 refers to "domicile" and not to "residence." Otherwise, we
shall be faced with a situation where the wife is left in the domicile while the husband, for professional or other reasons, stays in one of their (various)
residences. As Dr. Tolentino further explains:
Residence and Domicile Whether the word "residence" as used with reference to particular matters is synonymous with
"domicile" is a question of some difficulty, and the ultimate decision must be made from a consideration of the purpose
and intent with which the word is used. Sometimes they are used synonymously, at other times they are distinguished
from one another.
xxx xxx xxx
Residence in the civil law is a material fact, referring to the physical presence of a person in a place. A person can have
two or more residences, such as a country residence and a city residence. Residence is acquired by living in place; on the
other hand, domicile can exist without actually living in the place. The important thing for domicile is that, once residence
has been established in one place, there be an intention to stay there permanently, even if residence is also established in
some other
place.
41

In fact, even the matter of a common residence between the husband and the wife during the marriage is not an iron-clad principle; In cases applying the
Civil Code on the question of a common matrimonial residence, our jurisprudence has recognized certain situations
42
where the spouses could not be
compelled to live with each other such that the wife is either allowed to maintain a residence different from that of her husband or, for obviously practical
reasons, revert to her original domicile (apart from being allowed to opt for a new one). In De la Vina vs. Villareal
43
this Court held that "[a] married
woman may acquire a residence or domicile separate from that of her husband during the existence of the marriage where the husband has given cause
for divorce."
44
Note that the Court allowed the wife either to obtain new residence or to choose a new domicile in such an event. In instances where the
wife actually opts, .under the Civil Code, to live separately from her husband either by taking new residence or reverting to her domicile of origin, the
Court has held that the wife could not be compelled to live with her husband on pain of contempt. In Arroyo vs. Vasques de Arroyo
45
the Court held that:
Upon examination of the authorities, we are convinced that it is not within the province of the courts of this country to
attempt to compel one of the spouses to cohabit with, and render conjugal rights to, the other. Of course where the
property rights of one of the pair are invaded, an action for restitution of such rights can be maintained. But we are
disinclined to sanction the doctrine that an order, enforcible (sic) by process of contempt, may be entered to compel the
restitution of the purely personal right of consortium. At best such an order can be effective for no other purpose than to

36
compel the spouses to live under the same roof; and he experience of those countries where the courts of justice have
assumed to compel the cohabitation of married people shows that the policy of the practice is extremely questionable.
Thus in England, formerly the Ecclesiastical Court entertained suits for the restitution of conjugal rights at the instance of
either husband or wife; and if the facts were found to warrant it, that court would make a mandatory decree, enforceable
by process of contempt in case of disobedience, requiring the delinquent party to live with the other and render conjugal
rights. Yet this practice was sometimes criticized even by the judges who felt bound to enforce such orders, and
in Weldon v. Weldon (9 P.D. 52), decided in 1883, Sir James Hannen, President in the Probate, Divorce and Admiralty
Division of the High Court of Justice, expressed his regret that the English law on the subject was not the same as that
which prevailed in Scotland, where a decree of adherence, equivalent to the decree for the restitution of conjugal rights in
England, could be obtained by the injured spouse, but could not be enforced by imprisonment. Accordingly, in obedience
to the growing sentiment against the practice, the Matrimonial Causes Act (1884) abolished the remedy of imprisonment;
though a decree for the restitution of conjugal rights can still be procured, and in case of disobedience may serve in
appropriate cases as the basis of an order for the periodical payment of a stipend in the character of alimony.
In the voluminous jurisprudence of the United States, only one court, so far as we can discover, has ever attempted to
make a preemptory order requiring one of the spouses to live with the other; and that was in a case where a wife was
ordered to follow and live with her husband, who had changed his domicile to the City of New Orleans. The decision
referred to (Bahn v. Darby, 36 La. Ann., 70) was based on a provision of the Civil Code of Louisiana similar to article 56 of
the Spanish Civil Code. It was decided many years ago, and the doctrine evidently has not been fruitful even in the State
of Louisiana. In other states of the American Union the idea of enforcing cohabitation by process of contempt is rejected.
(21 Cyc., 1148).
In a decision of January 2, 1909, the Supreme Court of Spain appears to have affirmed an order of the Audiencia
Territorial de Valladolid requiring a wife to return to the marital domicile, and in the alternative, upon her failure to do so, to
make a particular disposition of certain money and effects then in her possession and to deliver to her husband, as
administrator of the ganancial property, all income, rents, and interest which might accrue to her from the property which
she had brought to the marriage. (113 Jur. Civ., pp. 1, 11) But it does not appear that this order for the return of the wife to
the marital domicile was sanctioned by any other penalty than the consequences that would be visited upon her in respect
to the use and control of her property; and it does not appear that her disobedience to that order would necessarily have
been followed by imprisonment for contempt.
Parenthetically when Petitioner was married to then Congressman Marcos, in 1954, petitioner was obliged by virtue of Article 110 of the Civil Code
to follow her husband's actual place of residence fixed by him. The problem here is that at that time, Mr. Marcos had several places of residence, among
which were San Juan, Rizal and Batac, Ilocos Norte. There is no showing which of these places Mr. Marcos did fix as his family's residence. But
assuming that Mr. Marcos had fixed any of these places as the conjugal residence, what petitioner gained upon marriage was actual residence. She did
not lose her domicile of origin.
On the other hand, the common law concept of "matrimonial domicile" appears to have been incorporated, as a result of our jurisprudential experiences
after the drafting of the Civil Code of 1950, into the New Family Code. To underscore the difference between the intentions of the Civil Code and the
Family Code drafters, the term residence has been supplanted by the term domicile in an entirely new provision (Art. 69) distinctly different in meaning
and spirit from that found in Article 110. The provision recognizes revolutionary changes in the concept of women's rights in the intervening years by
making the choice of domicile a product of mutual agreement between the spouses.
46

Without as much belaboring the point, the term residence may mean one thing in civil law (or under the Civil Code) and quite another thing in political
law. What stands clear is that insofar as the Civil Code is concerned-affecting the rights and obligations of husband and wife the term residence
should only be interpreted to mean "actual residence." The inescapable conclusion derived from this unambiguous civil law delineation therefore, is that
when petitioner married the former President in 1954, she kept her domicile of origin and merely gained a new home, not a domicilium necessarium.
Even assuming for the sake of argument that petitioner gained a new "domicile" after her marriage and only acquired a right to choose a new one after
her husband died, petitioner's acts following her return to the country clearly indicate that she not only impliedly but expressly chose her domicile of
origin (assuming this was lost by operation of law) as her domicile. This "choice" was unequivocally expressed in her letters to the Chairman of the
PCGG when petitioner sought the PCGG's permission to "rehabilitate (our) ancestral house in Tacloban and Farm in Olot, Leyte. . . to make them livable
for the Marcos family to have a home in our homeland."
47
Furthermore, petitioner obtained her residence certificate in 1992 in Tacloban, Leyte, while
living in her brother's house, an act which supports the domiciliary intention clearly manifested in her letters to the PCGG Chairman. She could not have
gone straight to her home in San Juan, as it was in a state of disrepair, having been previously looted by vandals. Her "homes" and "residences"
following her arrival in various parts of Metro Manila merely qualified as temporary or "actual residences," not domicile. Moreover, and proceeding from
our discussion pointing out specific situations where the female spouse either reverts to her domicile of origin or chooses a new one during the
subsistence of the marriage, it would be highly illogical for us to assume that she cannot regain her original domicile upon the death of her husband
absent a positive act of selecting a new one where situations exist within the subsistence of the marriage itself where the wife gains a domicile different
from her husband.
In the light of all the principles relating to residence and domicile enunciated by this court up to this point, we are persuaded that the facts established by
the parties weigh heavily in favor of a conclusion supporting petitioner's claim of legal residence or domicile in the First District of Leyte.
II. The jurisdictional issue
Petitioner alleges that the jurisdiction of the COMELEC had already lapsed considering that the assailed resolutions were rendered on April 24, 1995,
fourteen (14) days before the election in violation of Section 78 of the Omnibus Election Code.
48
Moreover, petitioner contends that it is the House of
Representatives Electoral Tribunal and not the COMELEC which has jurisdiction over the election of members of the House of Representatives in
accordance with Article VI Sec. 17 of the Constitution. This is untenable.
It is a settled doctrine that a statute requiring rendition of judgment within a specified time is generally construed to be merely directory,
49
"so that non-
compliance with them does not invalidate the judgment on the theory that if the statute had intended such result it would have clearly indicated it."
50
The

37
difference between a mandatory and a directory provision is often made on grounds of necessity. Adopting the same view held by several American
authorities, this court in Marcelino vs. Cruz held that:
51

The difference between a mandatory and directory provision is often determined on grounds of expediency, the reason
being that less injury results to the general public by disregarding than enforcing the letter of the law.
In Trapp v. Mc Cormick, a case calling for the interpretation of a statute containing a limitation of thirty (30) days within
which a decree may be entered without the consent of counsel, it was held that "the statutory provisions which may be
thus departed from with impunity, without affecting the validity of statutory proceedings, are usually those which relate to
the mode or time of doing that which is essential to effect the aim and purpose of the Legislature or some incident of the
essential act." Thus, in said case, the statute under examination was construed merely to be directory.
The mischief in petitioner's contending that the COMELEC should have abstained from rendering a decision after the period stated in the Omnibus
Election Code because it lacked jurisdiction, lies in the fact that our courts and other quasi-judicial bodies would then refuse to render judgments merely
on the ground of having failed to reach a decision within a given or prescribed period.
In any event, with the enactment of Sections 6 and 7 of R.A. 6646 in relation to Section 78 of B.P. 881,
52
it is evident that the respondent Commission
does not lose jurisdiction to hear and decide a pending disqualification case under Section 78 of B.P. 881 even after the elections.
As to the House of Representatives Electoral Tribunal's supposed assumption of jurisdiction over the issue of petitioner's qualifications after the May 8,
1995 elections, suffice it to say that HRET's jurisdiction as the sole judge of all contests relating to the elections, returns and qualifications of members of
Congress begins only after a candidate has become a member of the House of Representatives.
53
Petitioner not being a member of the House of
Representatives, it is obvious that the HRET at this point has no jurisdiction over the question.
It would be an abdication of many of the ideals enshrined in the 1987 Constitution for us to either to ignore or deliberately make distinctions in law solely
on the basis of the personality of a petitioner in a case. Obviously a distinction was made on such a ground here. Surely, many established principles of
law, even of election laws were flouted for the sake perpetuating power during the pre-EDSA regime. We renege on these sacred ideals, including the
meaning and spirit of EDSA ourselves bending established principles of principles of law to deny an individual what he or she justly deserves in law.
Moreover, in doing so, we condemn ourselves to repeat the mistakes of the past.
WHEREFORE, having determined that petitioner possesses the necessary residence qualifications to run for a seat in the House of Representatives in
the First District of Leyte, the COMELEC's questioned Resolutions dated April 24, May 7, May 11, and May 25, 1995 are hereby SET ASIDE.
Respondent COMELEC is hereby directed to order the Provincial Board of Canvassers to proclaim petitioner as the duly elected Representative of the
First District of Leyte.
SO ORDERED.
Feliciano, J., is on leave.

The Lawphil Project - Arellano Law Foundation



SECOND DIVISION
[A.M. No. MTJ-96-1088. July 19, 1996]
RODOLFO G. NAVARRO, complainant, vs. JUDGE HERNANDO C. DOMAGTOY, respondent.
D E C I S I O N
ROMERO, J .:
The complainant in this administrative case is the Municipal Mayor of Dapa, Surigao del Norte, Rodolfo G. Navarro. He has submitted evidence in
relation to two specific acts committed by respondent Municipal Circuit Trial Court Judge Hernando Domagtoy, which, he contends, exhibits gross
misconduct as well as inefficiency in office and ignorance of the law.
First, on September 27, 1994, respondent judge solemnized the wedding between Gaspar A. Tagadan and Arlyn F. Borga, despite the knowledge
that the groom is merely separated from his first wife.
Second, it is alleged that he performed a marriage ceremony between Floriano Dador Sumaylo and Gemma G. del Rosario outside his court's
jurisdiction on October 27, 1994. Respondent judge holds office and has jurisdiction in the Municipal Circuit Trial Court of Sta. Monica-Burgos, Surigao

38
del Norte. The wedding was solemnized at the respondent judge's residence in the municipality of Dapa, which does not fall within his jurisdictional area
of the municipalities of Sta. Monica and Burgos, located some 40 to 45 kilometers away from the municipality of Dapa, Surigao del Norte.
In his letter-comment to the Office of the Court Administrator, respondent judge avers that the office and name of the Municipal Mayor of Dapa
have been used by someone else, who, as the mayor's "lackey," is overly concerned with his actuations both as judge and as a private person. The
same person had earlier filed Administrative Matter No. 94-980-MTC, which was dismissed for lack of merit on September 15, 1994, and Administrative
Matter No. OCA-IPI-95-16, "Antonio Adapon v. Judge Hernando C. Domagtoy," which is still pending.
In relation to the charges against him, respondent judge seeks exculpation from his act of having solemnized the marriage between Gaspar
Tagadan, a married man separated from his wife, and Arlyn F. Borga by stating that he merely relied on the Affidavit issued by the Municipal Trial Judge
of Basey, Samar, confirming the fact that Mr. Tagadan and his first wife have not seen each other for almost seven years.
[1]
With respect to the second
charge, he maintains that in solemnizing the marriage between Sumaylo and del Rosario, he did not violate Article 7, paragraph 1 of the Family Code
which states that: "Marriage may be solemnized by: (1) Any incumbent member of the judiciary within the court's jurisdiction; and that Article 8 thereof
applies to the case in question.
The complaint was not referred, as is usual, for investigation, since the pleadings submitted were considered sufficient for a resolution of the
case.
[2]

Since the countercharges of sinister motives and fraud on the part of complainant have not been sufficiently proven, they wil l not be dwelt
upon. The acts complained of and respondent judge's answer thereto will suffice and can be objectively assessed by themselves to prove the latter's
malfeasance.
The certified true copy of the marriage contract between Gaspar Tagadan and Arlyn Borga states that Tagadan's civil status is "separated."
Despite this declaration, the wedding ceremony was solemnized by respondent judge. He presented in evidence a joint affidavit by Maurecio A. Labado,
Sr. and Eugenio Bullecer, subscribed and sworn to before Judge Demosthenes C. Duquilla, Municipal Trial Judge of Basey, Samar.
[3]
The affidavit was
not issued by the latter judge, as claimed by respondent judge, but merely acknowledged before him. In their affidavit, the affiants stated that they knew
Gaspar Tagadan to have been civilly married to Ida D. Pearanda in September 1983; that after thirteen years of cohabitation and having borne five
children, Ida Pearanda left the conjugal dwelling in Valencia, Bukidnon and that she has not returned nor been heard of for almost seven years, thereby
giving rise to the presumption that she is already dead.
In effect, Judge Domagtoy maintains that the aforementioned joint affidavit is sufficient proof of Ida Pearanda's presumptive death, and ample
reason for him to proceed with the marriage ceremony. We do not agree.
Article 41 of the Family Code expressly provides:
"A marriage contracted by any person during the subsistence of a previous marriage shall be null and void, unless before the celebration of the
subsequent marriage, the prior spouse had been absent for four consecutive years and the spouse present had a well-founded belief that the absent
spouse was already dead. In case of disappearance where there is danger of death under the circumstances set forth in the provisions of Articles 391 of
the Civil Code, an absence of only two years shall be sufficient.
For the purpose of contracting the subsequent marriage under the preceding paragraph, the spouse present must institute a summary proceeding as
provided in this Code for the declaration of presumptive death of the absentee, without prejudice to the effect of reappearance of the absent
spouse." (Emphasis added.)
There is nothing ambiguous or difficult to comprehend in this provision. In fact, the law is clear and simple. Even if the spouse present has a well-
founded belief that the absent spouse was already dead, a summary proceeding for the declaration of presumptive death is necessary in order to
contract a subsequent marriage, a mandatory requirement which has been precisely incorporated into the Family Code to discourage subsequent
marriages where it is not proven that the previous marriage has been dissolved or a missing spouse is factually or presumptively dead, in accordance
with pertinent provisions of law.
In the case at bar, Gaspar Tagadan did not institute a summary proceeding for the declaration of his first wife's presumptive death. Absent this
judicial declaration, he remains married to Ida Pearanda. Whether wittingly, or unwittingly, it was manifest error on the part of respondent judge to have
accepted the joint affidavit submitted by the groom. Such neglect or ignorance of the law has resulted in a bigamous, and therefore void,
marriage. Under Article 35 of the Family Code, "The following marriage shall be void from the beginning: (4) Those bigamous x x x marriages not falling
under Article 41."
The second issue involves the solemnization of a marriage ceremony outside the court's jurisdiction, covered by Articl es 7 and 8 of the Family
Code, thus:
"Art. 7. Marriage may be solemnized by:
(1) Any incumbent member of the judiciary within the court's jurisdiction;
x x x x x x xxx (Emphasis supplied.)
Art. 8. The marriage shall be solemnized publicly in the chambers of the judge or in open court, in the church, chapel or temple, or in the office of the
consul-general, consul or vice-consul, as the case may be, and not elsewhere, except in cases of marriages contracted on the point of death or in
remote places in accordance with Article 29 of this Code, or where both parties request the solemnizing officer in writing in which case the
marriage may be solemnized at a house or place designated by them in a sworn statement to that effect."
Respondent judge points to Article 8 and its exceptions as the justifications for his having solemnized the marriage between Floriano Sumaylo and
Gemma del Rosario outside of his court's jurisdiction. As the aforequoted provision states, a marriage can be held outside of the judge's chambers or
courtroom only in the following instances: (1) at the point of death, (2) in remote places in accordance with Article 29 or (3) upon request of both parties
in writing in a sworn statement to this effect. There is no pretense that either Sumaylo or del Rosario was at the point of death or in a remote
place. Moreover, the written request presented addressed to the respondent judge was made by only one party, Gemma del Rosario.
[4]


39
More importantly, the elementary principle underlying this provision is the authority of the solemnizing judge. Under Article 3, one of the formal
requisites of marriage is the "authority of the solemnizing officer." Under Article 7, marriage may be solemnized by, among others, "any incumbent
member of the judiciary within the court's jurisdiction." Article 8, which is a directory provision, refers only to the venue of the marriage ceremony and
does not alter or qualify the authority of the solemnizing officer as provided in the preceding provision. Non-compliance herewith will not invalidate the
marriage.
A priest who is commissioned and allowed by his local ordinary to marry the faithful, is authorized to do so only within the area of the diocese or
place allowed by his Bishop. An appellate court Justice or a Justice of this Court has jurisdiction over the entire Philippines to solemnize marriages,
regardless of the venue, as long as the requisites of the law are complied with. However, judges who are appointed to specific jurisdictions, may
officiate in weddings only within said areas and not beyond. Where a judge solemnizes a marriage outside his court's jurisdiction, there is a resultant
irregularity in the formal requisite laid down in Article 3, which while it may not affect the validity of the marriage, may subject the officiating official to
administrative liability.
[5]

Inasmuch as respondent judge's jurisdiction covers the municipalities of Sta. Monica and Burgos, he was not clothed with authority to solemnize a
marriage in the municipality of Dapa, Surigao del Norte. By citing Article 8 and the exceptions therein as grounds for the exercise of his misplaced
authority, respondent judge again demonstrated a lack of understanding of the basic principles of civil law.
Accordingly, the Court finds respondent to have acted in gross ignorance of the law. The legal principles applicable in the cases brought to our
attention are elementary and uncomplicated, prompting us to conclude that respondent's failure to apply them is due to a lack of comprehension of the
law.
The judiciary should be composed of persons who, if not experts, are at least, proficient in the law they are sworn to apply, more than the ordinary
laymen. They should be skilled and competent in understanding and applying the law. It is imperative that they be conversant with basic legal principles
like the ones involved in instant case.
[6]
It is not too much to expect them to know and apply the law intelligently.
[7]
Otherwise, the system of justice rests
on a shaky foundation indeed, compounded by the errors committed by those not learned in the law. While magistrates may at times make mistakes in
judgment, for which they are not penalized, the respondent judge exhibited ignorance of elementary provisions of law, in an area which has greatly
prejudiced the status of married persons.
The marriage between Gaspar Tagadan and Arlyn Borga is considered bigamous and void, there being a subsisting marriage between Gaspar
Tagadan and Ida Pearanda.
The Office of the Court Administrator recommends, in its Memorandum to the Court, a six-month suspension and a stern warning that a repetition
of the same or similar acts will be dealt with more severely. Considering that one of the marriages in question resulted in a bigamous union and
therefore void, and the other lacked the necessary authority of respondent judge, the Court adopts said recommendation. Respondent is advised to be
more circumspect in applying the law and to cultivate a deeper understanding of the law.
IN VIEW OF THE FOREGOING, respondent Judge Hernando C. Domagtoy is hereby SUSPENDED for a period of six (6) months and given a
STERN WARNING that a repetition of the same or similar acts will be dealt with more severely.
SO ORDERED.
Regalado (Chairman), Puno, Mendoza, and Torres, Jr., JJ., concur.
FIRST DIVISION
[A.M. No. MTJ-02-1390. April 11, 2002]
MERCEDITA MATA ARAES, petitioner, vs. JUDGE SALVADOR M. OCCIANO, respondent.
D E C I S I O N
PUNO, J .:
Petitioner Mercedita Mata Araes charges respondent judge with Gross Ignorance of the Law via a sworn Letter-Complaint dated 23 May 2001.
Respondent is the Presiding Judge of the Municipal Trial Court of Balatan, Camarines Sur. Petitioner alleges that on 17 February 2000, respondent
judge solemnized her marriage to her late groom Dominador B. Orobia without the requisite marriage license and at Nabua, Camarines Sur which is
outside his territorial jurisdiction.
They lived together as husband and wife on the strength of this marriage until her husband passed away. However, since the marriage was a
nullity, petitioners right to inherit the vast properties left by Orobia was not recognized. She was likewise deprived of receiving the pensions of Orobia,
a retired Commodore of the Philippine Navy.
Petitioner prays that sanctions be imposed against respondent judge for his illegal acts and unethical misrepresentations whi ch allegedly caused
her so much hardships, embarrassment and sufferings.
On 28 May 2001, the case was referred by the Office of the Chief Justice to then Acting Court Administrator Zenaida N. Elepao for appropriate
action. On 8 June 2001, the Office of the Court Administrator required respondent judge to comment.
In his Comment dated 5 July 2001, respondent judge averred that he was requested by a certain Juan Arroyo on 15 February 2000 to solemnize
the marriage of the parties on 17 February 2000. Having been assured that all the documents to the marriage were complete, he agreed to solemnize
the marriage in his sala at the Municipal Trial Court of Balatan, Camarines Sur. However, on 17 February 2000, Arroyo informed him that Orobia had a
difficulty walking and could not stand the rigors of travelling to Balatan which is located almost 25 kilometers from his residence in Nabua. Arroyo then
requested if respondent judge could solemnize the marriage in Nabua, to which request he acceded.

40
Respondent judge further avers that before he started the ceremony, he carefully examined the documents submitted to him by petitioner. When
he discovered that the parties did not possess the requisite marriage license, he refused to solemnize the marriage and suggested its resetting to
another date. However, due to the earnest pleas of the parties, the influx of visitors, and the delivery of provisions for the occasion, he proceeded to
solemnize the marriage out of human compassion. He also feared that if he reset the wedding, it might aggravate the physical condition of Orobia who
just suffered from a stroke. After the solemnization, he reiterated the necessity for the marriage license and admonished the parties that their failure to
give it would render the marriage void. Petitioner and Orobia assured respondent judge that they would give the license to him in the afternoon of that
same day. When they failed to comply, respondent judge followed it up with Arroyo but the latter only gave him the same reassurance that the marriage
license would be delivered to his sala at the Municipal Trial Court of Balatan, Camarines Sur.
Respondent judge vigorously denies that he told the contracting parties that their marriage is valid despite the absence of a marriage license. He
attributes the hardships and embarrassment suffered by the petitioner as due to her own fault and negligence.
On 12 September 2001, petitioner filed her Affidavit of Desistance dated 28 August 2001 with the Office of the Court Administ rator. She attested
that respondent judge initially refused to solemnize her marriage due to the want of a duly issued marriage license and that it was because of her
prodding and reassurances that he eventually solemnized the same. She confessed that she filed this administrative case out of rage. However, after
reading the Comment filed by respondent judge, she realized her own shortcomings and is now bothered by her conscience.
Reviewing the records of the case, it appears that petitioner and Orobia filed their Application for Marriage License on 5 January 2000. It was
stamped in this Application that the marriage license shall be issued on 17 January 2000. However, neither petitioner nor Orobia claimed it.
It also appears that the Office of the Civil Registrar General issued a Certification that it has no record of such marriage that allegedly took place
on 17 February 2000. Likewise, the Office of the Local Civil Registrar of Nabua, Camarines Sur issued another Certification dated 7 May 2001 that it
cannot issue a true copy of the Marriage Contract of the parties since it has no record of their marriage.
On 8 May 2001, petitioner sought the assistance of respondent judge so the latter could communicate with the Office of the Local Civil Registrar of
Nabua, Camarines Sur for the issuance of her marriage license. Respondent judge wrote the Local Civil Registrar of Nabua, Camarines Sur. In a letter
dated 9 May 2001, a Clerk of said office, Grace T. Escobal, informed respondent judge that their office cannot issue the marriage license due to the
failure of Orobia to submit the Death Certificate of his previous spouse.
The Office of the Court Administrator, in its Report and Recommendation dated 15 November 2000, found the respondent judge guilty of
solemnizing a marriage without a duly issued marriage license and for doing so outside his territorial jurisdiction. A fine of P5,000.00 was recommended
to be imposed on respondent judge.
We agree.
Under the Judiciary Reorganization Act of 1980, or B.P.129, the authority of the regional trial court judges and judges of inferior courts to
solemnize marriages is confined to their territorial jurisdiction as defined by the Supreme Court.
The case at bar is not without precedent. In Navarro vs. Domagtoy,
[1]
respondent judge held office and had jurisdiction in the Municipal Circuit
Trial Court of Sta. Monica-Burgos, Surigao del Norte. However, he solemnized a wedding at his residence in the municipality of Dapa, Surigao del Norte
which did not fall within the jurisdictional area of the municipalities of Sta. Monica and Burgos. We held that:
A priest who is commissioned and allowed by his local ordinance to marry the faithful is authorized to do so only within the area or diocese or place
allowed by his Bishop. An appellate court Justice or a Justice of this Court has jurisdiction over the entire Philippines to solemnize marriages, regardless
of the venue, as long as the requisites of the law are complied with. However, judges who are appointed to specific jurisdictions, may officiate in
weddings only within said areas and not beyond. Where a judge solemnizes a marriage outside his courts jurisdiction, there is a resultant
irregularity in the formal requisite laid down in Article 3, which while it may not affect the validity of the marriage, may subject the officiating
official to administrative liability.
[2]
(Emphasis supplied.)
In said case, we suspended respondent judge for six (6) months on the ground that his act of solemnizing a marriage outside his jurisdiction
constitutes gross ignorance of the law. We further held that:
The judiciary should be composed of persons who, if not experts, are at least, proficient in the law they are sworn to apply, more than the ordinary
laymen. They should be skilled and competent in understanding and applying the law. It is imperative that they be conversant with basic legal principles
like the ones involved in the instant case. x x x While magistrates may at times make mistakes in judgment, for which they are not penalized, the
respondent judge exhibited ignorance of elementary provisions of law, in an area which has greatly prejudiced the status of married persons.
[3]

In the case at bar, the territorial jurisdiction of respondent judge is limited to the municipality of Balatan, Camarines Sur. His act of solemnizing the
marriage of petitioner and Orobia in Nabua, Camarines Sur therefore is contrary to law and subjects him to administrative liability. His act may not
amount to gross ignorance of the law for he allegedly solemnized the marriage out of human compassion but nonetheless, he cannot avoid liability for
violating the law on marriage.
Respondent judge should also be faulted for solemnizing a marriage without the requisite marriage license. In People vs. Lara,
[4]
we held that a
marriage which preceded the issuance of the marriage license is void, and that the subsequent issuance of such license cannot render valid or even add
an iota of validity to the marriage. Except in cases provided by law, it is the marriage license that gives the solemnizing officer the authority to solemnize
a marriage. Respondent judge did not possess such authority when he solemnized the marriage of petitioner. In this respect, respondent judge acted in
gross ignorance of the law.
Respondent judge cannot be exculpated despite the Affidavit of Desistance filed by petitioner. This Court has consistently held in a catena of
cases that the withdrawal of the complaint does not necessarily have the legal effect of exonerating respondent from discipli nary action. Otherwise, the
prompt and fair administration of justice, as well as the discipline of court personnel, would be undermined.
[5]
Disciplinary actions of this nature do not
involve purely private or personal matters. They can not be made to depend upon the will of every complainant who may, for one reason or another,
condone a detestable act. We cannot be bound by the unilateral act of a complainant in a matter which involves the Courts constitutional power to
discipline judges. Otherwise, that power may be put to naught, undermine the trust character of a public office and impair the integrity and dignity of this
Court as a disciplining authority.
[6]

WHEREFORE, respondent Judge Salvador M. Occiano, Presiding Judge of the Municipal Trial Court of Balatan, Camarines Sur, is fined
P5,000.00 pesos with a STERN WARNING that a repetition of the same or similar offense in the future will be dealt with more severely.

41
SO ORDERED.
Davide, Jr., C.J., (Chairman), Kapunan, and Ynares-Santiago, JJ., concur.

Republic of the Philippines
SUPREME COURT
Manila
FIRST DIVISION
G.R. No. 133778 March 14, 2000
ENGRACE NIAL for Herself and as Guardian ad Litem of the minors BABYLINE NIAL, INGRID NIAL, ARCHIE NIAL & PEPITO NIAL,
JR., petitioners,
vs.
NORMA BAYADOG, respondent.
YNARES-SANTIAGO, J .:
May the heirs of a deceased person file a petition for the declaration of nullity of his marriage after his death?
Pepito Nial was married to Teodulfa Bellones on September 26, 1974. Out of their marriage were born herein petitioners. Teodulfa was shot by Pepito
resulting in her death on April 24, 1985. One year and 8 months thereafter or on December 11, 1986, Pepito and respondent Norma Badayog got
married without any marriage license. In lieu thereof, Pepito and Norma executed an affidavit dated December 11, 1986 stating that they had lived
together as husband and wife for at least five years and were thus exempt from securing a marriage license. On February 19, 1997, Pepito died in a car
accident. After their father's death, petitioners filed a petition for declaration of nullity of the marriage of Pepito to Norma alleging that the said marriage
was void for lack of a marriage license. The case was filed under the assumption that the validity or invalidity of the second marriage would affect
petitioner's successional rights. Norma filed a motion to dismiss on the ground that petitioners have no cause of action since they are not among the
persons who could file an action for "annulment of marriage" under Article 47 of the Family Code.
Judge Ferdinand J. Marcos of the Regional Trial Court of Toledo City, Cebu, Branch 59, dismissed the petition after finding that the Family Code is
"rather silent, obscure, insufficient" to resolve the following issues:
(1) Whether or not plaintiffs have a cause of action against defendant in asking for the declaration of the nullity of marriage of their
deceased father, Pepito G. Nial, with her specially so when at the time of the filing of this instant suit, their father Pepito G. Nial is
already dead;
(2) Whether or not the second marriage of plaintiffs' deceased father with defendant is null and void ab initio;
(3) Whether or not plaintiffs are estopped from assailing the validity of the second marriage after it was dissolved due to their father's
death.
1

Thus, the lower court ruled that petitioners should have filed the action to declare null and void their father's marriage to respondent before his death,
applying by analogy Article 47 of the Family Code which enumerates the time and the persons who could initiate an action for annulment of
marriage.
2
Hence, this petition for review with this Court grounded on a pure question of law.
This petition was originally dismissed for non-compliance with Section 11, Rule 13 of the 1997 Rules of Civil Procedure, and because "the verification
failed to state the basis of petitioner's averment that the allegations in the petition are "true and correct"." It was thus treated as an unsigned pleading
which produces no legal effect under Section 3, Rule 7, of the 1997 Rules.
3
However, upon motion of petitioners, this Court reconsidered the dismissal
and reinstated the petition for review.
4

The two marriages involved herein having been solemnized prior to the effectivity of the Family Code (FC), the applicable law to determine their validity
is the Civil Code which was the law in effect at the time of their celebration.
5
A valid marriage license is a requisite of marriage under Article 53 of the
Civil Code,
6
the absence of which renders the marriage void ab initio pursuant to Article 80(3)
7
in relation to Article 58.
8
The requirement and issuance
of marriage license is the State's demonstration of its involvement and participation in every marriage, in the maintenance of which the general public is
interested.
9
This interest proceeds from the constitutional mandate that the State recognizes the sanctity of family life and of affording protection to the
family as a basic "autonomous social institution."
10
Specifically, the Constitution considers marriage as an "inviolable social institution," and is the
foundation of family life which shall be protected by the State.
11
This is why the Family Code considers marriage as "a special contract of permanent
union"
12
and case law considers it "not just an adventure but a lifetime commitment."
13

However, there are several instances recognized by the Civil Code wherein a marriage license is dispensed with, one of which is that provided in Article
76,
14
referring to the marriage of a man and a woman who have lived together and exclusively with each other as husband and wife for a continuous
and unbroken period of at least five years before the marriage. The rationale why no license is required in such case is to avoid exposing the parties to
humiliation, shame and embarrassment concomitant with the scandalous cohabitation of persons outside a valid marriage due to the publication of every
applicant's name for a marriage license. The publicity attending the marriage license may discourage such persons from legitimizing their status.
15
To
preserve peace in the family, avoid the peeping and suspicious eye of public exposure and contain the source of gossip arising from the publication of
their names, the law deemed it wise to preserve their privacy and exempt them from that requirement.

42
There is no dispute that the marriage of petitioners' father to respondent Norma was celebrated without any marriage license. In lieu thereof, they
executed an affidavit stating that "they have attained the age of majority, and, being unmarried, have lived together as husband and wife for at least five
years, and that we now desire to marry each other."
16
The only issue that needs to be resolved pertains to what nature of cohabitation is contemplated
under Article 76 of the Civil Code to warrant the counting of the five year period in order to exempt the future spouses from securing a marriage license.
Should it be a cohabitation wherein both parties are capacitated to marry each other during the entire five-year continuous period or should it be a
cohabitation wherein both parties have lived together and exclusively with each other as husband and wife during the entire five-year continuous period
regardless of whether there is a legal impediment to their being lawfully married, which impediment may have either disappeared or intervened
sometime during the cohabitation period?
Working on the assumption that Pepito and Norma have lived together as husband and wife for five years without the benefit of marriage, that five-year
period should be computed on the basis of a cohabitation as "husband and wife" where the only missing factor is the special contract of marriage to
validate the union. In other words, the five-year common-law cohabitation period, which is counted back from the date of celebration of marriage, should
be a period of legal union had it not been for the absence of the marriage. This 5-year period should be the years immediately before the day of the
marriage and it should be a period of cohabitation characterized by exclusivity meaning no third party was involved at anytime within the 5 years and
continuity that is unbroken. Otherwise, if that continuous 5-year cohabitation is computed without any distinction as to whether the parties were
capacitated to marry each other during the entire five years, then the law would be sanctioning immorality and encouraging parties to have common law
relationships and placing them on the same footing with those who lived faithfully with their spouse. Marriage being a special relationship must be
respected as such and its requirements must be strictly observed. The presumption that a man and a woman deporting themselves as husband and wife
is based on the approximation of the requirements of the law. The parties should not be afforded any excuse to not comply with every single requirement
and later use the same missing element as a pre-conceived escape ground to nullify their marriage. There should be no exemption from securing a
marriage license unless the circumstances clearly fall within the ambit of the exception. It should be noted that a license is required in order to notify the
public that two persons are about to be united in matrimony and that anyone who is aware or has knowledge of any impediment to the union of the two
shall make it known to the local civil registrar.
17
The Civil Code provides:
Art. 63: . . . This notice shall request all persons having knowledge of any impediment to the marriage to advice the local civil
registrar thereof. . . .
Art. 64: Upon being advised of any alleged impediment to the marriage, the local civil registrar shall forthwith make an investigation,
examining persons under oath. . . .
This is reiterated in the Family Code thus:
Art. 17 provides in part: . . . This notice shall request all persons having knowledge of any impediment to the marriage to advise the
local civil registrar thereof. . . .
Art. 18 reads in part: . . . In case of any impediment known to the local civil registrar or brought to his attention, he shall note down
the particulars thereof and his findings thereon in the application for a marriage license. . . .
This is the same reason why our civil laws, past or present, absolutely prohibited the concurrence of multiple marriages by the same person during the
same period. Thus, any marriage subsequently contracted during the lifetime of the first spouse shall be illegal and void,
18
subject only to the exception
in cases of absence or where the prior marriage was dissolved or annulled. The Revised Penal Code complements the civil law in that the contracting of
two or more marriages and the having of extramarital affairs are considered felonies, i.e., bigamy and concubinage and adultery.
19
The law sanctions
monogamy.
In this case, at the time of Pepito and respondent's marriage, it cannot be said that they have lived with each other as husband and wife for at least five
years prior to their wedding day. From the time Pepito's first marriage was dissolved to the time of his marriage with respondent, only about twenty
months had elapsed. Even assuming that Pepito and his first wife had separated in fact, and thereafter both Pepito and respondent had started living
with each other that has already lasted for five years, the fact remains that their five-year period cohabitation was not the cohabitation contemplated by
law. It should be in the nature of a perfect union that is valid under the law but rendered imperfect only by the absence of the marriage contract. Pepito
had a subsisting marriage at the time when he started cohabiting with respondent. It is immaterial that when they lived with each other, Pepito had
already been separated in fact from his lawful spouse. The subsistence of the marriage even where there was actual severance of the filial
companionship between the spouses cannot make any cohabitation by either spouse with any third party as being one as "husband and wife".
Having determined that the second marriage involved in this case is not covered by the exception to the requirement of a marriage license, it is void ab
initio because of the absence of such element.
The next issue to be resolved is: do petitioners have the personality to file a petition to declare their father's marriage void after his death?
Contrary to respondent judge's ruling, Article 47 of the Family Code
20
cannot be applied even by analogy to petitions for declaration of nullity of
marriage. The second ground for annulment of marriage relied upon by the trial court, which allows "the sane spouse" to file an annulment suit "at
anytime before the death of either party" is inapplicable. Article 47 pertains to the grounds, periods and persons who can file an annulment suit, not a
suit for declaration of nullity of marriage. The Code is silent as to who can file a petition to declare the nullity of a marriage. Voidable and void marriages
are not identical. A marriage that is annulable is valid until otherwise declared by the court; whereas a marriage that is void ab initio is considered as
having never to have taken place
21
and cannot be the source of rights. The first can be generally ratified or confirmed by free cohabitation or prescription
while the other can never be ratified. A voidable marriage cannot be assailed collaterally except in a direct proceeding while a void marriage can be
attacked collaterally. Consequently, void marriages can be questioned even after the death of either party but voidable marriages can be assailed only
during the lifetime of the parties and not after death of either, in which case the parties and their offspring will be left as if the marriage had been perfectly
valid.
22
That is why the action or defense for nullity is imprescriptible, unlike voidable marriages where the action prescribes. Only the parties to a
voidable marriage can assail it but any proper interested party may attack a void marriage. Void marriages have no legal effects except those declared
by law concerning the properties of the alleged spouses, regarding co-ownership or ownership through actual joint contribution,
23
and its effect on the
children born to such void marriages as provided in Article 50 in relation to Article 43 and 44 as well as Article 51, 53 and 54 of the Family Code. On the

43
contrary, the property regime governing voidable marriages is generally conjugal partnership and the children conceived before its annulment are
legitimate.
Contrary to the trial court's ruling, the death of petitioner's father extinguished the alleged marital bond between him and respondent. The conclusion is
erroneous and proceeds from a wrong premise that there was a marriage bond that was dissolved between the two. It should be noted that their
marriage was void hence it is deemed as if it never existed at all and the death of either extinguished nothing.
Jurisprudence under the Civil Code states that no judicial decree is necessary in order to establish the nullity of a marriage.
24
"A void marriage does not
require a judicial decree to restore the parties to their original rights or to make the marriage void but though no sentence of avoidance be absolutely
necessary, yet as well for the sake of good order of society as for the peace of mind of all concerned, it is expedient that the nullity of the marriage
should be ascertained and declared by the decree of a court of competent jurisdiction."
25
"Under ordinary circumstances, the effect of a void marriage,
so far as concerns the conferring of legal rights upon the parties, is as though no marriage had ever taken place. And therefore, being good for no legal
purpose, its invalidity can be maintained in any proceeding in which the fact of marriage may be material, either direct or collateral, in any civil court
between any parties at any time, whether before or after the death of either or both the husband and the wife, and upon mere proof of the facts rendering
such marriage void, it will be disregarded or treated as non-existent by the courts." It is not like a voidable marriage which cannot be collaterally attacked
except in direct proceeding instituted during the lifetime of the parties so that on the death of either, the marriage cannot be impeached, and is made
good ab initio.
26
But Article 40 of the Family Code expressly provides that there must be a judicial declaration of the nullity of a previous marriage,
though void, before a party can enter into a second marriage
27
and such absolute nullity can be based only on a final judgment to that effect.
28
For the
same reason, the law makes either the action or defense for the declaration of absolute nullity of marriage imprescriptible.
29
Corollarily, if the death of
either party would extinguish the cause of action or the ground for defense, then the same cannot be considered imprescriptible.
However, other than for purposes of remarriage, no judicial action is necessary to declare a marriage an absolute nullity.1wphi1 For other purposes,
such as but not limited to determination of heirship, legitimacy or illegitimacy of a child, settlement of estate, dissolution of property regime, or a criminal
case for that matter, the court may pass upon the validity of marriage even in a suit not directly instituted to question the same so long as it is essential
to the determination of the case. This is without prejudice to any issue that may arise in the case. When such need arises, a final judgment of declaration
of nullity is necessary even if the purpose is other than to remarry. The clause "on the basis of a final judgment declaring such previous marriage void" in
Article 40 of the Family Code connotes that such final judgment need not be obtained only for purpose of remarriage.
WHEREFORE, the petition is GRANTED. The assailed Order of the Regional Trial Court, Toledo City, Cebu, Branch 59, dismissing Civil Case No. T-
639, is REVERSED and SET ASIDE. The said case is ordered REINSTATED.1wphi1.nt
SO ORDERED.
Davide, Jr., C.J., Puno and Kapunan, JJ., concur.
Pardo, J., on official business abroad.

The Lawphil Project - Arellano Law Foundation

FIRST DIVISION
[A.M. No. MTJ-00-1329. March 8, 2001]
HERMINIA BORJA-MANZANO, petitioner, vs. JUDGE ROQUE R. SANCHEZ, MTC, Infanta, Pangasinan, respondent.
R E S O L U T I O N
DAVIDE, JR., C.J .:
The solemnization of a marriage between two contracting parties who were both bound by a prior existing marriage is the bone of contention of the
instant complaint against respondent Judge Roque R. Sanchez, Municipal Trial Court, Infanta, Pangasinan. For this act, complainant Herminia Borja-
Manzano charges respondent Judge with gross ignorance of the law in a sworn Complaint-Affidavit filed with the Office of the Court Administrator on 12
May 1999.
Complainant avers that she was the lawful wife of the late David Manzano, having been married to him on 21 May 1966 in San Gabriel Archangel
Parish, Araneta Avenue, Caloocan City.
[1]
Four children were born out of that marriage.
[2]
On 22 March 1993, however, her husband contracted another
marriage with one Luzviminda Payao before respondent Judge.
[3]
When respondent Judge solemnized said marriage, he knew or ought to know that the
same was void and bigamous, as the marriage contract clearly stated that both contracting parties were separated.
Respondent Judge, on the other hand, claims in his Comment that when he officiated the marriage between Manzano and Payao he did not know
that Manzano was legally married. What he knew was that the two had been living together as husband and wife for seven years already without the
benefit of marriage, as manifested in their joint affidavit.
[4]
According to him, had he known that the late Manzano was married, he would have advised
the latter not to marry again; otherwise, he (Manzano) could be charged with bigamy. He then prayed that the complaint be dismissed for lack of merit
and for being designed merely to harass him.
After an evaluation of the Complaint and the Comment, the Court Administrator recommended that respondent Judge be found guil ty of gross
ignorance of the law and be ordered to pay a fine of P2,000, with a warning that a repetition of the same or similar act would be dealt with more severely.

44
On 25 October 2000, this Court required the parties to manifest whether they were willing to submit the case for resolution on the basis of the
pleadings thus filed. Complainant answered in the affirmative.
For his part, respondent Judge filed a Manifestation reiterating his plea for the dismissal of the complaint and setting aside hi s earlier
Comment. He therein invites the attention of the Court to two separate affidavits
[5]
of the late Manzano and of Payao, which were allegedly unearthed by
a member of his staff upon his instruction. In those affidavits, both David Manzano and Luzviminda Payao expressly stated that they were married to
Herminia Borja and Domingo Relos, respectively; and that since their respective marriages had been marked by constant quarrel s, they had both left
their families and had never cohabited or communicated with their spouses anymore. Respondent Judge alleges that on the basis of those affidavits, he
agreed to solemnize the marriage in question in accordance with Article 34 of the Family Code.
We find merit in the complaint.
Article 34 of the Family Code provides:
No license shall be necessary for the marriage of a man and a woman who have lived together as husband and wife for at least five years and without
any legal impediment to marry each other. The contracting parties shall state the foregoing facts in an affidavit before any person authorized by law to
administer oaths. The solemnizing officer shall also state under oath that he ascertained the qualifications of the contracting parties and found no legal
impediment to the marriage.
For this provision on legal ratification of marital cohabitation to apply, the following requisites must concur:
1. The man and woman must have been living together as husband and wife for at least five years before the marriage;
2. The parties must have no legal impediment to marry each other;
3. The fact of absence of legal impediment between the parties must be present at the time of marriage;
4. The parties must execute an affidavit stating that they have lived together for at least five years [and are without legal impediment to
marry each other]; and
5. The solemnizing officer must execute a sworn statement that he had ascertained the qualifications of the parties and that he had found no
legal impediment to their marriage.
[6]

Not all of these requirements are present in the case at bar. It is significant to note that in their separate affidavits executed on 22 March 1993 and
sworn to before respondent Judge himself, David Manzano and Luzviminda Payao expressly stated the fact of their prior existing marriage. Also, in their
marriage contract, it was indicated that both were separated.
Respondent Judge knew or ought to know that a subsisting previous marriage is a diriment impediment, which would make the subsequent
marriage null and void.
[7]
In fact, in his Comment, he stated that had he known that the late Manzano was married he would have discouraged him from
contracting another marriage. And respondent Judge cannot deny knowledge of Manzanos and Payaos subsisting previous marriage, as the same
was clearly stated in their separate affidavits which were subscribed and sworn to before him.
The fact that Manzano and Payao had been living apart from their respective spouses for a long time already is immaterial. Article 63(1) of the
Family Code allows spouses who have obtained a decree of legal separation to live separately from each other, but in such a case the marriage bonds
are not severed. Elsewise stated, legal separation does not dissolve the marriage tie, much less authorize the parties to remarry. This holds true all
the more when the separation is merely de facto, as in the case at bar.
Neither can respondent Judge take refuge on the Joint Affidavit of David Manzano and Luzviminda Payao stating that they had been cohabiting as
husband and wife for seven years. Just like separation, free and voluntary cohabitation with another person for at least five years does not severe the
tie of a subsisting previous marriage. Marital cohabitation for a long period of time between two individuals who are legally capacitatedto marry each
other is merely a ground for exemption from marriage license. It could not serve as a justification for respondent Judge to solemnize a subsequent
marriage vitiated by the impediment of a prior existing marriage.
Clearly, respondent Judge demonstrated gross ignorance of the law when he solemnized a void and bigamous marriage. The maxim ignorance
of the law excuses no one has special application to judges,
[8]
who, under Rule 1.01 of the Code of Judicial Conduct, should be the embodiment of
competence, integrity, and independence. It is highly imperative that judges be conversant with the law and basic legal principles.
[9]
And when the law
transgressed is simple and elementary, the failure to know it constitutes gross ignorance of the law.
[10]

ACCORDINGLY, the recommendation of the Court Administrator is hereby ADOPTED, with the MODIFICATION that the amount of fine to be
imposed upon respondent Judge Roque Sanchez is increased to P20,000.
SO ORDERED.
Puno, Kapunan, Pardo, and Ynares-Santiago, JJ., concur.


Republic of the Philippines
SUPREME COURT
Manila
THIRD DIVISION
G.R. No. 175581 March 28, 2008
REPUBLIC OF THE PHILIPPINES, Petitioner,
vs.
JOSE A. DAYOT, Respondent.

45
x - - - - - - - - - - - - - - - - - - - - - - -x
G.R. No. 179474
FELISA TECSON-DAYOT, Petitioner,
vs.
JOSE A. DAYOT, Respondent.
D E C I S I O N
CHICO-NAZARIO, J .:
Before us are two consolidated petitions. G.R. No. 175581 and G.R. No. 179474 are Petitions for Review under Rule 45 of the Rules of Court filed by the
Republic of the Philippines and Felisa Tecson-Dayot (Felisa), respectively, both challenging the Amended Decision
1
of the Court of Appeals, dated 7
November 2006, in CA-G.R. CV No. 68759, which declared the marriage between Jose Dayot (Jose) and Felisa void ab initio.
The records disclose that on 24 November 1986, Jose and Felisa were married at the Pasay City Hall. The marriage was solemnized by Rev. Tomas V.
Atienza.
2
In lieu of a marriage license, Jose and Felisa executed a sworn affidavit,
3
also dated 24 November 1986, attesting that both of them had
attained the age of maturity, and that being unmarried, they had lived together as husband and wife for at least five years.
On 7 July 1993, Jose filed a Complaint
4
for Annulment and/or Declaration of Nullity of Marriage with the Regional Trial Court (RTC), Bian, Laguna,
Branch 25. He contended that his marriage with Felisa was a sham, as no marriage ceremony was celebrated between the parties; that he did not
execute the sworn affidavit stating that he and Felisa had lived as husband and wife for at least five years; and that his consent to the marriage was
secured through fraud.
In his Complaint, Jose gave his version of the events which led to his filing of the same. According to Jose, he was introduced to Felisa in 1986.
Immediately thereafter, he came to live as a boarder in Felisas house, the latter being his landlady. Some three weeks later, Felisa requested him to
accompany her to the Pasay City Hall, ostensibly so she could claim a package sent to her by her brother from Saudi Arabia. At the Pasay City Hall,
upon a pre-arranged signal from Felisa, a man bearing three folded pieces of paper approached them. They were told that Jose needed to sign the
papers so that the package could be released to Felisa. He initially refused to do so. However, Felisa cajoled him, and told him that his refusal could get
both of them killed by her brother who had learned about their relationship. Reluctantly, he signed the pieces of paper, and gave them to the man who
immediately left. It was in February 1987 when he discovered that he had contracted marriage with Felisa. He alleged that he saw a piece of paper lying
on top of the table at the sala of Felisas house. When he perused the same, he discovered that it was a copy of his marriage contract with Felisa. When
he confronted Felisa, the latter feigned ignorance.
In opposing the Complaint, Felisa denied Joses allegations and defended the validity of their marriage. She declared that they had maintained their
relationship as man and wife absent the legality of marriage in the early part of 1980, but that she had deferred contracting marriage with him on account
of their age difference.
5
In her pre-trial brief, Felisa expounded that while her marriage to Jose was subsisting, the latter contracted marriage with a
certain Rufina Pascual (Rufina) on 31 August 1990. On 3 June 1993, Felisa filed an action for bigamy against Jose. Subsequently, she filed an
administrative complaint against Jose with the Office of the Ombudsman, since Jose and Rufina were both employees of the National Statistics and
Coordinating Board.
6
The Ombudsman found Jose administratively liable for disgraceful and immoral conduct, and meted out to him the penalty of
suspension from service for one year without emolument.
7

On 26 July 2000, the RTC rendered a Decision
8
dismissing the Complaint. It disposed:
WHEREFORE, after a careful evaluation and analysis of the evidence presented by both parties, this Court finds and so holds that the [C]omplaint does
not deserve a favorable consideration. Accordingly, the above-entitled case is hereby ordered DISMISSED with costs against [Jose].
9

The RTC ruled that from the testimonies and evidence presented, the marriage celebrated between Jose and Felisa on 24 November 1986 was valid. It
dismissed Joses version of the story as implausible, and rationalized that:
Any person in his right frame of mind would easily suspect any attempt to make him or her sign a blank sheet of paper. [Jose] could have already
detected that something was amiss, unusual, as they were at Pasay City Hall to get a package for [Felisa] but it [was] he who was made to sign the
pieces of paper for the release of the said package. Another indirect suggestion that could have put him on guard was the fact that, by his own
admission, [Felisa] told him that her brother would kill them if he will not sign the papers. And yet it took him, more or less, three months to "discover"
that the pieces of paper that he signed was [sic] purportedly the marriage contract. [Jose] does not seem to be that ignorant, as perceived by this Court,
to be "taken in for a ride" by [Felisa.]
[Joses] claim that he did not consent to the marriage was belied by the fact that he acknowledged Felisa Tecson as his wife when he wrote [Felisas]
name in the duly notarized statement of assets and liabilities he filled up on May 12, 1988, one year after he discovered the marriage contract he is now
claiming to be sham and false. [Jose], again, in his company I.D., wrote the name of [Felisa] as the person to be contacted in case of emergency. This
Court does not believe that the only reason why her name was written in his company I.D. was because he was residing there then. This is just but a
lame excuse because if he really considers her not his lawfully wedded wife, he would have written instead the name of his sister.
When [Joses] sister was put into the witness stand, under oath, she testified that she signed her name voluntarily as a witness to the marriage in the
marriage certificate (T.S.N., page 25, November 29, 1996) and she further testified that the signature appearing over the name of Jose Dayot was the
signature of his [sic] brother that he voluntarily affixed in the marriage contract (page 26 of T.S.N. taken on November 29, 1996), and when she was
asked by the Honorable Court if indeed she believed that Felisa Tecson was really chosen by her brother she answered yes. The testimony of his sister
all the more belied his claim that his consent was procured through fraud.
10


46
Moreover, on the matter of fraud, the RTC ruled that Joses action had prescribed. It cited Article 87
11
of the New Civil Code which requires that the
action for annulment of marriage must be commenced by the injured party within four years after the discovery of the fraud. Thus:
That granting even for the sake of argument that his consent was obtained by [Felisa] through fraud, trickery and machinations, he could have filed an
annulment or declaration of nullity of marriage at the earliest possible opportunity, the time when he discovered the alleged sham and false marriage
contract. [Jose] did not take any action to void the marriage at the earliest instance. x x x.
12

Undeterred, Jose filed an appeal from the foregoing RTC Decision to the Court of Appeals. In a Decision dated 11 August 2005, the Court of Appeals
found the appeal to be without merit. The dispositive portion of the appellate courts Decision reads:
WHEREFORE, the Decision appealed from is AFFIRMED.
13

The Court of Appeals applied the Civil Code to the marriage between Jose and Felisa as it was solemnized prior to the effectivity of the Family Code.
The appellate court observed that the circumstances constituting fraud as a ground for annulment of marriage under Article 86
14
of the Civil Code did not
exist in the marriage between the parties. Further, it ruled that the action for annulment of marriage on the ground of fraud was filed beyond the
prescriptive period provided by law. The Court of Appeals struck down Joses appeal in the following manner:
Nonetheless, even if we consider that fraud or intimidation was employed on Jose in giving his consent to the marriage, the action for the annulment
thereof had already prescribed. Article 87 (4) and (5) of the Civil Code provides that the action for annulment of marriage on the ground that the consent
of a party was obtained by fraud, force or intimidation must be commenced by said party within four (4) years after the discovery of the fraud and within
four (4) years from the time the force or intimidation ceased. Inasmuch as the fraud was allegedly discovered by Jose in February, 1987 then he had
only until February, 1991 within which to file an action for annulment of marriage. However, it was only on July 7, 1993 that Jose filed the complaint for
annulment of his marriage to Felisa.
15

Likewise, the Court of Appeals did not accept Joses assertion that his marriage to Felisa was void ab initio for lack of a marriage license. It ruled that the
marriage was solemnized under Article 76
16
of the Civil Code as one of exceptional character, with the parties executing an affidavit of marriage between
man and woman who have lived together as husband and wife for at least five years. The Court of Appeals concluded that the falsity in the affidavit to
the effect that Jose and Felisa had lived together as husband and wife for the period required by Article 76 did not affect the validity of the marriage,
seeing that the solemnizing officer was misled by the statements contained therein. In this manner, the Court of Appeals gave credence to the good-faith
reliance of the solemnizing officer over the falsity of the affidavit. The appellate court further noted that on the dorsal side of said affidavit of marriage,
Rev. Tomas V. Atienza, the solemnizing officer, stated that he took steps to ascertain the ages and other qualifications of the contracting parties and
found no legal impediment to their marriage. Finally, the Court of Appeals dismissed Joses argument that neither he nor Felisa was a member of the
sect to which Rev. Tomas V. Atienza belonged. According to the Court of Appeals, Article 56
17
of the Civil Code did not require that either one of the
contracting parties to the marriage must belong to the solemnizing officers church or religious sect. The prescription was established only in Article
7
18
of the Family Code which does not govern the parties marriage.
Differing with the ruling of the Court of Appeals, Jose filed a Motion for Reconsideration thereof.1avvphi1 His central opposition was that the requisites
for the proper application of the exemption from a marriage license under Article 76 of the Civil Code were not fully attendant in the case at bar. In
particular, Jose cited the legal condition that the man and the woman must have been living together as husband and wife for at least five years before
the marriage. Essentially, he maintained that the affidavit of marital cohabitation executed by him and Felisa was false.
The Court of Appeals granted Joses Motion for Reconsideration and reversed itself. Accordingly, it rendered an Amended Decision, dated 7 November
2006, the fallo of which reads:
WHEREFORE, the Decision dated August 11, 2005 is RECALLED and SET ASIDE and another one entered declaring the marriage between Jose A.
Dayot and Felisa C. Tecson void ab initio.
Furnish a copy of this Amended Decision to the Local Civil Registrar of Pasay City.
19

In its Amended Decision, the Court of Appeals relied on the ruling of this Court in Nial v. Bayadog,
20
and reasoned that:
In Nial v. Bayadog, where the contracting parties to a marriage solemnized without a marriage license on the basis of their affidavit that they had
attained the age of majority, that being unmarried, they had lived together for at least five (5) years and that they desired to marry each other, the
Supreme Court ruled as follows:
"x x x In other words, the five-year common-law cohabitation period, which is counted back from the date of celebration of marriage, should be a period
of legal union had it not been for the absence of the marriage. This 5-year period should be the years immediately before the day of the marriage and it
should be a period of cohabitation characterized by exclusivity meaning no third party was involved at any time within the 5 years and continuity that
is unbroken. Otherwise, if that continuous 5-year cohabitation is computed without any distinction as to whether the parties were capacitated to marry
each other during the entire five years, then the law would be sanctioning immorality and encouraging parties to have common law relationships and
placing them on the same footing with those who lived faithfully with their spouse. Marriage being a special relationship must be respected as such and
its requirements must be strictly observed. The presumption that a man and a woman deporting themselves as husband and wife is based on the
approximation of the requirements of the law. The parties should not be afforded any excuse to not comply with every single requirement and later use
the same missing element as a pre-conceived escape ground to nullify their marriage. There should be no exemption from securing a marriage license
unless the circumstances clearly fall within the ambit of the exception. It should be noted that a license is required in order to notify the public that two
persons are about to be united in matrimony and that anyone who is aware or has knowledge of any impediment to the union of the two shall make it
known to the local civil registrar.

47
Article 80(3) of the Civil Code provides that a marriage solemnized without a marriage license, save marriages of exceptional character, shall be void
from the beginning. Inasmuch as the marriage between Jose and Felisa is not covered by the exception to the requirement of a marriage license, it is,
therefore, void ab initio because of the absence of a marriage license.
21

Felisa sought reconsideration of the Amended Decision, but to no avail. The appellate court rendered a Resolution
22
dated 10 May 2007, denying
Felisas motion.
Meanwhile, the Republic of the Philippines, through the Office of the Solicitor General (OSG), filed a Petition for Review before this Court in G.R. No.
175581, praying that the Court of Appeals Amended Decision dated 7 November 2006 be reversed and set aside for lack of merit, and that the marriage
between Jose and Felisa be declared valid and subsisting. Felisa filed a separate Petition for Review, docketed as G.R. No. 179474, similarly assailing
the appellate courts Amended Decision. On 1 August 2007, this Court resolved to consolidate the two Petitions in the interest of uniformity of the Court
rulings in similar cases brought before it for resolution.
23

The Republic of the Philippines propounds the following arguments for the allowance of its Petition, to wit:
I
RESPONDENT FAILED TO OVERTHROW THE PRESUMPTION OF THE VALIDITY OF HIS MARRIAGE TO FELISA.
II
RESPONDENT DID NOT COME TO THE COURT WITH CLEAN HANDS AND SHOULD NOT BE ALLOWED TO PROFIT FROM HIS OWN
FRAUDULENT CONDUCT.
III
RESPONDENT IS ESTOPPED FROM ASSAILING THE LEGALITY OF HIS MARRIAGE FOR LACK OF MARRIAGE LICEN[S]E.
24

Correlative to the above, Felisa submits that the Court of Appeals misapplied Nial.
25
She differentiates the case at bar from Nial by reasoning that one
of the parties therein had an existing prior marriage, a circumstance which does not obtain in her cohabitation with Jose. Finally, Felisa adduces that
Jose only sought the annulment of their marriage after a criminal case for bigamy and an administrative case had been filed against him in order to avoid
liability. Felisa surmises that the declaration of nullity of their marriage would exonerate Jose from any liability.
For our resolution is the validity of the marriage between Jose and Felisa. To reach a considered ruling on the issue, we shall jointly tackle the related
arguments vented by petitioners Republic of the Philippines and Felisa.
The Republic of the Philippines asserts that several circumstances give rise to the presumption that a valid marriage exists between Jose and Felisa.
For her part, Felisa echoes the claim that any doubt should be resolved in favor of the validity of the marriage by citing this Courts ruling in Hernandez v.
Court of Appeals.
26
To buttress its assertion, the Republic points to the affidavit executed by Jose and Felisa, dated 24 November 1986, attesting that
they have lived together as husband and wife for at least five years, which they used in lieu of a marriage license. It is the Republics position that the
falsity of the statements in the affidavit does not affect the validity of the marriage, as the essential and formal requisites were complied with; and the
solemnizing officer was not required to investigate as to whether the said affidavit was legally obtained. The Republic opines that as a marriage under a
license is not invalidated by the fact that the license was wrongfully obtained, so must a marriage not be invalidated by the fact that the parties
incorporated a fabricated statement in their affidavit that they cohabited as husband and wife for at least five years. In addition, the Republic posits that
the parties marriage contract states that their marriage was solemnized under Article 76 of the Civil Code. It also bears the signature of the parties and
their witnesses, and must be considered a primary evidence of marriage. To further fortify its Petition, the Republic adduces the following documents: (1)
Joses notarized Statement of Assets and Liabilities, dated 12 May 1988 wherein he wrote Felisas name as his wife; (2) Certification dated 25 July 1993
issued by the Barangay Chairman 192, Zone ZZ, District 24 of Pasay City, attesting that Jose and Felisa had lived together as husband and wife in said
barangay; and (3) Joses company ID card, dated 2 May 1988, indicating Felisas name as his wife.
The first assignment of error compels this Court to rule on the issue of the effect of a false affidavit under Article 76 of the Civil Code. A survey of the
prevailing rules is in order.
It is beyond dispute that the marriage of Jose and Felisa was celebrated on 24 November 1986, prior to the effectivity of the Family Code. Accordingly,
the Civil Code governs their union. Article 53 of the Civil Code spells out the essential requisites of marriage as a contract:
ART. 53. No marriage shall be solemnized unless all these requisites are complied with:
(1) Legal capacity of the contracting parties;
(2) Their consent, freely given;
(3) Authority of the person performing the marriage; and
(4) A marriage license, except in a marriage of exceptional character. (Emphasis ours.)

48
Article 58
27
makes explicit that no marriage shall be solemnized without a license first being issued by the local civil registrar of the municipality where
either contracting party habitually resides, save marriages of an exceptional character authorized by the Civil Code, but not those under Article
75.
28
Article 80(3)
29
of the Civil Code makes it clear that a marriage performed without the corresponding marriage license is void, this being nothing
more than the legitimate consequence flowing from the fact that the license is the essence of the marriage contract.
30
This is in stark contrast to the old
Marriage Law,
31
whereby the absence of a marriage license did not make the marriage void. The rationale for the compulsory character of a marriage
license under the Civil Code is that it is the authority granted by the State to the contracting parties, after the proper government official has inquired into
their capacity to contract marriage.
32

Under the Civil Code, marriages of exceptional character are covered by Chapter 2, Title III, comprising Articles 72 to 79. To wit, these marriages are:
(1) marriages in articulo mortis or at the point of death during peace or war, (2) marriages in remote places, (2) consular marriages,
33
(3) ratification of
marital cohabitation, (4) religious ratification of a civil marriage, (5) Mohammedan or pagan marriages, and (6) mixed marriages.
34

The instant case pertains to a ratification of marital cohabitation under Article 76 of the Civil Code, which provides:
ART. 76. No marriage license shall be necessary when a man and a woman who have attained the age of majority and who, being unmarried, have lived
together as husband and wife for at least five years, desire to marry each other. The contracting parties shall state the foregoing facts in an affidavit
before any person authorized by law to administer oaths. The official, priest or minister who solemnized the marriage shall also state in an affidavit that
he took steps to ascertain the ages and other qualifications of the contracting parties and that he found no legal impediment to the marriage.
The reason for the law,
35
as espoused by the Code Commission, is that the publicity attending a marriage license may discourage such persons who
have lived in a state of cohabitation from legalizing their status.
36

It is not contested herein that the marriage of Jose and Felisa was performed without a marriage license. In lieu thereof, they executed an affidavit
declaring that "they have attained the age of maturity; that being unmarried, they have lived together as husband and wife for at least five years; and that
because of this union, they desire to marry each other."
37
One of the central issues in the Petition at bar is thus: whether the falsity of an affidavit of
marital cohabitation, where the parties have in truth fallen short of the minimum five-year requirement, effectively renders the marriage void ab initio for
lack of a marriage license.
We answer in the affirmative.
Marriages of exceptional character are, doubtless, the exceptions to the rule on the indispensability of the formal requisite of a marriage license. Under
the rules of statutory construction, exceptions, as a general rule, should be strictly
38
but reasonably construed.
39
They extend only so far as their
language fairly warrants, and all doubts should be resolved in favor of the general provisions rather than the exception.
40
Where a general rule is
established by statute with exceptions, the court will not curtail the former or add to the latter by implication.
41
For the exception in Article 76 to apply, it is
a sine qua non thereto that the man and the woman must have attained the age of majority, and that, being unmarried, they have lived together as
husband and wife for at least five years.
A strict but reasonable construction of Article 76 leaves us with no other expediency but to read the law as it is plainly written. The exception of a
marriage license under Article 76 applies only to those who have lived together as husband and wife for at least five years and desire to marry each
other. The Civil Code, in no ambiguous terms, places a minimum period requirement of five years of cohabitation. No other reading of the law can be
had, since the language of Article 76 is precise. The minimum requisite of five years of cohabitation is an indispensability carved in the language of the
law. For a marriage celebrated under Article 76 to be valid, this material fact cannot be dispensed with. It is embodied in the law not as a directory
requirement, but as one that partakes of a mandatory character. It is worthy to mention that Article 76 also prescribes that the contracting parties shall
state the requisite facts
42
in an affidavit before any person authorized by law to administer oaths; and that the official, priest or minister who solemnized
the marriage shall also state in an affidavit that he took steps to ascertain the ages and other qualifications of the contracting parties and that he found
no legal impediment to the marriage.
It is indubitably established that Jose and Felisa have not lived together for five years at the time they executed their sworn affidavit and contracted
marriage. The Republic admitted that Jose and Felisa started living together only in June 1986, or barely five months before the celebration of their
marriage.
43
The Court of Appeals also noted Felisas testimony that Jose was introduced to her by her neighbor, Teresita Perwel, sometime in February
or March 1986 after the EDSA Revolution.
44
The appellate court also cited Felisas own testimony that it was only in June 1986 when Jose commenced
to live in her house.
45

Moreover, it is noteworthy that the question as to whether they satisfied the minimum five-year requisite is factual in nature. A question of fact arises
when there is a need to decide on the truth or falsehood of the alleged facts.
46
Under Rule 45, factual findings are ordinarily not subject to this Courts
review.
47
It is already well-settled that:
The general rule is that the findings of facts of the Court of Appeals are binding on this Court. A recognized exception to this rule is when the Court of
Appeals and the trial court, or in this case the administrative body, make contradictory findings. However, the exception does not apply in every instance
that the Court of Appeals and the trial court or administrative body disagree. The factual findings of the Court of Appeals remain conclusive on this Court
if such findings are supported by the record or based on substantial evidence.
48

Therefore, the falsity of the affidavit dated 24 November 1986, executed by Jose and Felisa to exempt them from the requirement of a marriage license,
is beyond question.
We cannot accept the insistence of the Republic that the falsity of the statements in the parties affidavit will not affect the validity of marriage, since all
the essential and formal requisites were complied with. The argument deserves scant merit. Patently, it cannot be denied that the marriage between
Jose and Felisa was celebrated without the formal requisite of a marriage license. Neither did Jose and Felisa meet the explicit legal requirement in
Article 76, that they should have lived together as husband and wife for at least five years, so as to be excepted from the requirement of a marriage
license.

49
Anent petitioners reliance on the presumption of marriage, this Court holds that the same finds no applicability to the case at bar. Essentially, when we
speak of a presumption of marriage, it is with reference to the prima facie presumption that a man and a woman deporting themselves as husband and
wife have entered into a lawful contract of marriage.
49
Restated more explicitly, persons dwelling together in apparent matrimony are presumed, in the
absence of any counter-presumption or evidence special to the case, to be in fact married.
50
The present case does not involve an apparent marriage to
which the presumption still needs to be applied. There is no question that Jose and Felisa actually entered into a contract of marriage on 24 November
1986, hence, compelling Jose to institute a Complaint for Annulment and/or Declaration of Nullity of Marriage, which spawned the instant consolidated
Petitions.
In the same vein, the declaration of the Civil Code
51
that every intendment of law or fact leans towards the validity of marriage will not salvage the
parties marriage, and extricate them from the effect of a violation of the law. The marriage of Jose and Felisa was entered into without the requisite
marriage license or compliance with the stringent requirements of a marriage under exceptional circumstance. The solemnization of a marriage without
prior license is a clear violation of the law and would lead or could be used, at least, for the perpetration of fraud against innocent and unwary parties,
which was one of the evils that the law sought to prevent by making a prior license a prerequisite for a valid marriage.
52
The protection of marriage as a
sacred institution requires not just the defense of a true and genuine union but the exposure of an invalid one as well.
53
To permit a false affidavit to take
the place of a marriage license is to allow an abject circumvention of the law. If this Court is to protect the fabric of the institution of marriage, we must be
wary of deceptive schemes that violate the legal measures set forth in our laws.
Similarly, we are not impressed by the ratiocination of the Republic that as a marriage under a license is not invalidated by the fact that the license was
wrongfully obtained, so must a marriage not be invalidated by a fabricated statement that the parties have cohabited for at least five years as required by
law. The contrast is flagrant. The former is with reference to an irregularity of the marriage license, and not to the absence of one. Here, there is no
marriage license at all. Furthermore, the falsity of the allegation in the sworn affidavit relating to the period of Jose and Felisas cohabitation, which would
have qualified their marriage as an exception to the requirement for a marriage license, cannot be a mere irregularity, for it refers to a quintessential fact
that the law precisely required to be deposed and attested to by the parties under oath. If the essential matter in the sworn affidavit is a lie, then it is but
a mere scrap of paper, without force and effect. Hence, it is as if there was no affidavit at all.
In its second assignment of error, the Republic puts forth the argument that based on equity, Jose should be denied relief because he perpetrated the
fabrication, and cannot thereby profit from his wrongdoing. This is a misplaced invocation. It must be stated that equity finds no room for application
where there is a law.
54
There is a law on the ratification of marital cohabitation, which is set in precise terms under Article 76 of the Civil Code.
Nonetheless, the authorities are consistent that the declaration of nullity of the parties marriage is without prejudice to their criminal liability.
55

The Republic further avers in its third assignment of error that Jose is deemed estopped from assailing the legality of his marriage for lack of a marriage
license. It is claimed that Jose and Felisa had lived together from 1986 to 1990, notwithstanding Joses subsequent marriage to Rufina Pascual on 31
August 1990, and that it took Jose seven years before he sought the declaration of nullity; hence, estoppel had set in.
This is erroneous. An action for nullity of marriage is imprescriptible.
56
Jose and Felisas marriage was celebrated sans a marriage license. No other
conclusion can be reached except that it is void ab initio. In this case, the right to impugn a void marriage does not prescribe, and may be raised any
time.
Lastly, to settle all doubts, jurisprudence has laid down the rule that the five-year common-law cohabitation period under Article 76 means a five-year
period computed back from the date of celebration of marriage, and refers to a period of legal union had it not been for the absence of a marriage.
57
It
covers the years immediately preceding the day of the marriage, characterized by exclusivity - meaning no third party was involved at any time within the
five years - and continuity that is unbroken.
58

WHEREFORE, the Petitions are DENIED. The Amended Decision of the Court of Appeals, dated 7 November 2006 in CA-G.R. CV No. 68759, declaring
the marriage of Jose Dayot to Felisa Tecson-Dayot void ab initio, is AFFIRMED, without prejudice to their criminal liability, if any. No costs.
SO ORDERED.
MINITA V. CHICO-NAZARIO
Associate Justice
WE CONCUR:
MA. ALICIA AUSTRIA-MARTINEZ
Associate Justice
Acting Chairperson
DANTE O. TINGA
*

Associate Justice
PRESBITERO J. VELASCO, JR.
**

Associate Justice
RUBEN T. REYES
Associate Justice
A T T E S T A T I O N
I attest that the conclusions in the above Decision were reached in consultation before the case was assigned to the writer of the opinion of the Courts
Division.

50
MA. ALICIA AUSTRIA-MARTINEZ
Associate Justice
Acting Chairperson, Third Division
C E R T I F I C A T I O N
Pursuant to Section 13, Article VIII of the Constitution, and the Division Acting Chairpersons attestation, it is hereby certified that the conclusions in the
above Decision were reached in consultation before the case was assigned to the writer of the opinion of the Courts Division.
REYNATO S. PUNO
Chief Justice



REINEL ANTHONY B. DE CASTRO, G.R. No. 160172
Petitioner,
Present:

QUISUMBING, J .,
Chairperson,
- versus - CARPIO,
CARPIO MORALES,
TINGA, and
VELASCO, JR., J J .
ANNABELLE ASSIDAO-DE CASTRO,
Respondent.
Promulgated:

February 13, 2008

x---------------------------------------------------------------------------x


D E C I S I O N

TINGA, J.:

This is a petition for review of the Decision
[1]
of the Court of Appeals in CA-GR CV. No. 69166,
[2]
declaring that (1) Reianna Tricia A. De Castro is
the legitimate child of the petitioner; and (2) that the marriage between petitioner and respondent is valid until properly nullified by a competent court in a
proceeding instituted for that purpose.

The facts of the case, as culled from the records, follow.

Petitioner and respondent met and became sweethearts in 1991. They planned to get married, thus they applied for a marriage license with the
Office of the Civil Registrar of Pasig City in September 1994. They had their first sexual relation sometime in October 1994, and had regularly engaged
in sex thereafter. When the couple went back to the Office of the Civil Registrar, the marriage license had already expired. Thus, in order to push
through with the plan, in lieu of a marriage license, they executed an affidavit dated 13 March 1995 stating that they had been living together as husband
and wife for at least five years. The couple got married on the same date, with Judge Jose C. Bernabe, presiding judge of
the Metropolitan Trial Court of Pasig City, administering the civil rites. Nevertheless, after the ceremony, petitioner and respondent went back to their
respective homes and did not live together as husband and wife.

On 13 November 1995, respondent gave birth to a child named Reinna Tricia A. De Castro. Since the childs birth, respondent has been the one
supporting her out of her income as a government dentist and from her private practice.


51
On 4 June 1998, respondent filed a complaint for support against petitioner before the Regional Trial Court of Pasig City (trial court.
[3]
In her
complaint, respondent alleged that she is married to petitioner and that the latter has reneged on his responsibility/obligation to financially support her
as his wife and Reinna Tricia as his child.
[4]


Petitioner denied that he is married to respondent, claiming that their marriage is void ab initio since the marriage was facilitated by a fake
affidavit; and that he was merely prevailed upon by respondent to sign the marriage contract to save her from embarrassment and possible
administrative prosecution due to her pregnant state; and that he was not able to get parental advice from his parents before he got married. He also
averred that they never lived together as husband and wife and that he has never seen nor acknowledged the child.

In its Decision dated 16 October 2000,
[5]
the trial court ruled that the marriage between petitioner and respondent is not valid because it was
solemnized without a marriage license. However, it declared petitioner as the natural father of the child, and thus obliged to give her support. Petitioner
elevated the case to the Court of Appeals, arguing that the lower court committed grave abuse of discretion when, on the basis of mere belief and
conjecture, it ordered him to provide support to the child when the latter is not, and could not have been, his own child.

The Court of Appeals denied the appeal. Prompted by the rule that a marriage is presumed to be subsisting until a judicial declaration of
nullity has been made, the appellate court declared that the child was born during the subsistence and validity of the parties marriage. In addition, the
Court of Appeals frowned upon petitioners refusal to undergo DNA testing to prove the paternity and filiation, as well as his refusal to state with certainty
the last time he had carnal knowledge with respondent, saying that petitioners forgetfulness should not be used as a vehicle to relieve him of his
obligation and reward him of his being irresponsible.
[6]
Moreover, the Court of Appeals noted the affidavit dated 7 April 1998 executed by petitioner,
wherein he voluntarily admitted that he is the legitimate father of the child.

The appellate court also ruled that since this case is an action for support, it was improper for the trial court to declare the marriage of
petitioner and respondent as null and void in the very same case. There was no participation of the State, through the prosecuting attorney or fiscal, to
see to it that there is no collusion between the parties, as required by the Family Code in actions for declaration of nullity of a marriage. The burden of
proof to show that the marriage is void rests upon petitioner, but it is a matter that can be raised in an action for declaration of nullity, and not in the
instant proceedings. The proceedings before the trial court should have been limited to the obligation of petitioner to support the child and his wife on
the basis of the marriage apparently and voluntarily entered into by petitioner and respondent.
[7]
The dispositive portion of the decision reads:

WHEREFORE, premises considered, the Decision dated 16 October 2000, of the Regional Trial Court of Pasig City,
National Capital Judicial Region, Brach 70, in JDRC No. 4626, isAFFIRMED with the MODIFICATIONS (1) declaring Reianna
Tricia A. De Castro, as the legitimate child of the appellant and the appellee and (2) declaring the marriage on 13 March 1995
between the appellant and the appellee valid until properly annulled by a competent court in a proceeding instituted for that
purpose. Costs against the appellant.
[8]



Petitioner filed a motion for reconsideration, but the motion was denied by the Court of Appeals.
[9]
Hence this petition.

Before us, petitioner contends that the trial court properly annulled his marriage with respondent because as shown by the evidence and
admissions of the parties, the marriage was celebrated without a marriage license. He stresses that the affidavit they executed, in lieu of a marriage
license, contained a false narration of facts, the truth being that he and respondent never lived together as husband and wife. The false affidavit should
never be allowed or admitted as a substitute to fill the absence of a marriage license.
[10]
Petitioner additionally argues that there was no need for the
appearance of a prosecuting attorney in this case because it is only an ordinary action for support and not an action for annulment or declaration of

52
absolute nullity of marriage. In any case, petitioner argues that the trial court had jurisdiction to determine the invalidity of their marriage since it was
validly invoked as an affirmative defense in the instant action for support. Citing several authorities,
[11]
petitioner claims that a void marriage can be the
subject of a collateral attack. Thus, there is no necessity to institute another independent proceeding for the declaration of nullity of the marriage
between the parties. The refiling of another case for declaration of nullity where the same evidence and parties would be presented would entail
enormous expenses and anxieties, would be time-consuming for the parties, and would increase the burden of the courts.
[12]
Finally, petitioner claims
that in view of the nullity of his marriage with respondent and his vigorous denial of the childs paternity and filiation, the Court of Appeals gravely erred in
declaring the child as his legitimate child.

In a resolution dated 16 February 2004, the Court required respondent and the Office of the Solicitor General (OSG) to file their respective
comments on the petition.
[13]


In her Comment,
[14]
respondent claims that the instant petition is a mere dilatory tactic to thwart the finality of the decision of the Court of
Appeals. Echoing the findings and rulings of the appellate court, she argues that the legitimacy of their marriage cannot be attacked collaterally, but can
only be repudiated or contested in a direct suit specifically brought for that purpose. With regard to the filiation of her child, she pointed out that
compared to her candid and straightforward testimony, petitioner was uncertain, if not evasive in answering questions about their sexual
encounters. Moreover, she adds that despite the challenge from her and from the trial court, petitioner strongly objected to being subjected to DNA
testing to prove paternity and filiation.
[15]


For its part, the OSG avers that the Court of Appeals erred in holding that it was improper for the trial court to declare null and void the marriage of
petitioner and respondent in the action for support. Citing the case of Nial v. Bayadog,
[16]
it states that courts may pass upon the validity of a marriage
in an action for support, since the right to support from petitioner hinges on the existence of a valid marriage. Moreover, the evidence presented during
the proceedings in the trial court showed that the marriage between petitioner and respondent was solemnized without a marriage license, and that their
affidavit (of a man and woman who have lived together and exclusively with each other as husband and wife for at least five years) was false. Thus, it
concludes the trial court correctly held that the marriage between petitioner and respondent is not valid.
[17]
In addition, the OSG agrees
with the findings of the trial court that the child is an illegitimate child of petitioner and thus entitled to support.
[18]


Two key issues are presented before us. First, whether the trial court had the jurisdiction to determine the validity of the marriage between
petitioner and respondent in an action for support and second, whether the child is the daughter of petitioner.

Anent the first issue, the Court holds that the trial court had jurisdiction to determine the validity of the marriage between petitioner and
respondent. The validity of a void marriage may be collaterally attacked.
[19]
Thus, in Nial v. Bayadog, we held:

However, other than for purposes of remarriage, no judicial action is necessary to declare a marriage an absolute
nullity. For other purposes, such as but not limited to determination of heirship, legitimacy or illegitimacy of a child, settlement of
estate, dissolution of property regime, or a criminal case for that matter, the court may pass upon the validity of marriage even in a suit
not directly instituted to question the same so long as it is essential to the determination of the case. This is without prejudice to any
issue that may arise in the case. When such need arises, a final judgment of declaration of nullity is necessary even if the purpose is
other than to remarry. The clause on the basis of a final judgment declaring such previous marriage void in Article 40 of the Family
Code connotes that such final judgment need not be obtained only for purpose of remarriage.
[20]




53
Likewise, in Nicdao Cario v. Yee Cario,
[21]
the Court ruled that it is clothed with sufficient authority to pass upon the validity of two marriages
despite the main case being a claim for death benefits. Reiterating Nial, we held that the Court may pass upon the validity of a marriage even in a suit
not directly instituted to question the validity of said marriage, so long as it is essential to the determination of the case. However, evidence must be
adduced, testimonial or documentary, to prove the existence of grounds rendering such a marriage an absolute nullity.
[22]


Under the Family Code, the absence of any of the essential or formal requisites shall render the marriage void ab initio, whereas a defect in any of
the essential requisites shall render the marriage voidable.
[23]
In the instant case, it is clear from the evidence presented that petitioner and respondent
did not have a marriage license when they contracted their marriage. Instead, they presented an affidavit stating that they had been living together for
more than five years.
[24]
However, respondent herself in effect admitted the falsity of the affidavit when she was asked during cross-examination, thus

ATTY. CARPIO:

Q But despite of (sic) the fact that you have not been living together as husband and wife for the last five years on or before
March 13, 1995, you signed the Affidavit, is that correct?
A Yes, sir.
[25]



The falsity of the affidavit cannot be considered as a mere irregularity in the formal requisites of marriage. The law dispenses with the marriage license
requirement for a man and a woman who have lived together and exclusively with each other as husband and wife for a continuous and unbroken period
of at least five years before the marriage. The aim of this provision is to avoid exposing the parties to humiliation, shame and embarrassment
concomitant with the scandalous cohabitation of persons outside a valid marriage due to the publication of every applicants name for a marriage
license.
[26]
In the instant case, there was no scandalous cohabitation to protect; in fact, there was no cohabitation at all. The false affidavit which
petitioner and respondent executed so they could push through with the marriage has no value whatsoever; it is a mere scrap of paper. They were not
exempt from the marriage license requirement. Their failure to obtain and present a marriage license renders their marriage void ab initio.

Anent the second issue, we find that the child is petitioners illegitimate daughter, and therefore entitled to support.

Illegitimate children may establish their illegitimate filiation in the same way and on the same evidence as legitimate children.
[27]
Thus, one can
prove illegitimate filiation through the record of birth appearing in the civil register or a final judgment, an admission of legitimate filiation in a public
document or a private handwritten instrument and signed by the parent concerned, or the open and continuous possession of the status of a legitimate
child, or any other means allowed by the Rules of Court and special laws.
[28]


The Certificate of Live Birth
[29]
of the child lists petitioner as the father. In addition, petitioner, in an affidavit waiving additional tax exemption in
favor of respondent, admitted that he is the father of the child, thus stating:

1. I am the legitimate father of REIANNA TRICIA A. DE CASTRO who was born on November 3, 1995 at Better Living,
Paraaque, Metro Manila;
[30]



We are likewise inclined to agree with the following findings of the trial court:

That Reinna Tricia is the child of the respondent with the petitioner is supported not only by the testimony of the latter, but
also by respondents own admission in the course of his testimony wherein he conceded that petitioner was his former

54
girlfriend. While they were sweethearts, he used to visit petitioner at the latters house or clinic. At times, they would go to a motel to
have sex. As a result of their sexual dalliances, petitioner became pregnant which ultimately led to their marriage, though invalid, as
earlier ruled. While respondent claims that he was merely forced to undergo the marriage ceremony, the pictures taken of the
occasion reveal otherwise (Exhs. B, B-1, to B-3, C, C-1 and C-2, D, D-1 and D-2, E, E-1 and E-2, F, F-1 and F-
2, G, G-1 and G-2 and H, H-1 to H-3). In one of the pictures (Exhs. D, D-1 and D-2), defendant is seen putting the
wedding ring on petitioners finger and in another picture (Exhs. E, E-1 and E-2) respondent is seen in the act of kissing the
petitioner.
[31]



WHEREFORE, the petition is granted in part. The assailed Decision and Resolution of the Court of Appeals in CA-GR CV No. 69166 are
SET ASIDE and the decision of the Regional Trial Court Branch 70 of Pasig City in JDRC No. 4626 dated 16 October 2000 is hereby
REINSTATED.

SO ORDERED.



Republic of the Philippines
SUPREME COURT
Manila
THIRD DIVISION
G.R. No. 183896 January 30, 2013
SYED AZHAR ABBAS, Petitioner,
vs.
GLORIA GOO ABBAS, Respondent.
D E C I S I O N
VELASCO, JR., J .:
This is a Petition for Review on Certiorari under Rule 45 of the 1997 Rules of Civil Procedure, questioning the Decision
1
of the Court of Appeals (CA) dated March 11,
2008 in CA-G.R. CV No. 86760, which reversed the Decision
2
in Civil Case No. 03-0382-CFM dated October 5, 2005 of the Regional Trial Court (RTC), Branch 109,
Pasay City, and the CA Resolution dated July 24, 2008, denying petitioner's Motion for Reconsideration of the CA Decision.
The present case stems from a petition filed by petitioner Syed Azhar Abbas (Syed) for the declaration of nullity of his marriage to Gloria Goo-Abbas (Gloria) with the
RTC of Pasay City, docketed as Civil Case No. 03-0382-CFM, and raffled to RTC Branch 109. Syed alleged the absence of a marriage license, as provided for in Article
4, Chapter I, Title 1 of Executive Order No. 269, otherwise known as the Family Code of the Philippines, as a ground for the annulment of his marriage to Gloria.
In the Marriage Contract
3
of Gloria and Syed, it is stated that Marriage License No. 9969967, issued at Carmona, Cavite on January 8, 1993, was presented to the
solemnizing officer. It is this information that is crucial to the resolution of this case.
At the trial court, Syed, a Pakistani citizen, testified that he met Gloria, a Filipino citizen, in Taiwan in 1991, and they were married on August 9, 1992 at the Taipei
Mosque in Taiwan.
4
He arrived in the Philippines in December of 1992. On January 9, 1993, at around 5 oclock in the afternoon, he was at his mother-in-laws residence,
located at 2676 F. Muoz St., Malate, Manila, when his mother-in-law arrived with two men. He testified that he was told that he was going to undergo some ceremony,
one of the requirements for his stay in the Philippines, but was not told of the nature of said ceremony. During the ceremony he and Gloria signed a document. He
claimed that he did not know that the ceremony was a marriage until Gloria told him later. He further testified that he did not go to Carmona, Cavite to apply for a
marriage license, and that he had never resided in that area. In July of 2003, he went to the Office of the Civil Registrar of Carmona, Cavite, to check on their marriage
license, and was asked to show a copy of their marriage contract wherein the marriage license number could be found.
5
The Municipal Civil Registrar, Leodivinia C.
Encarnacion, issued a certification on July 11, 2003 to the effect that the marriage license number appearing in the marriage contract he submitted, Marriage License No.
9969967, was the number of another marriage license issued to a certain Arlindo Getalado and Myra Mabilangan.
6
Said certification reads as follows:
11 July 2003
TO WHOM IT MAY CONCERN:
This is to certify as per Registry Records of Marriage License filed in this office, Marriage License No. 9969967 was issued in favor of MR. ARLINDO GETALADO and
MISS MYRA MABILANGAN on January 19, 1993.
No Marriage License appear [sic] to have been issued to MR. SYED AZHAR ABBAS and MISS GLORIA F. GOO on January 8, 1993.

55
This certification is being issued to Mr. Syed Azhar Abbas for whatever legal purpose or intents it may serve.
7

On cross-examination, Syed testified that Gloria had filed bigamy cases against him in 2001 and 2002, and that he had gone to the Municipal Civil Registrar of Carmona,
Cavite to get certification on whether or not there was a marriage license on advice of his counsel.
8

Petitioner also presented Norberto Bagsic (Bagsic), an employee of the Municipal Civil Registrar of Carmona, Cavite. Bagsic appeared under a letter of authority from the
Municipal Civil Registrar of Carmona, Cavite, and brought documents pertaining to Marriage License No. 9969967, which was issued to Arlindo Getalado and Myra
Mabilangan on January 20, 1993.
9

Bagsic testified that their office issues serial numbers for marriage licenses and that the numbers are issued chronologically.
10
He testified that the certification dated July
11, 2003, was issued and signed by Leodivina Encarnacion, Registrar of the Municipality of Carmona, Cavite, certifying that Marriage License No. 9969967 was issued
for Arlindo Getalado and Myra Mabilangan on January 19, 1993, and that their office had not issued any other license of the same serial number, namely 9969967, to any
other person.
11

For her part, Gloria testified on her own behalf, and presented Reverend Mario Dauz, Atty. Lorenzo Sanchez, Felicitas Goo and May Ann Ceriola.
Reverend Mario Dauz (Rev. Dauz) testified that he was a minister of the Gospel and a barangay captain, and that he is authorized to solemnize marriages within the
Philippines.
12
He testified that he solemnized the marriage of Syed Azhar Abbas and Gloria Goo at the residence of the bride on January 9, 1993.
13
He stated that the
witnesses were Atty. Lorenzo Sanchez (Atty. Sanchez) and Mary Ann Ceriola.
14
He testified that he had been solemnizing marriages since 1982, and that he is familiar
with the requirements.
15
Rev. Dauz further testified that Atty. Sanchez gave him the marriage license the day before the actual wedding, and that the marriage contract
was prepared by his secretary.
16
After the solemnization of the marriage, it was registered with the Local Civil Registrar of Manila, and Rev. Dauz submitted the marriage
contract and copy of the marriage license with that office.
17

Atty. Sanchez testified that he was asked to be the sponsor of the wedding of Syed Abbas and Gloria Goo by the mother of the bride, Felicitas Goo.
18
He testified that he
requested a certain Qualin to secure the marriage license for the couple, and that this Qualin secured the license and gave the same to him on January 8, 1993.
19
He
further testified that he did not know where the marriage license was obtained.
20
He attended the wedding ceremony on January 9, 1993, signed the marriage contract as
sponsor, and witnessed the signing of the marriage contract by the couple, the solemnizing officer and the other witness, Mary Ann Ceriola.
21

Felicitas Goo testified that Gloria Goo is her daughter and Syed Azhar Abbas is her son-in-law, and that she was present at the wedding ceremony held on January 9,
1993 at her house.
22
She testified that she sought the help of Atty. Sanchez at the Manila City Hall in securing the marriage license, and that a week before the marriage
was to take place, a male person went to their house with the application for marriage license.
23
Three days later, the same person went back to their house, showed her
the marriage license before returning it to Atty. Sanchez who then gave it to Rev. Dauz, the solemnizing officer.
24
She further testified that she did not read all of the
contents of the marriage license, and that she was told that the marriage license was obtained from Carmona.
25
She also testified that a bigamy case had been filed by
Gloria against Syed at the Regional Trial Court of Manila, evidenced by an information for Bigamy dated January 10, 2003, pending before Branch 47 of the Regional
Trial Court of Manila.
26

As to Mary Ann Ceriolas testimony, the counsels for both parties stipulated that: (a) she is one of the sponsors at the wedding of Gloria Goo and Syed Abbas on January
9, 1993; (b) she was seen in the wedding photos and she could identify all the persons depicted in said photos; and (c) her testimony corroborates that of Felicitas Goo
and Atty. Sanchez.
The respondent, Gloria, testified that Syed is her husband, and presented the marriage contract bearing their signatures as proof.
27
She and her mother sought the help
of Atty. Sanchez in securing a marriage license, and asked him to be one of the sponsors. A certain Qualin went to their house and said that he will get the marriage
license for them, and after several days returned with an application for marriage license for them to sign, which she and Syed did. After Qualin returned with the
marriage license, they gave the license to Atty. Sanchez who gave it to Rev. Dauz, the solemnizing officer. Gloria testified that she and Syed were married on January 9,
1993 at their residence.
28

Gloria further testified that she has a daughter with Syed, born on June 15, 1993.
29

Gloria also testified that she filed a bigamy case against Syed, who had married a certain Maria Corazon Buenaventura during the existence of the previous marriage,
and that the case was docketed as Criminal Case No. 02A-03408, with the RTC of Manila.
30

Gloria stated that she and Syed had already been married on August 9, 1992 in Taiwan, but that she did not know if said marriage had been celebrated under Muslim
rites, because the one who celebrated their marriage was Chinese, and those around them at the time were Chinese.
31

The Ruling of the RTC
In its October 5, 2005 Decision, the Pasay City RTC held that no valid marriage license was issued by the Municipal Civil Registrar of Carmona, Cavite in favor of Gloria
and Syed, as Marriage License No. 9969967 had been issued to Arlindo Getalado and Myra Mabilangan, and the Municipal Civil Registrar of Carmona, Cavite had
certified that no marriage license had been issued for Gloria and Syed.
32
It also took into account the fact that neither party was a resident of Carmona, Cavite, the place
where Marriage License No. 9969967 was issued, in violation of Article 9 of the Family Code.
33
As the marriage was not one of those exempt from the license
requirement, and that the lack of a valid marriage license is an absence of a formal requisite, the marriage of Gloria and Syed on January 9, 1993 was void ab initio.
The dispositive portion of the Decision reads as follows:
WHEREFORE, judgment is hereby rendered in favor of the petitioner, and against the respondent declaring as follows:
1. The marriage on January 9, 1993 between petitioner Syed Azhar Abbas and respondent Gloria Goo-Abbas is hereby annulled;

56
2. Terminating the community of property relations between the petitioner and the respondent even if no property was acquired during their
cohabitation by reason of the nullity of the marriage of the parties.
3. The Local Civil Registrar of Manila and the Civil Registrar General, National Statistics Office, are hereby ordered to cancel from their respective
civil registries the marriage contracted by petitioner Syed Azhar Abbas and respondent Gloria Goo-Abbas on January 9, 1993 in Manila.
SO ORDERED.
34

Gloria filed a Motion for Reconsideration dated November 7, 2005, but the RTC denied the same, prompting her to appeal the questioned decision to the Court of
Appeals.
The Ruling of the CA
In her appeal to the CA, Gloria submitted the following assignment of errors:
I
THE LOWER COURT ERRED IN DECLARING THE MARRIAGE BETWEEN THE PETITIONER AND RESPONDENT AS NULL AND VOID DUE TO
THE ABSENCE OF A MARRIAGE LICENSE DESPITE EVIDENCE CLEARLY SHOWING THAT THERE WAS ONE.
II
THE LOWER COURT ERRED IN NOT CONSIDERING, AS A REQUISITE OF A VALID MARRIAGE, THE OVERWHELMING EVIDENCE
SHOWING THAT A MARRIAGE CEREMONY TOOK PLACE WITH THE APPEARANCE OF THE CONTRACTING PARTIES BEFORE THE
SOLEMNIZING OFFICER AND THEIR PERSONAL DECLARATION THAT THEY TOOK EACH OTHER AS HUSBAND AND WIFE IN THE
PRESENCE OF NOT LESS THAN TWO WITNESSES OF LEGAL AGE.
III
THE LOWER COURT ERRED IN NOT RULING ON THE ISSUE OF ESTOPPEL BY LACHES ON THE PART OF THE PETITIONER, AN ISSUE
TIMELY RAISED IN THE COURT BELOW.
35

The CA gave credence to Glorias arguments, and granted her appeal. It held that the certification of the Municipal Civil Registrar failed to categorically state that a
diligent search for the marriage license of Gloria and Syed was conducted, and thus held that said certification could not be accorded probative value.
36
The CA ruled that
there was sufficient testimonial and documentary evidence that Gloria and Syed had been validly married and that there was compliance with all the requisites laid down
by law.
37

It gave weight to the fact that Syed had admitted to having signed the marriage contract. The CA also considered that the parties had comported themselves as husband
and wife, and that Syed only instituted his petition after Gloria had filed a case against him for bigamy.
38

The dispositive portion of the CA Decision reads as follows:
WHEREFORE, premises considered, the appeal is GRANTED. The Decision dated 05 October 2005 and Order dated 27 January 2006 of the Regional Trial Court of
Pasay City, Branch 109, in Civil Case No. 03-0382-CFM are REVERSED and SET ASIDE and the Petition for Declaration of Nullity of Marriage is DISMISSED. The
marriage between Shed [sic] Azhar Abbas and Gloria Goo Abbas contracted on 09 January 1993 remains valid and subsisting. No costs.
SO ORDERED.
39

Syed then filed a Motion for Reconsideration dated April 1, 2008
40
but the same was denied by the CA in a Resolution dated July 24, 2008.
41

Hence, this petition.
Grounds in Support of Petition
I
THE HONORABLE COURT OF APPEALS COMMITTED SERIOUS ERROR OF LAW IN CITING REPUBLIC VS. COURT OF APPEALS AS THE
SAME IS DIAMETRICALLY INCONSISTENT AND CONTRARY TO THE COURTS OWN FINDINGS AND CONCLUSIONS IN THIS CASE.
II
THE HONORABLE COURT OF APPEALS GRAVELY ERRED IN REVERSING AND SETTING ASIDE, WITHOUT ANY FACTUAL AND LEGAL
BASIS, THE DECISION OF THE REGIONAL TRIAL COURT GRANTING THE PETITION FOR DECLARATION OF NULLITY OF MARRIAGE.
42


57
The Ruling of this Court
The petition is meritorious.
As the marriage of Gloria and Syed was solemnized on January 9, 1993, Executive Order No. 209, or the Family Code of the Philippines, is the applicable law. The
pertinent provisions that would apply to this particular case are Articles 3, 4 and 35(3), which read as follows:
Art. 3. The formal requisites of marriage are:
(1) Authority of the solemnizing officer;
(2) A valid marriage license except in the cases provided for in Chapter 2 of this Title; and
(3) A marriage ceremony which takes place with the appearance of the contracting parties before the solemnizing officer and their personal
declaration that they take each other as husband and wife in the presence of not less than two witnesses of legal age.
Art. 4. The absence of any of the essential or formal requisites shall render the marriage void ab initio, except as stated in Article 35(2).
A defect in any of the essential requisites shall render the marriage voidable as provided in Article 45.
An irregularity in the formal requisites shall not affect the validity of the marriage but the party or parties responsible for the irregularity shall be civilly, criminally and
administratively liable.
Art. 35. The following marriages shall be void from the beginning:
x x x x
(3) Those solemnized without a license, except those covered by the preceding Chapter.
There is no issue with the essential requisites under Art. 2 of the Family Code, nor with the formal requisites of the authority of the solemnizing officer and the conduct of
the marriage ceremony. Nor is the marriage one that is exempt from the requirement of a valid marriage license under Chapter 2, Title I of the Family Code. The
resolution of this case, thus, hinges on whether or not a valid marriage license had been issued for the couple. The RTC held that no valid marriage license had been
issued. The CA held that there was a valid marriage license.
We find the RTC to be correct in this instance.
Respondent Gloria failed to present the actual marriage license, or a copy thereof, and relied on the marriage contract as well as the testimonies of her witnesses to
prove the existence of said license. To prove that no such license was issued, Syed turned to the office of the Municipal Civil Registrar of Carmona, Cavite which had
allegedly issued said license. It was there that he requested certification that no such license was issued. In the case of Republic v. Court of Appeals
43
such certification
was allowed, as permitted by Sec. 29, Rule 132 of the Rules of Court, which reads:
SEC. 28. Proof of lack of record. A written statement signed by an officer having the custody of an official record or by his deputy that after diligent search, no record or
entry of a specified tenor is found to exist in the records of his office, accompanied by a certificate as above provided, is admissible as evidence that the records of his
office contain no such record or entry.
In the case of Republic, in allowing the certification of the Civil Registrar of Pasig to prove the non-issuance of a marriage license, the Court held:
The above Rule authorized the custodian of the documents to certify that despite diligent search, a particular document does not exist in his office or that a particular
entry of a specified tenor was not to be found in a register. As custodians of public documents, civil registrars are public officers charged with the duty, inter alia, of
maintaining a register book where they are required to enter all applications for marriage licenses, including the names of the applicants, the date the marriage license
was issued and such other relevant data.
44

The Court held in that case that the certification issued by the civil registrar enjoyed probative value, as his duty was to maintain records of data relative to the issuance of
a marriage license.
The Municipal Civil Registrar of Carmona, Cavite, where the marriage license of Gloria and Syed was allegedly issued, issued a certification to the effect that no such
marriage license for Gloria and Syed was issued, and that the serial number of the marriage license pertained to another couple, Arlindo Getalado and Myra Mabilangan.
A certified machine copy of Marriage License No. 9969967 was presented, which was issued in Carmona, Cavite, and indeed, the names of Gloria and Syed do not
appear in the document.
In reversing the RTC, the CA focused on the wording of the certification, stating that it did not comply with Section 28, Rule 132 of the Rules of Court.
The CA deduced that from the absence of the words "despite diligent search" in the certification, and since the certification used stated that no marriage license appears
to have been issued, no diligent search had been conducted and thus the certification could not be given probative value.
To justify that deduction, the CA cited the case of Republic v. Court of Appeals.
45
It is worth noting that in that particular case, the Court, in sustaining the finding of the

58
lower court that a marriage license was lacking, relied on the Certification issued by the Civil Registrar of Pasig, which merely stated that the alleged marriage license
could not be located as the same did not appear in their records. Nowhere in the Certification was it categorically stated that the officer involved conducted a diligent
search, nor is a categorical declaration absolutely necessary for Sec. 28, Rule 132 of the Rules of Court to apply.
Under Sec. 3(m), Rule 131 of the Rules of Court, it is a disputable presumption that an official duty has been regularly performed, absent contradiction or other evidence
to the contrary. We held, "The presumption of regularity of official acts may be rebutted by affirmative evidence of irregularity or failure to perform a duty."
46
No such
affirmative evidence was shown that the Municipal Civil Registrar was lax in performing her duty of checking the records of their office, thus the presumption must stand.
In fact, proof does exist of a diligent search having been conducted, as Marriage License No. 996967 was indeed located and submitted to the court. The fact that the
names in said license do not correspond to those of Gloria and Syed does not overturn the presumption that the registrar conducted a diligent search of the records of
her office.
It is telling that Gloria failed to present their marriage license or a copy thereof to the court. She failed to explain why the marriage license was secured in Carmona,
Cavite, a location where, admittedly, neither party resided. She took no pains to apply for the license, so she is not the best witness to testify to the validity and existence
of said license. Neither could the other witnesses she presented prove the existence of the marriage license, as none of them applied for the license in Carmona, Cavite.
Her mother, Felicitas Goo, could not even testify as to the contents of the license, having admitted to not reading all of its contents. Atty. Sanchez, one of the sponsors,
whom Gloria and Felicitas Goo approached for assistance in securing the license, admitted not knowing where the license came from. The task of applying for the license
was delegated to a certain Qualin, who could have testified as to how the license was secured and thus impeached the certification of the Municipal Civil Registrar as well
as the testimony of her representative. As Gloria failed to present this Qualin, the certification of the Municipal Civil Registrar still enjoys probative value.
It is also noted that the solemnizing officer testified that the marriage contract and a copy of the marriage license were submitted to the Local Civil Registrar of Manila.
Thus, a copy of the marriage license could have simply been secured from that office and submitted to the court. However, Gloria inexplicably failed to do so, further
weakening her claim that there was a valid marriage license issued for her and Syed.
In the case of Cario v. Cario,
47
following the case of Republic,
48
it was held that the certification of the Local Civil Registrar that their office had no record of a marriage
license was adequate to prove the non-issuance of said license. The case of Cario further held that the presumed validity of the marriage of the parties had been
overcome, and that it became the burden of the party alleging a valid marriage to prove that the marriage was valid, and that the required marriage license had been
secured.
49
Gloria has failed to discharge that burden, and the only conclusion that can be reached is that no valid marriage license was issued. It cannot be said that
there was a simple irregularity in the marriage license that would not affect the validity of the marriage, as no license was presented by the respondent. No marriage
license was proven to have been issued to Gloria and Syed, based on the certification of the Municipal Civil Registrar of Carmona, Cavite and Glorias failure to produce
a copy of the alleged marriage license.
To bolster its ruling, the CA cited other evidence to support its conclusion that Gloria and Syed were validly married. To quote the CA:
Moreover, the record is replete with evidence, testimonial and documentary, that appellant and appellee have been validly married and there was compliance with all the
requisites laid down by law. Both parties are legally capacitated to marry. A certificate of legal capacity was even issued by the Embassy of Pakistan in favor of appellee.
The parties herein gave their consent freely. Appellee admitted that the signature above his name in the marriage contract was his. Several pictures were presented
showing appellant and appellee, before the solemnizing officer, the witnesses and other members of appellants family, taken during the marriage ceremony, as well as in
the restaurant where the lunch was held after the marriage ceremony. Most telling of all is Exhibit "5-C" which shows appellee signing the Marriage Contract.
x x x x
The parties have comported themselves as husband and wife and has [sic] one offspring, Aliea Fatima Goo Abbas, who was born on 15 June 1993. It took appellee more
than ten (10) years before he filed on 01 August 2003 his Petition for Declaration of Nullity of Marriage under Article 4 of the Family Code. We take serious note that said
Petition appears to have been instituted by him only after an Information for Bigamy (Exhibit "1") dated 10 January 2003 was filed against him for contracting a second or
subsequent marriage with one Ma. Corazon (Maryam) T. Buenaventura. We are not ready to reward (appellee) by declaring the nullity of his marriage and give him his
freedom and in the process allow him to profit from his own deceit and perfidy.
50

All the evidence cited by the CA to show that a wedding ceremony was conducted and a marriage contract was signed does not operate to cure the absence of a valid
marriage license. Article 4 of the Family Code is clear when it says, "The absence of any of the essential or formal requisites shall render the marriage void ab initio,
except as stated in Article 35(2)." Article 35(3) of the Family Code also provides that a marriage solemnized without a license is void from the beginning, except those
exempt from the license requirement under Articles 27 to 34, Chapter 2, Title I of the same Code.
51
Again, this marriage cannot be characterized as among the
exemptions, and thus, having been solemnized without a marriage license, is void ab initio.1wphi1
As to the motive of Syed in seeking to annul his marriage to Gloria, it may well be that his motives are less than pure, that he seeks to evade a bigamy suit. Be that as it
may, the same does not make up for the failure of the respondent to prove that they had a valid marriage license, given the weight of evidence presented by petitioner.
The lack of a valid marriage license cannot be attributed to him, as it was Gloria who took steps to procure the same. The law must be applied. As the marriage license, a
formal requisite, is clearly absent, the marriage of Gloria and Syed is void ab initio.
WHEREFORE, in light of the foregoing, the petition is hereby GRANTED. The assailed Decision dated March 11, 2008 and Resolution dated July 24, 2008 of the Court
of Appeals in CA-G.R. CV No. 86760 are hereby REVERSED and SET ASIDE. The Decision of the Regional Trial Court, Branch 109, Pasay City dated October 5, 2005
in Civil Case No. 03-0382-CFM annulling the marriage of petitioner with respondent on January 9, 1993 is hereby REINSTATED.
No costs.
SO ORDERED.
PRESBITERO J. VELASCO, JR.
Associate Justice

59
WE CONCUR:
DIOSDADO M. PERALTA
Associate Justice
ROBERTO A. ABAD
Associate Justice
JOSE CATRAL MENDOZA
Associate Justice
MARVIC MARIO VICTOR F. LEONEN
Associate Justice
A T T E S T A T I O N
I attest that the conclusions in the above Decision had been reached in consultation before the case was assigned to the writer of the opinion of the Court's Division.
PRESBITERO J. VELASCO, JR.
Associate Justice
Chairperson
C E R T I F I C A T I O N
Pursuant to Section 13, Article VIII of the Constitution and the Division Chairperson's Attestation, I certify that the conclusions in the above Decision had been reached in
consultation before the case was assigned to the writer of the opinion of the Court's Division.
MARIA LOURDES P. A. SERENO
Chief Justice

Footnotes
Art. 34. No license shall be necessary for the marriage of a man and a woman who have lived together as husband and wife for at least
five years and without any legal impediment to marry each other. The contracting parties shall state the foregoing facts in an affidavit
before any person authorized by law to administer oaths. The solemnizing officer shall also state under oath that he ascertained the
qualifications of the contracting parties and found no legal impediment to the marriage.

Republic of the Philippines
SUPREME COURT
Manila
FIRST DIVISION
G.R. No. L-68470 October 8, 1985
ALICE REYES VAN DORN, petitioner,
vs.
HON. MANUEL V. ROMILLO, JR., as Presiding Judge of Branch CX, Regional Trial Court of the National Capital Region Pasay City and
RICHARD UPTON respondents.

MELENCIO-HERRERA, J .:\
In this Petition for certiorari and Prohibition, petitioner Alice Reyes Van Dorn seeks to set aside the Orders, dated September 15, 1983 and August 3,
1984, in Civil Case No. 1075-P, issued by respondent Judge, which denied her Motion to Dismiss said case, and her Motion for Reconsideration of the
Dismissal Order, respectively.
The basic background facts are that petitioner is a citizen of the Philippines while private respondent is a citizen of the United States; that they were
married in Hongkong in 1972; that, after the marriage, they established their residence in the Philippines; that they begot two children born on April 4,
1973 and December 18, 1975, respectively; that the parties were divorced in Nevada, United States, in 1982; and that petitioner has re-married also in
Nevada, this time to Theodore Van Dorn.
Dated June 8, 1983, private respondent filed suit against petitioner in Civil Case No. 1075-P of the Regional Trial Court, Branch CXV, in Pasay City,
stating that petitioner's business in Ermita, Manila, (the Galleon Shop, for short), is conjugal property of the parties, and asking that petitioner be ordered

60
to render an accounting of that business, and that private respondent be declared with right to manage the conjugal property. Petitioner moved to
dismiss the case on the ground that the cause of action is barred by previous judgment in the divorce proceedings before the Nevada Court wherein
respondent had acknowledged that he and petitioner had "no community property" as of June 11, 1982. The Court below denied the Motion to Dismiss
in the mentioned case on the ground that the property involved is located in the Philippines so that the Divorce Decree has no bearing in the case. The
denial is now the subject of this certiorari proceeding.
Generally, the denial of a Motion to Dismiss in a civil case is interlocutory and is not subject to appeal. certiorari and Prohibition are neither the remedies
to question the propriety of an interlocutory order of the trial Court. However, when a grave abuse of discretion was patently committed, or the lower
Court acted capriciously and whimsically, then it devolves upon this Court in a certiorari proceeding to exercise its supervisory authority and to correct
the error committed which, in such a case, is equivalent to lack of jurisdiction.
1
Prohibition would then lie since it would be useless and a waste of time
to go ahead with the proceedings.
2
Weconsider the petition filed in this case within the exception, and we have given it due course.
For resolution is the effect of the foreign divorce on the parties and their alleged conjugal property in the Philippines.
Petitioner contends that respondent is estopped from laying claim on the alleged conjugal property because of the representation he made in the divorce
proceedings before the American Court that they had no community of property; that the Galleon Shop was not established through conjugal funds, and
that respondent's claim is barred by prior judgment.
For his part, respondent avers that the Divorce Decree issued by the Nevada Court cannot prevail over the prohibitive laws of the Philippines and its
declared national policy; that the acts and declaration of a foreign Court cannot, especially if the same is contrary to public policy, divest Philippine
Courts of jurisdiction to entertain matters within its jurisdiction.
For the resolution of this case, it is not necessary to determine whether the property relations between petitioner and private respondent, after their
marriage, were upon absolute or relative community property, upon complete separation of property, or upon any other regime. The pivotal fact in this
case is the Nevada divorce of the parties.
The Nevada District Court, which decreed the divorce, had obtained jurisdiction over petitioner who appeared in person before the Court during the trial
of the case. It also obtained jurisdiction over private respondent who, giving his address as No. 381 Bush Street, San Francisco, California, authorized
his attorneys in the divorce case, Karp & Gradt Ltd., to agree to the divorce on the ground of incompatibility in the understanding that there were neither
community property nor community obligations.
3
As explicitly stated in the Power of Attorney he executed in favor of the law firm of KARP & GRAD
LTD., 336 W. Liberty, Reno, Nevada, to represent him in the divorce proceedings:
xxx xxx xxx
You are hereby authorized to accept service of Summons, to file an Answer, appear on my behalf and do an things necessary and
proper to represent me, without further contesting, subject to the following:
1. That my spouse seeks a divorce on the ground of incompatibility.
2. That there is no community of property to be adjudicated by the Court.
3. 'I'hat there are no community obligations to be adjudicated by the court.
xxx xxx xxx
4

There can be no question as to the validity of that Nevada divorce in any of the States of the United States. The decree is binding on private respondent
as an American citizen. For instance, private respondent cannot sue petitioner, as her husband, in any State of the Union. What he is contending in this
case is that the divorce is not valid and binding in this jurisdiction, the same being contrary to local law and public policy.
It is true that owing to the nationality principle embodied in Article 15 of the Civil Code,
5
only Philippine nationals are covered by the policy against
absolute divorces the same being considered contrary to our concept of public police and morality. However, aliens may obtain divorces abroad, which
may be recognized in the Philippines, provided they are valid according to their national law.
6
In this case, the divorce in Nevada released private
respondent from the marriage from the standards of American law, under which divorce dissolves the marriage. As stated by the Federal Supreme Court
of the United States in Atherton vs. Atherton, 45 L. Ed. 794, 799:
The purpose and effect of a decree of divorce from the bond of matrimony by a court of competent jurisdiction are to change the
existing status or domestic relation of husband and wife, and to free them both from the bond. The marriage tie when thus severed
as to one party, ceases to bind either. A husband without a wife, or a wife without a husband, is unknown to the law. When the law
provides, in the nature of a penalty. that the guilty party shall not marry again, that party, as well as the other, is still absolutely freed
from the bond of the former marriage.
Thus, pursuant to his national law, private respondent is no longer the husband of petitioner. He would have no standing to sue in the case below as
petitioner's husband entitled to exercise control over conjugal assets. As he is bound by the Decision of his own country's Court, which validly exercised
jurisdiction over him, and whose decision he does not repudiate, he is estopped by his own representation before said Court from asserting his right over
the alleged conjugal property.
To maintain, as private respondent does, that, under our laws, petitioner has to be considered still married to private respondent and still subject to a
wife's obligations under Article 109, et. seq. of the Civil Code cannot be just. Petitioner should not be obliged to live together with, observe respect and

61
fidelity, and render support to private respondent. The latter should not continue to be one of her heirs with possible rights to conjugal property. She
should not be discriminated against in her own country if the ends of justice are to be served.
WHEREFORE, the Petition is granted, and respondent Judge is hereby ordered to dismiss the Complaint filed in Civil Case No. 1075-P of his Court.
Without costs.
SO ORDERED.
Teehankee (Chairman), Plana, Relova, Gutierrez, Jr., De la Fuente and Patajo, JJ., concur.

Footnotes
1 Sanchez vs. Zosa, 68 SCRA 171 (1975); Malit vs. People, 114 SCRA 348 (1982).
2 U.S.T. vs. Hon. Villanueva, et al., 106 Phil. 439 (1959).
3 Annex "Y", Petition for Certiorari.
4 p. 98, Rollo.
5 "Art. 15. Laws relating to family rights and duties or to the status, condition and legal capacity of persons are binding upon citizens
of the Philippines, even though living abroad.
6 cf. Recto vs. Harden, 100 Phil. 427 [1956]; Paras, Civil Code, 1971 ed., Vol. I, p. 52; Salonga, Private International Law, 1979 ed.,
p. 231."
Republic of the Philippines
SUPREME COURT
Manila
SECOND DIVISION
G.R. No. 80116 June 30, 1989
IMELDA MANALAYSAY PILAPIL, petitioner,
vs.
HON. CORONA IBAY-SOMERA, in her capacity as Presiding Judge of the Regional Trial Court of Manila, Branch XXVI; HON. LUIS C. VICTOR,
in his capacity as the City Fiscal of Manila; and ERICH EKKEHARD GEILING, respondents.

REGALADO, J .:
An ill-starred marriage of a Filipina and a foreigner which ended in a foreign absolute divorce, only to be followed by a criminal infidelity suit of the latter
against the former, provides Us the opportunity to lay down a decisional rule on what hitherto appears to be an unresolved jurisdictional question.
On September 7, 1979, petitioner Imelda Manalaysay Pilapil, a Filipino citizen, and private respondent Erich Ekkehard Geiling, a German national, were
married before the Registrar of Births, Marriages and Deaths at Friedensweiler in the Federal Republic of Germany. The marriage started auspiciously
enough, and the couple lived together for some time in Malate, Manila where their only child, Isabella Pilapil Geiling, was born on April 20, 1980.
1

Thereafter, marital discord set in, with mutual recriminations between the spouses, followed by a separation de facto between them.
After about three and a half years of marriage, such connubial disharmony eventuated in private respondent initiating a divorce proceeding against
petitioner in Germany before the Schoneberg Local Court in January, 1983. He claimed that there was failure of their marriage and that they had been
living apart since April, 1982.
2

Petitioner, on the other hand, filed an action for legal separation, support and separation of property before the Regional Trial Court of Manila, Branch
XXXII, on January 23, 1983 where the same is still pending as Civil Case No. 83-15866.
3

On January 15, 1986, Division 20 of the Schoneberg Local Court, Federal Republic of Germany, promulgated a decree of divorce on the ground of
failure of marriage of the spouses. The custody of the child was granted to petitioner. The records show that under German law said court was locally

62
and internationally competent for the divorce proceeding and that the dissolution of said marriage was legally founded on and authorized by the
applicable law of that foreign jurisdiction.
4

On June 27, 1986, or more than five months after the issuance of the divorce decree, private respondent filed two complaints for adultery before the City
Fiscal of Manila alleging that, while still married to said respondent, petitioner "had an affair with a certain William Chia as early as 1982 and with yet
another man named Jesus Chua sometime in 1983". Assistant Fiscal Jacinto A. de los Reyes, Jr., after the corresponding investigation, recommended
the dismissal of the cases on the ground of insufficiency of evidence.
5
However, upon review, the respondent city fiscal approved a resolution, dated
January 8, 1986, directing the filing of two complaints for adultery against the petitioner.
6
The complaints were accordingly filed and were eventually
raffled to two branches of the Regional Trial Court of Manila. The case entitled "People of the Philippines vs. Imelda Pilapil and William Chia", docketed
as Criminal Case No. 87-52435, was assigned to Branch XXVI presided by the respondent judge; while the other case, "People of the Philippines vs.
Imelda Pilapil and James Chua", docketed as Criminal Case No. 87-52434 went to the sala of Judge Leonardo Cruz, Branch XXV, of the same court.
7

On March 14, 1987, petitioner filed a petition with the Secretary of Justice asking that the aforesaid resolution of respondent fiscal be set aside and the
cases against her be dismissed.
8
A similar petition was filed by James Chua, her co-accused in Criminal Case No. 87-52434. The Secretary of Justice,
through the Chief State Prosecutor, gave due course to both petitions and directed the respondent city fiscal to inform the Department of Justice "if the
accused have already been arraigned and if not yet arraigned, to move to defer further proceedings" and to elevate the entire records of both cases to
his office for review.
9

Petitioner thereafter filed a motion in both criminal cases to defer her arraignment and to suspend further proceedings thereon.
10
As a consequence,
Judge Leonardo Cruz suspended proceedings in Criminal Case No. 87-52434. On the other hand, respondent judge merely reset the date of the
arraignment in Criminal Case No. 87-52435 to April 6, 1987. Before such scheduled date, petitioner moved for the cancellation of the arraignment and
for the suspension of proceedings in said Criminal Case No. 87-52435 until after the resolution of the petition for review then pending before the
Secretary of Justice.
11
A motion to quash was also filed in the same case on the ground of lack of jurisdiction,
12
which motion was denied by the
respondent judge in an order dated September 8, 1987. The same order also directed the arraignment of both accused therein, that is, petitioner and
William Chia. The latter entered a plea of not guilty while the petitioner refused to be arraigned. Such refusal of the petitioner being considered by
respondent judge as direct contempt, she and her counsel were fined and the former was ordered detained until she submitted herself for
arraignment.
13
Later, private respondent entered a plea of not guilty.
14

On October 27, 1987, petitioner filed this special civil action for certiorari and prohibition, with a prayer for a temporary restraining order, seeking the
annulment of the order of the lower court denying her motion to quash. The petition is anchored on the main ground that the court is without jurisdiction
"to try and decide the charge of adultery, which is a private offense that cannot be prosecuted de officio (sic), since the purported complainant, a
foreigner, does not qualify as an offended spouse having obtained a final divorce decree under his national law prior to his filing the criminal
complaint."
15

On October 21, 1987, this Court issued a temporary restraining order enjoining the respondents from implementing the aforesaid order of September 8,
1987 and from further proceeding with Criminal Case No. 87-52435. Subsequently, on March 23, 1988 Secretary of Justice Sedfrey A. Ordoez acted
on the aforesaid petitions for review and, upholding petitioner's ratiocinations, issued a resolution directing the respondent city fiscal to move for the
dismissal of the complaints against the petitioner.
16

We find this petition meritorious. The writs prayed for shall accordingly issue.
Under Article 344 of the Revised Penal Code,
17
the crime of adultery, as well as four other crimes against chastity, cannot be prosecuted except upon a
sworn written complaint filed by the offended spouse. It has long since been established, with unwavering consistency, that compliance with this rule is a
jurisdictional, and not merely a formal, requirement.
18
While in point of strict law the jurisdiction of the court over the offense is vested in it by the
Judiciary Law, the requirement for a sworn written complaint is just as jurisdictional a mandate since it is that complaint which starts the prosecutory
proceeding
19
and without which the court cannot exercise its jurisdiction to try the case.
Now, the law specifically provides that in prosecutions for adultery and concubinage the person who can legally file the complaint should be the offended
spouse, and nobody else. Unlike the offenses of seduction, abduction, rape and acts of lasciviousness, no provision is made for the prosecution of the
crimes of adultery and concubinage by the parents, grandparents or guardian of the offended party. The so-called exclusive and successive rule in the
prosecution of the first four offenses above mentioned do not apply to adultery and concubinage. It is significant that while the State, as parens patriae,
was added and vested by the 1985 Rules of Criminal Procedure with the power to initiate the criminal action for a deceased or incapacitated victim in the
aforesaid offenses of seduction, abduction, rape and acts of lasciviousness, in default of her parents, grandparents or guardian, such amendment did
not include the crimes of adultery and concubinage. In other words, only the offended spouse, and no other, is authorized by law to initiate the action
therefor.
Corollary to such exclusive grant of power to the offended spouse to institute the action, it necessarily follows that such initiator must have the status,
capacity or legal representation to do so at the time of the filing of the criminal action. This is a familiar and express rule in civil actions; in fact, lack of
legal capacity to sue, as a ground for a motion to dismiss in civil cases, is determined as of the filing of the complaint or petition.
The absence of an equivalent explicit rule in the prosecution of criminal cases does not mean that the same requirement and rationale would not apply.
Understandably, it may not have been found necessary since criminal actions are generally and fundamentally commenced by the State, through the
People of the Philippines, the offended party being merely the complaining witness therein. However, in the so-called "private crimes" or those which
cannot be prosecuted de oficio, and the present prosecution for adultery is of such genre, the offended spouse assumes a more predominant role since
the right to commence the action, or to refrain therefrom, is a matter exclusively within his power and option.
This policy was adopted out of consideration for the aggrieved party who might prefer to suffer the outrage in silence rather than go through the scandal
of a public trial.
20
Hence, as cogently argued by petitioner, Article 344 of the Revised Penal Code thus presupposes that the marital relationship is still
subsisting at the time of the institution of the criminal action for, adultery. This is a logical consequence since the raison d'etre of said provision of law
would be absent where the supposed offended party had ceased to be the spouse of the alleged offender at the time of the filing of the criminal case.
21


63
In these cases, therefore, it is indispensable that the status and capacity of the complainant to commence the action be definitely established and, as
already demonstrated, such status or capacity must indubitably exist as of the time he initiates the action. It would be absurd if his capacity to bring the
action would be determined by his status before or subsequent to the commencement thereof, where such capacity or status existed prior to but ceased
before, or was acquired subsequent to but did not exist at the time of, the institution of the case. We would thereby have the anomalous spectacle of a
party bringing suit at the very time when he is without the legal capacity to do so.
To repeat, there does not appear to be any local precedential jurisprudence on the specific issue as to when precisely the status of a complainant as an
offended spouse must exist where a criminal prosecution can be commenced only by one who in law can be categorized as possessed of such status.
Stated differently and with reference to the present case, the inquiry ;would be whether it is necessary in the commencement of a criminal action for
adultery that the marital bonds between the complainant and the accused be unsevered and existing at the time of the institution of the action by the
former against the latter.
American jurisprudence, on cases involving statutes in that jurisdiction which are in pari materia with ours, yields the rule that after a divorce has been
decreed, the innocent spouse no longer has the right to institute proceedings against the offenders where the statute provides that the innocent spouse
shall have the exclusive right to institute a prosecution for adultery. Where, however, proceedings have been properly commenced, a divorce
subsequently granted can have no legal effect on the prosecution of the criminal proceedings to a conclusion.
22

In the cited Loftus case, the Supreme Court of Iowa held that
'No prosecution for adultery can be commenced except on the complaint of the husband or wife.' Section 4932,
Code. Though Loftus was husband of defendant when the offense is said to have been committed, he had ceased to be
such when the prosecution was begun; and appellant insists that his status was not such as to entitle him to make the
complaint. We have repeatedly said that the offense is against the unoffending spouse, as well as the state, in explaining
the reason for this provision in the statute; and we are of the opinion that the unoffending spouse must be such when the
prosecution is commenced. (Emphasis supplied.)
We see no reason why the same doctrinal rule should not apply in this case and in our jurisdiction, considering our statutory law and jural policy on the
matter. We are convinced that in cases of such nature, the status of the complainant vis-a-vis the accused must be determined as of the time the
complaint was filed. Thus, the person who initiates the adultery case must be an offended spouse, and by this is meant that he is still married to the
accused spouse, at the time of the filing of the complaint.
In the present case, the fact that private respondent obtained a valid divorce in his country, the Federal Republic of Germany, is admitted. Said divorce
and its legal effects may be recognized in the Philippines insofar as private respondent is concerned
23
in view of the nationality principle in our civil law
on the matter of status of persons.
Thus, in the recent case of Van Dorn vs. Romillo, Jr., et al.,
24
after a divorce was granted by a United States court between Alice Van Dornja Filipina,
and her American husband, the latter filed a civil case in a trial court here alleging that her business concern was conjugal property and praying that she
be ordered to render an accounting and that the plaintiff be granted the right to manage the business. Rejecting his pretensions, this Court perspicuously
demonstrated the error of such stance, thus:
There can be no question as to the validity of that Nevada divorce in any of the States of the United States. The decree is
binding on private respondent as an American citizen. For instance, private respondent cannot sue petitioner, as her
husband, in any State of the Union. ...
It is true that owing to the nationality principle embodied in Article 15 of the Civil Code, only Philippine nationals are
covered by the policy against absolute divorces the same being considered contrary to our concept of public policy and
morality. However, aliens may obtain divorces abroad, which may be recognized in the Philippines, provided they are
valid according to their national law. ...
Thus, pursuant to his national law, private respondent is no longer the husband of petitioner. He would have no standing
to sue in the case below as petitioner's husband entitled to exercise control over conjugal assets. ...
25

Under the same considerations and rationale, private respondent, being no longer the husband of petitioner, had no legal standing to commence the
adultery case under the imposture that he was the offended spouse at the time he filed suit.
The allegation of private respondent that he could not have brought this case before the decree of divorce for lack of knowledge, even if true, is of no
legal significance or consequence in this case. When said respondent initiated the divorce proceeding, he obviously knew that there would no longer be
a family nor marriage vows to protect once a dissolution of the marriage is decreed. Neither would there be a danger of introducing spurious heirs into
the family, which is said to be one of the reasons for the particular formulation of our law on adultery,
26
since there would thenceforth be no spousal
relationship to speak of. The severance of the marital bond had the effect of dissociating the former spouses from each other, hence the actuations of
one would not affect or cast obloquy on the other.
The aforecited case of United States vs. Mata cannot be successfully relied upon by private respondent. In applying Article 433 of the old Penal Code,
substantially the same as Article 333 of the Revised Penal Code, which punished adultery "although the marriage be afterwards declared void", the
Court merely stated that "the lawmakers intended to declare adulterous the infidelity of a married woman to her marital vows, even though it should be
made to appear that she is entitled to have her marriage contract declared null and void, until and unless she actually secures a formal judicial
declaration to that effect". Definitely, it cannot be logically inferred therefrom that the complaint can still be filed after the declaration of nullity because
such declaration that the marriage is void ab initio is equivalent to stating that it never existed. There being no marriage from the beginning, any
complaint for adultery filed after said declaration of nullity would no longer have a leg to stand on. Moreover, what was consequently contemplated and
within the purview of the decision in said case is the situation where the criminal action for adultery was filed before the termination of the marriage by a

64
judicial declaration of its nullity ab initio. The same rule and requisite would necessarily apply where the termination of the marriage was effected, as in
this case, by a valid foreign divorce.
Private respondent's invocation of Donio-Teves, et al. vs. Vamenta, hereinbefore cited,
27
must suffer the same fate of inapplicability. A cursory reading
of said case reveals that the offended spouse therein had duly and seasonably filed a complaint for adultery, although an issue was raised as to its
sufficiency but which was resolved in favor of the complainant. Said case did not involve a factual situation akin to the one at bar or any issue
determinative of the controversy herein.
WHEREFORE, the questioned order denying petitioner's motion to quash is SET ASIDE and another one enteredDISMISSING the complaint in Criminal
Case No. 87-52435 for lack of jurisdiction. The temporary restraining order issued in this case on October 21, 1987 is hereby made permanent.
SO ORDERED.
Melencio-Herrera, Padilla and Sarmiento, JJ., concur.


Separate Opinions

PARAS, J ., concurring:
It is my considered opinion that regardless of whether We consider the German absolute divorce as valid also in the Philippines, the fact is that the
husband in the instant case, by the very act of his obtaining an absolute divorce in Germany can no longer be considered as the offended party in case
his former wife actually has carnal knowledge with another, because in divorcing her, he already implicitly authorized the woman to have sexual relations
with others. A contrary ruling would be less than fair for a man, who is free to have sex will be allowed to deprive the woman of the same privilege.
In the case of Recto v. Harden (100 Phil. 427 [1956]), the Supreme Court considered the absolute divorce between the American husband and his
American wife as valid and binding in the Philippines on the theory that their status and capacity are governed by their National law, namely, American
law. There is no decision yet of the Supreme Court regarding the validity of such a divorce if one of the parties, say an American, is married to a Filipino
wife, for then two (2) different nationalities would be involved.
In the book of Senate President Jovito Salonga entitled Private International Law and precisely because of theNational law doctrine, he considers the
absolute divorce as valid insofar as the American husband is concerned but void insofar as the Filipino wife is involved. This results in what he calls a
"socially grotesque situation," where a Filipino woman is still married to a man who is no longer her husband. It is the opinion however, of the
undersigned that very likely the opposite expresses the correct view. While under the national law of the husband the absolute divorce will be valid, still
one of the exceptions to the application of the proper foreign law (one of the exceptions to comity) is when the foreign law will work an injustice or injury
to the people or residents of the forum. Consequently since to recognize the absolute divorce as valid on the part of the husband would be injurious or
prejudicial to the Filipino wife whose marriage would be still valid under her national law, it would seem that under our law existing before the new Family
Code (which took effect on August 3, 1988) the divorce should be considered void both with respect to the American husband and the Filipino wife.
The recent case of Van Dorn v. Romillo, Jr. (139 SCRA [1985]) cannot apply despite the fact that the husband was an American can with a Filipino wife
because in said case the validity of the divorce insofar as the Filipino wife is concerned was NEVER put in issue.

Separate Opinions
PARAS, J ., concurring:
It is my considered opinion that regardless of whether We consider the German absolute divorce as valid also in the Philippines, the fact is that the
husband in the instant case, by the very act of his obtaining an absolute divorce in Germany can no longer be considered as the offended party in case
his former wife actually has carnal knowledge with another, because in divorcing her, he already implicitly authorized the woman to have sexual relations
with others. A contrary ruling would be less than fair for a man, who is free to have sex will be allowed to deprive the woman of the same privilege.
In the case of Recto v. Harden (100 Phil. 427 [1956]), the Supreme Court considered the absolute divorce between the American husband and his
American wife as valid and binding in the Philippines on the theory that their status and capacity are governed by their National law, namely, American
law. There is no decision yet of the Supreme Court regarding the validity of such a divorce if one of the parties, say an American, is married to a Filipino
wife, for then two (2) different nationalities would be involved.
In the book of Senate President Jovito Salonga entitled Private International Law and precisely because of theNational law doctrine, he considers the
absolute divorce as valid insofar as the American husband is concerned but void insofar as the Filipino wife is involved. This results in what he calls a
"socially grotesque situation," where a Filipino woman is still married to a man who is no longer her husband. It is the opinion however, of the
undersigned that very likely the opposite expresses the correct view. While under the national law of the husband the absolute divorce will be valid, still
one of the exceptions to the application of the proper foreign law (one of the exceptions to comity) is when the foreign law will work an injustice or injury

65
to the people or residents of the forum. Consequently since to recognize the absolute divorce as valid on the part of the husband would be injurious or
prejudicial to the Filipino wife whose marriage would be still valid under her national law, it would seem that under our law existing before the new Family
Code (which took effect on August 3, 1988) the divorce should be considered void both with respect to the American husband and the Filipino wife.
The recent case of Van Dorn v. Romillo, Jr. (139 SCRA [1985]) cannot apply despite the fact that the husband was an American can with a Filipino wife
because in said case the validity of the divorce insofar as the Filipino wife is concerned was NEVER put in issue.


Republic of the Philippines
SUPREME COURT
Manila
THIRD DIVISION
G.R. No. 138322 October 2, 2001
GRACE J. GARCIA, a.k.a. GRACE J. GARCIA-RECIO, petitioner,
vs.
REDERICK A. RECIO, respondents.
PANGANIBAN, J .:
A divorce obtained abroad by an alien may be recognized in our jurisdiction, provided such decree is valid according to the national law of the foreigner.
However, the divorce decree and the governing personal law of the alien spouse who obtained the divorce must be proven. Our courts do not take
judicial notice of foreign laws and judgment; hence, like any other facts, both the divorce decree and the national law of the alien must be alleged and
proven according to our law on evidence.
The Case
Before us is a Petition for Review under Rule 45 of the Rules of Court, seeking to nullify the January 7, 1999 Decision
1
and the March 24, 1999 Order
2
of
the Regional Trial Court of Cabanatuan City, Branch 28, in Civil Case No. 3026-AF. The assailed Decision disposed as follows:
"WHEREFORE, this Court declares the marriage between Grace J. Garcia and Rederick A. Recio solemnized on January 12, 1994 at
Cabanatuan City as dissolved and both parties can now remarry under existing and applicable laws to any and/or both parties."
3

The assailed Order denied reconsideration of the above-quoted Decision.
The Facts
Rederick A. Recio, a Filipino, was married to Editha Samson, an Australian citizen, in Malabon, Rizal, on March 1, 1987.
4
They lived together as
husband and wife in Australia. On May 18, 1989,
5
a decree of divorce, purportedly dissolving the marriage, was issued by an Australian family court.
On June 26, 1992, respondent became an Australian citizen, as shown by a "Certificate of Australian Citizenship" issued by the Australian
government.
6
Petitioner a Filipina and respondent were married on January 12, 1994 in Our Lady of Perpetual Help Church in Cabanatuan City.
7
In
their application for a marriage license, respondent was declared as "single" and "Filipino."
8

Starting October 22, 1995, petitioner and respondent lived separately without prior judicial dissolution of their marriage. While the two were still in
Australia, their conjugal assets were divided on May 16, 1996, in accordance with their Statutory Declarations secured in Australia.
9

On March 3, 1998, petitioner filed a Complaint for Declaration of Nullity of Marriage
10
in the court a quo, on the ground of bigamy respondent allegedly
had a prior subsisting marriage at the time he married her on January 12, 1994. She claimed that she learned of respondent's marriage to Editha
Samson only in November, 1997.
In his Answer, respondent averred that, as far back as 1993, he had revealed to petitioner his prior marriage andits subsequent dissolution.
11
He
contended that his first marriage to an Australian citizen had been validly dissolved by a divorce decree obtained in Australian in 1989;
12
thus, he was
legally capacitated to marry petitioner in 1994.1wphi1.nt
On July 7, 1998 or about five years after the couple's wedding and while the suit for the declaration of nullity was pending respondent was able to
secure a divorce decree from a family court in Sydney, Australia because the "marriage ha[d] irretrievably broken down."
13

Respondent prayed in his Answer that the Complained be dismissed on the ground that it stated no cause of action.
14
The Office of the Solicitor General
agreed with respondent.
15
The court marked and admitted the documentary evidence of both parties.
16
After they submitted their respective memoranda,
the case was submitted for resolution.
17

Thereafter, the trial court rendered the assailed Decision and Order.

66
Ruling of the Trial Court
The trial court declared the marriage dissolved on the ground that the divorce issued in Australia was valid and recognized in the Philippines. It deemed
the marriage ended, but not on the basis of any defect in an essential element of the marriage; that is, respondent's alleged lack of legal capacity to
remarry. Rather, it based its Decision on the divorce decree obtained by respondent. The Australian divorce had ended the marriage; thus, there was no
more martial union to nullify or annual.
Hence, this Petition.
18

Issues
Petitioner submits the following issues for our consideration:
"I
The trial court gravely erred in finding that the divorce decree obtained in Australia by the respondent ipso facto terminated his first marriage to
Editha Samson thereby capacitating him to contract a second marriage with the petitioner.
"2
The failure of the respondent, who is now a naturalized Australian, to present a certificate of legal capacity to marry constitutes absence of a
substantial requisite voiding the petitioner' marriage to the respondent.
"3
The trial court seriously erred in the application of Art. 26 of the Family Code in this case.
"4
The trial court patently and grievously erred in disregarding Arts. 11, 13, 21, 35, 40, 52 and 53 of the Family Code as the applicable provisions
in this case.
"5
The trial court gravely erred in pronouncing that the divorce gravely erred in pronouncing that the divorce decree obtained by the respondent
in Australia ipso facto capacitated the parties to remarry, without first securing a recognition of the judgment granting the divorce decree
before our courts."
19

The Petition raises five issues, but for purposes of this Decision, we shall concentrate on two pivotal ones: (1) whether the divorce between respondent
and Editha Samson was proven, and (2) whether respondent was proven to be legally capacitated to marry petitioner. Because of our ruling on these
two, there is no more necessity to take up the rest.
The Court's Ruling
The Petition is partly meritorious.
First Issue:
Proving the Divorce Between Respondent and Editha Samson
Petitioner assails the trial court's recognition of the divorce between respondent and Editha Samson. Citing Adong v. Cheong Seng Gee,
20
petitioner
argues that the divorce decree, like any other foreign judgment, may be given recognition in this jurisdiction only upon proof of the existence of (1) the
foreign law allowing absolute divorce and (2) the alleged divorce decree itself. She adds that respondent miserably failed to establish these elements.
Petitioner adds that, based on the first paragraph of Article 26 of the Family Code, marriages solemnized abroad are governed by the law of the place
where they were celebrated (the lex loci celebrationist). In effect, the Code requires the presentation of the foreign law to show the conformity of the
marriage in question to the legal requirements of the place where the marriage was performed.
At the outset, we lay the following basic legal principles as the take-off points for our discussion. Philippine law does not provide for absolute divorce;
hence, our courts cannot grant it.
21
A marriage between two Filipinos cannot be dissolved even by a divorce obtained abroad, because of Articles
15
22
and 17
23
of the Civil Code.
24
In mixed marriages involving a Filipino and a foreigner, Article 26
25
of the Family Code allows the former to contract a
subsequent marriage in case the divorce is "validly obtained abroad by the alien spouse capacitating him or her to remarry."
26
A divorce obtained abroad
by a couple, who are both aliens, may be recognized in the Philippines, provided it is consistent with their respective national laws.
27

A comparison between marriage and divorce, as far as pleading and proof are concerned, can be made. Van Dorn v. Romillo Jr. decrees that "aliens
may obtain divorces abroad, which may be recognized in the Philippines, provided they are valid according to their national law."
28
Therefore, before a

67
foreign divorce decree can be recognized by our courts, the party pleading it must prove the divorce as a fact and demonstrate its conformity to the
foreign law allowing it.
29
Presentation solely of the divorce decree is insufficient.
Divorce as a Question of Fact
Petitioner insists that before a divorce decree can be admitted in evidence, it must first comply with the registration requirements under Articles 11, 13
and 52 of the Family Code. These articles read as follows:
"ART. 11. Where a marriage license is required, each of the contracting parties shall file separately a sworn application for such license with
the proper local civil registrar which shall specify the following:
x x x x x x x x x
"(5) If previously married, how, when and where the previous marriage was dissolved or annulled;
x x x x x x x x x
"ART. 13. In case either of the contracting parties has been previously married, the applicant shall be required to furnish, instead of the birth of
baptismal certificate required in the last preceding article, the death certificate of the deceased spouse or the judicial decree of annulment or
declaration of nullity of his or her previous marriage. x x x.
"ART. 52. The judgment of annulment or of absolute nullity of the marriage, the partition and distribution of the properties of the spouses, and
the delivery of the children's presumptive legitimes shall be recorded in the appropriate civil registry and registries of property; otherwise, the
same shall not affect their persons."
Respondent, on the other hand, argues that the Australian divorce decree is a public document a written official act of an Australian family court.
Therefore, it requires no further proof of its authenticity and due execution.
Respondent is getting ahead of himself. Before a foreign judgment is given presumptive evidentiary value, the document must first be presented and
admitted in evidence.
30
A divorce obtained abroad is proven by the divorce decree itself. Indeed the best evidence of a judgment is the judgment
itself.
31
The decree purports to be a written act or record of an act of an officially body or tribunal of a foreign country.
32

Under Sections 24 and 25 of Rule 132, on the other hand, a writing or document may be proven as a public or official record of a foreign country by
either (1) an official publication or (2) a copy thereof attested
33
by the officer having legal custody of the document. If the record is not kept in the
Philippines, such copy must be (a) accompanied by a certificate issued by the proper diplomatic or consular officer in the Philippine foreign service
stationed in the foreign country in which the record is kept and (b) authenticated by the seal of his office.
34

The divorce decree between respondent and Editha Samson appears to be an authentic one issued by an Australian family court.
35
However,
appearance is not sufficient; compliance with the aforemetioned rules on evidence must be demonstrated.
Fortunately for respondent's cause, when the divorce decree of May 18, 1989 was submitted in evidence, counsel for petitioner objected, not to its
admissibility, but only to the fact that it had not been registered in the Local Civil Registry of Cabanatuan City.
36
The trial court ruled that it was
admissible, subject to petitioner's qualification.
37
Hence, it was admitted in evidence and accorded weight by the judge. Indeed, petitioner's failure to
object properly rendered the divorce decree admissible as a written act of the Family Court of Sydney, Australia.
38

Compliance with the quoted articles (11, 13 and 52) of the Family Code is not necessary; respondent was no longer bound by Philippine personal laws
after he acquired Australian citizenship in 1992.
39
Naturalization is the legal act of adopting an alien and clothing him with the political and civil rights
belonging to a citizen.
40
Naturalized citizens, freed from the protective cloak of their former states, don the attires of their adoptive countries. By
becoming an Australian, respondent severed his allegiance to the Philippines and the vinculum juris that had tied him to Philippine personal laws.
Burden of Proving Australian Law
Respondent contends that the burden to prove Australian divorce law falls upon petitioner, because she is the party challenging the validity of a foreign
judgment. He contends that petitioner was satisfied with the original of the divorce decree and was cognizant of the marital laws of Australia, because
she had lived and worked in that country for quite a long time. Besides, the Australian divorce law is allegedly known by Philippine courts: thus, judges
may take judicial notice of foreign laws in the exercise of sound discretion.
We are not persuaded. The burden of proof lies with "the party who alleges the existence of a fact or thing necessary in the prosecution or defense of an
action."
41
In civil cases, plaintiffs have the burden of proving the material allegations of the complaint when those are denied by the answer; and
defendants have the burden of proving the material allegations in their answer when they introduce new matters.
42
Since the divorce was a defense
raised by respondent, the burden of proving the pertinent Australian law validating it falls squarely upon him.
It is well-settled in our jurisdiction that our courts cannot take judicial notice of foreign laws.
43
Like any other facts, they must be alleged and proved.
Australian marital laws are not among those matters that judges are supposed to know by reason of their judicial function.
44
The power of judicial notice
must be exercised with caution, and every reasonable doubt upon the subject should be resolved in the negative.
Second Issue:

68
Respondent's Legal Capacity to Remarry
Petitioner contends that, in view of the insufficient proof of the divorce, respondent was legally incapacitated to marry her in 1994.
Hence, she concludes that their marriage was void ab initio.
Respondent replies that the Australian divorce decree, which was validly admitted in evidence, adequately established his legal capacity to marry under
Australian law.
Respondent's contention is untenable. In its strict legal sense, divorce means the legal dissolution of a lawful union for a cause arising after marriage.
But divorces are of different types. The two basic ones are (1) absolute divorce or a vinculo matrimonii and (2) limited divorce or a mensa et thoro. The
first kind terminates the marriage, while the second suspends it and leaves the bond in full force.
45
There is no showing in the case at bar which type of
divorce was procured by respondent.
Respondent presented a decree nisi or an interlocutory decree a conditional or provisional judgment of divorce. It is in effect the same as a separation
from bed and board, although an absolute divorce may follow after the lapse of the prescribed period during which no reconciliation is effected.
46

Even after the divorce becomes absolute, the court may under some foreign statutes and practices, still restrict remarriage. Under some other
jurisdictions, remarriage may be limited by statute; thus, the guilty party in a divorce which was granted on the ground of adultery may be prohibited from
remarrying again. The court may allow a remarriage only after proof of good behavior.
47

On its face, the herein Australian divorce decree contains a restriction that reads:
"1. A party to a marriage who marries again before this decree becomes absolute (unless the other party has died) commits the offence of
bigamy."
48

This quotation bolsters our contention that the divorce obtained by respondent may have been restricted. It did not absolutely establish his legal capacity
to remarry according to his national law. Hence, we find no basis for the ruling of the trial court, which erroneously assumed that the Australian
divorce ipso facto restored respondent's capacity to remarry despite the paucity of evidence on this matter.
We also reject the claim of respondent that the divorce decree raises a disputable presumption or presumptive evidence as to his civil status based on
Section 48, Rule 39
49
of the Rules of Court, for the simple reason that no proof has been presented on the legal effects of the divorce decree obtained
under Australian laws.
Significance of the Certificate of Legal Capacity
Petitioner argues that the certificate of legal capacity required by Article 21 of the Family Code was not submitted together with the application for a
marriage license. According to her, its absence is proof that respondent did not have legal capacity to remarry.
We clarify. To repeat, the legal capacity to contract marriage is determined by the national law of the party concerned. The certificate mentioned in
Article 21 of the Family Code would have been sufficient to establish the legal capacity of respondent, had he duly presented it in court. A duly
authenticated and admitted certificate is prima facie evidence of legal capacity to marry on the part of the alien applicant for a marriage license.
50

As it is, however, there is absolutely no evidence that proves respondent's legal capacity to marry petitioner. A review of the records before this Court
shows that only the following exhibits were presented before the lower court: (1) for petitioner: (a) Exhibit "A" Complaint;
51
(b) Exhibit "B" Certificate
of Marriage Between Rederick A. Recto (Filipino-Australian) and Grace J. Garcia (Filipino) on January 12, 1994 in Cabanatuan City, Nueva Ecija;
52
(c)
Exhibit "C" Certificate of Marriage Between Rederick A. Recio (Filipino) and Editha D. Samson (Australian) on March 1, 1987 in Malabon, Metro
Manila;
53
(d) Exhibit "D" Office of the City Registrar of Cabanatuan City Certification that no information of annulment between Rederick A. Recto and
Editha D. Samson was in its records;
54
and (e) Exhibit "E" Certificate of Australian Citizenship of Rederick A. Recto;
55
(2) for respondent: (Exhibit "1"
Amended Answer;
56
(b) Exhibit "S" Family Law Act 1975 Decree Nisi of Dissolution of Marriage in the Family Court of Australia;
57
(c) Exhibit "3"
Certificate of Australian Citizenship of Rederick A. Recto;
58
(d) Exhibit "4" Decree Nisi of Dissolution of Marriage in the Family Court of Australia
Certificate;
59
and Exhibit "5" Statutory Declaration of the Legal Separation Between Rederick A. Recto and Grace J. Garcia Recio since October 22,
1995.
60

Based on the above records, we cannot conclude that respondent, who was then a naturalized Australian citizen, was legally capacitated to marry
petitioner on January 12, 1994. We agree with petitioner's contention that the court a quo erred in finding that the divorce decree ipso facto clothed
respondent with the legal capacity to remarry without requiring him to adduce sufficient evidence to show the Australian personal law governing his
status; or at the very least, to prove his legal capacity to contract the second marriage.
Neither can we grant petitioner's prayer to declare her marriage to respondent null and void on the ground of bigamy. After all, it may turn out that under
Australian law, he was really capacitated to marry petitioner as a direct result of the divorce decree. Hence, we believe that the most judicious course is
to remand this case to the trial court to receive evidence, if any, which show petitioner's legal capacity to marry petitioner. Failing in that, then the court a
quo may declare a nullity of the parties' marriage on the ground of bigamy, there being already in evidence two existing marriage certificates, which were
both obtained in the Philippines, one in Malabon, Metro Manila dated March 1, 1987 and the other, in Cabanatuan City dated January 12, 1994.
WHEREFORE, in the interest of orderly procedure and substantial justice, we REMAND the case to the court a quo for the purpose of receiving
evidence which conclusively show respondent's legal capacity to marry petitioner; and failing in that, of declaring the parties' marriage void on the
ground of bigamy, as above discussed. No costs.

69
SO ORDERED.

FIRST DIVISION

REPUBLIC OF THE PHILIPPINES,
Petitioner,
G.R. No. 154380




- versus -

Present:

Davide, Jr., C.J.,
(Chairman),
Quisumbing,
Ynares-Santiago,
Carpio, and
Azcuna, JJ.
CIPRIANO ORBECIDO III,
Respondent.

Promulgated:
October 5, 2005
x - - - - - - - - - - - - - - - - - - - - - - - - - - - - - - - - - - - - - - - - - - - - - - - - - - x

DECISION
QUISUMBING, J .:
Given a valid marriage between two Filipino citizens, where one party is later naturalized as a foreign citizen and obtains a valid divorce
decree capacitating him or her to remarry, can the Filipino spouse likewise remarry under Philippine law?
Before us is a case of first impression that behooves the Court to make a definite ruling on this apparently novel question, presented as a pure
question of law.
In this petition for review, the Solicitor General assails the Decision
[1]
dated May 15, 2002, of the Regional Trial Court of Molave, Zamboanga
del Sur, Branch 23 and itsResolution
[2]
dated July 4, 2002 denying the motion for reconsideration. The court a quo had declared that herein respondent
Cipriano Orbecido III is capacitated to remarry. The fallo of the impugned Decision reads:
WHEREFORE, by virtue of the provision of the second paragraph of Art. 26 of the Family Code and by reason of the divorce
decree obtained against him by his American wife, the petitioner is given the capacity to remarry under the Philippine Law.
IT IS SO ORDERED.
[3]

The factual antecedents, as narrated by the trial court, are as follows.
On May 24, 1981, Cipriano Orbecido III married Lady Myros M. Villanueva at the United Church of Christ in the Philippines in Lam-an, Ozamis
City. Their marriage was blessed with a son and a daughter, Kristoffer Simbortriz V. Orbecido and Lady Kimberly V. Orbecido.
In 1986, Ciprianos wife left for the United States bringing along their son Kristoffer. A few years later, Cipriano discovered that his wife had
been naturalized as an American citizen.
Sometime in 2000, Cipriano learned from his son that his wife had obtained a divorce decree and then married a certain Innocent
Stanley. She, Stanley and her child by him currently live at 5566 A. Walnut Grove Avenue, San Gabriel, California.
Cipriano thereafter filed with the trial court a petition for authority to remarry invoking Paragraph 2 of Article 26 of the Family Code. No opposition
was filed. Finding merit in the petition, the court granted the same. The Republic, herein petitioner, through the Office of the Solicitor General (OSG),
sought reconsideration but it was denied.
In this petition, the OSG raises a pure question of law:
WHETHER OR NOT RESPONDENT CAN REMARRY UNDER ARTICLE 26 OF THE FAMILY CODE
[4]

The OSG contends that Paragraph 2 of Article 26 of the Family Code is not applicable to the instant case because it only applies to a valid mixed
marriage; that is, a marriage celebrated between a Filipino citizen and an alien. The proper remedy, according to the OSG, is to file a petition for
annulment or for legal separation.
[5]
Furthermore, the OSG argues there is no law that governs respondents situation. The OSG posits that this is a
matter of legislation and not of judicial determination.
[6]

For his part, respondent admits that Article 26 is not directly applicable to his case but insists that when his naturalized alien wife obtained a
divorce decree which capacitated her to remarry, he is likewise capacitated by operation of law pursuant to Section 12, Article II of the Constitution.
[7]

At the outset, we note that the petition for authority to remarry filed before the trial court actually constituted a petition for declaratory relief. In this
connection, Section 1, Rule 63 of the Rules of Court provides:
RULE 63

70
DECLARATORY RELIEF AND SIMILAR REMEDIES

Section 1. Who may file petitionAny person interested under a deed, will, contract or other written instrument, or whose
rights are affected by a statute, executive order or regulation, ordinance, or other governmental regulation may, before breach or
violation thereof, bring an action in the appropriate Regional Trial Court to determine any question of construction or validity arising,
and for a declaration of his rights or duties, thereunder.
. . .
The requisites of a petition for declaratory relief are: (1) there must be a justiciable controversy; (2) the controversy must be between persons
whose interests are adverse; (3) that the party seeking the relief has a legal interest in the controversy; and (4) that the issue is ripe for judicial
determination.
[8]

This case concerns the applicability of Paragraph 2 of Article 26 to a marriage between two Filipino citizens where one later acquired alien
citizenship, obtained a divorce decree, and remarried while in the U.S.A. The interests of the parties are also adverse, as petitioner representing the
State asserts its duty to protect the institution of marriage while respondent, a private citizen, insists on a declaration of his capacity to
remarry. Respondent, praying for relief, has legal interest in the controversy. The issue raised is also ripe for judicial determination inasmuch as when
respondent remarries, litigation ensues and puts into question the validity of his second marriage.
Coming now to the substantive issue, does Paragraph 2 of Article 26 of the Family Code apply to the case of respondent? Necessarily, we must
dwell on how this provision had come about in the first place, and what was the intent of the legislators in its enactment?
Brief Historical Background
On July 6, 1987, then President Corazon Aquino signed into law Executive Order No. 209, otherwise known as the Family Code, which took
effect on August 3, 1988. Article 26 thereof states:
All marriages solemnized outside the Philippines in accordance with the laws in force in the country where they were
solemnized, and valid there as such, shall also be valid in this country, except those prohibited under Articles 35, 37, and 38.
On July 17, 1987, shortly after the signing of the original Family Code, Executive Order No. 227 was likewise signed into law, amending
Articles 26, 36, and 39 of the Family Code. A second paragraph was added to Article 26. As so amended, it now provides:
ART. 26. All marriages solemnized outside the Philippines in accordance with the laws in force in the country where they
were solemnized, and valid there as such, shall also be valid in this country, except those prohibited under Articles 35(1), (4), (5)
and (6), 36, 37 and 38.
Where a marriage between a Filipino citizen and a foreigner is validly celebrated and a divorce is thereafter validly
obtained abroad by the alien spouse capacitating him or her to remarry, the Filipino spouse shall have capacity to remarry under
Philippine law. (Emphasis supplied)
On its face, the foregoing provision does not appear to govern the situation presented by the case at hand. It seems to apply only to cases
where at the time of the celebration of the marriage, the parties are a Filipino citizen and a foreigner. The instant case is one where at the time the
marriage was solemnized, the parties were two Filipino citizens, but later on, the wife was naturalized as an American citizen and subsequently obtained
a divorce granting her capacity to remarry, and indeed she remarried an American citizen while residing in the U.S.A.
Noteworthy, in the Report of the Public Hearings
[9]
on the Family Code, the Catholic Bishops Conference of the Philippines (CBCP) registered
the following objections to Paragraph 2 of Article 26:
1. The rule is discriminatory. It discriminates against those whose spouses are Filipinos who divorce them abroad. These
spouses who are divorced will not be able to re-marry, while the spouses of foreigners who validly divorce them abroad
can.
2. This is the beginning of the recognition of the validity of divorce even for Filipino citizens. For those whose foreign
spouses validly divorce them abroad will also be considered to be validly divorced here and can re-marry. We propose
that this be deleted and made into law only after more widespread consultation. (Emphasis supplied.)
Legislative Intent
Records of the proceedings of the Family Code deliberations showed that the intent of Paragraph 2 of Article 26, according to Judge Alicia
Sempio-Diy, a member of theCivil Code Revision Committee, is to avoid the absurd situation where the Filipino spouse remains married to the alien
spouse who, after obtaining a divorce, is no longer married to the Filipino spouse.
Interestingly, Paragraph 2 of Article 26 traces its origin to the 1985 case of Van Dorn v. Romillo, Jr.
[10]
The Van Dorn case involved a marriage
between a Filipino citizen and a foreigner. The Court held therein that a divorce decree validly obtained by the alien spouse is valid in the Philippines,
and consequently, the Filipino spouse is capacitated to remarry under Philippine law.
Does the same principle apply to a case where at the time of the celebration of the marriage, the parties were Filipino citizens, but later on,
one of them obtains a foreign citizenship by naturalization?
The jurisprudential answer lies latent in the 1998 case of Quita v. Court of Appeals.
[11]
In Quita, the parties were, as in this case, Filipino
citizens when they got married. The wife became a naturalized American citizen in 1954 and obtained a divorce in the same year. The Court therein
hinted, by way of obiter dictum, that a Filipino divorced by his naturalized foreign spouse is no longer married under Philippine law and can thus remarry.
Thus, taking into consideration the legislative intent and applying the rule of reason, we hold that Paragraph 2 of Article 26 should be
interpreted to include cases involving parties who, at the time of the celebration of the marriage were Filipino citizens, but later on, one of them becomes
naturalized as a foreign citizen and obtains a divorce decree. The Filipino spouse should likewise be allowed to remarry as if the other party were a
foreigner at the time of the solemnization of the marriage. To rule otherwise would be to sanction absurdity and injustice. Where the interpretation of a
statute according to its exact and literal import would lead to mischievous results or contravene the clear purpose of the legislature, it should be
construed according to its spirit and reason, disregarding as far as necessary the letter of the law. A statute may therefore be extended to cases not
within the literal meaning of its terms, so long as they come within its spirit or intent.
[12]

If we are to give meaning to the legislative intent to avoid the absurd situation where the Filipino spouse remains married to the alien spouse
who, after obtaining a divorce is no longer married to the Filipino spouse, then the instant case must be deemed as coming within the contemplation of
Paragraph 2 of Article 26.
In view of the foregoing, we state the twin elements for the application of Paragraph 2 of Article 26 as follows:
1. There is a valid marriage that has been celebrated between a Filipino citizen and a foreigner; and
2. A valid divorce is obtained abroad by the alien spouse capacitating him or her to remarry.
The reckoning point is not the citizenship of the parties at the time of the celebration of the marriage, but their citizenship at the time a valid divorce
is obtained abroad by the alien spouse capacitating the latter to remarry.
In this case, when Ciprianos wife was naturalized as an American citizen, there was still a valid marriage that has been celebrated between
her and Cipriano. As fate would have it, the naturalized alien wife subsequently obtained a valid divorce capacitating her to remarry. Clearly, the twin
requisites for the application of Paragraph 2 of Article 26 are both present in this case. Thus Cipriano, the divorced Filipino spouse, should be allowed
to remarry.
We are also unable to sustain the OSGs theory that the proper remedy of the Filipino spouse is to file either a petition for annulment or a
petition for legal separation. Annulment would be a long and tedious process, and in this particular case, not even feasible, considering that the
marriage of the parties appears to have all the badges of validity. On the other hand, legal separation would not be a sufficient remedy for it would not
sever the marriage tie; hence, the legally separated Filipino spouse would still remain married to the naturalized alien spouse.

71
However, we note that the records are bereft of competent evidence duly submitted by respondent concerning the divorce decree and the
naturalization of respondents wife. It is settled rule that one who alleges a fact has the burden of proving it and mere allegation is not evidence.
[13]

Accordingly, for his plea to prosper, respondent herein must prove his allegation that his wife was naturalized as an American citizen. Likewise,
before a foreign divorce decree can be recognized by our own courts, the party pleading it must prove the divorce as a fact and demonstrate its
conformity to the foreign law allowing it.
[14]
Such foreign law must also be proved as our courts cannot take judicial notice of foreign laws. Like any other
fact, such laws must be alleged and proved.
[15]
Furthermore, respondent must also show that the divorce decree allows his former wife to remarry as
specifically required in Article 26. Otherwise, there would be no evidence sufficient to declare that he is capacitated to enter into another marriage.
Nevertheless, we are unanimous in our holding that Paragraph 2 of Article 26 of the Family Code (E.O. No. 209, as amended by E.O. No. 227),
should be interpreted to allow a Filipino citizen, who has been divorced by a spouse who had acquired foreign citizenship and remarried, also to
remarry. However, considering that in the present petition there is no sufficient evidence submitted and on record, we are unable to declare, based on
respondents bare allegations that his wife, who was naturalized as an American citizen, had obtained a divorce decree and had remarried an American,
that respondent is now capacitated to remarry. Such declaration could only be made properly upon respondents submission of the aforecited evidence
in his favor.
ACCORDINGLY, the petition by the Republic of the Philippines is GRANTED. The assailed Decision dated May 15, 2002, and Resolution dated
July 4, 2002, of the Regional Trial Court of Molave, Zamboanga del Sur, Branch 23, are hereby SET ASIDE.
No pronouncement as to costs.
SO ORDERED.








Republic of the Philippines
Supreme Court
Manila


THIRD DIVISION


GERBERT R. CORPUZ,
Petitioner,




- versus -





DAISYLYN TIROL STO. TOMAS and The
SOLICITOR GENERAL,
Respondents. -- -
G.R. No. 186571

Present:

CARPIO MORALES, J., Chairperson,
BRION,
BERSAMIN,

*
ABAD, and
VILLARAMA, JR., JJ.



Promulgated:

August 11, 2010
x--------------------------------------------------------------------------------------------------------------x

D E C I S I O N

BRION, J .:


Before the Court is a direct appeal from the decision
[1]
of the Regional Trial Court (RTC) of Laoag City, Branch 11, elevated via a petition for
review on certiorari
[2]
under Rule 45 of the Rules of Court (present petition).

Petitioner Gerbert R. Corpuz was a former Filipino citizen who acquired Canadian citizenship through naturalization on November 29,
2000.
[3]
On January 18, 2005, Gerbert married respondent Daisylyn T. Sto. Tomas, a Filipina, in Pasig City.
[4]
Due to work and other professional
commitments, Gerbert left for Canada soon after the wedding. He returned to the Philippines sometime in April 2005 to surprise Daisylyn, but was
shocked to discover that his wife was having an affair with another man. Hurt and disappointed, Gerbert returned to Canada and filed a petition for
divorce. The Superior Court of Justice, Windsor, Ontario, Canada granted Gerberts petition for divorce onDecember 8, 2005. The divorce decree took
effect a month later, on January 8, 2006.
[5]


Two years after the divorce, Gerbert has moved on and has found another Filipina to love. Desirous of marrying his new Filipina fiance in
the Philippines, Gerbert went to the Pasig City Civil Registry Office and registered the Canadian divorce decree on his and Daisylyns marriage
certificate. Despite the registration of the divorce decree, an official of the National Statistics Office (NSO) informed Gerbert that the marriage between

72
him and Daisylyn still subsists under Philippine law; to be enforceable, the foreign divorce decree must first be judicially recognized by a competent
Philippine court, pursuant to NSO Circular No. 4, series of 1982.
[6]


Accordingly, Gerbert filed a petition for judicial recognition of foreign divorce and/or declaration of marriage as dissolved (petition)
with the RTC. Although summoned, Daisylyn did not file any responsive pleading but submitted instead a notarized letter/manifestation to the trial
court. She offered no opposition to Gerberts petition and, in fact, alleged her desire to file a similar case herself but was prevented by financial and
personal circumstances. She, thus, requested that she be considered as a party-in-interest with a similar prayer to Gerberts.

In its October 30, 2008 decision,
[7]
the RTC denied Gerberts petition. The RTC concluded that Gerbert was not the proper party to institute
the action for judicial recognition of the foreign divorce decree as he is a naturalized Canadian citizen. It ruled that only the Filipino spouse can avail of
the remedy, under the second paragraph of Article 26 of the Family Code,
[8]
in order for him or her to be able to remarry under Philippine law.
[9]
Article
26 of the Family Code reads:

Art. 26. All marriages solemnized outside the Philippines, in accordance with the laws in force in the country where they
were solemnized, and valid there as such, shall also be valid in this country, except those prohibited under Articles 35(1), (4), (5)
and (6), 36, 37 and 38.

Where a marriage between a Filipino citizen and a foreigner is validly celebrated and a divorce is thereafter
validly obtained abroad by the alien spouse capacitating him or her to remarry, the Filipino spouse shall likewise have
capacity to remarry under Philippine law.

This conclusion, the RTC stated, is consistent with the legislative intent behind the enactment of the second paragraph of Article 26 of the Family Code,
as determined by the Court in Republic v. Orbecido III;
[10]
the provision was enacted to avoid the absurd situation where the Filipino spouse remains
married to the alien spouse who, after obtaining a divorce, is no longer married to the Filipino spouse.
[11]


THE PETITION

From the RTCs ruling,
[12]
Gerbert filed the present petition.
[13]


Gerbert asserts that his petition before the RTC is essentially for declaratory relief, similar to that filed in Orbecido; he, thus, similarly asks for a
determination of his rights under the second paragraph of Article 26 of the Family Code. Taking into account the rationale behind the second paragraph
of Article 26 of the Family Code, he contends that the provision applies as well to the benefit of the alien spouse. He claims that the RTC ruling unduly
stretched the doctrine in Orbecido by limiting the standing to file the petition only to the Filipino spouse an interpretation he claims to be contrary to the
essence of the second paragraph of Article 26 of the Family Code. He considers himself as a proper party, vested with sufficient legal interest, to
institute the case, as there is a possibility that he might be prosecuted for bigamy if he marries his Filipina fiance in the Philippines since two marriage
certificates, involving him, would be on file with the Civil Registry Office. The Office of the Solicitor General and Daisylyn, in their respective
Comments,
[14]
both support Gerberts position.

Essentially, the petition raises the issue of whether the second paragraph of Article 26 of the Family Code extends to aliens the right to
petition a court of this jurisdiction for the recognition of a foreign divorce decree.

THE COURTS RULING

The alien spouse can claim no right under the second paragraph of Article 26 of the
Family Code as the substantive right it establishes is in favor of the Filipino spouse

The resolution of the issue requires a review of the legislative history and intent behind the second paragraph of Article 26 of the Family Code.

The Family Code recognizes only two types of defective marriages void
[15]
and voidable
[16]
marriages. In both cases, the basis for the judicial
declaration of absolute nullity or annulment of the marriage exists before or at the time of the marriage. Divorce, on the other hand, contemplates the
dissolution of the lawful union for cause arising afterthe marriage.
[17]
Our family laws do not recognize absolute divorce between Filipino citizens.
[18]


Recognizing the reality that divorce is a possibility in marriages between a Filipino and an alien, President Corazon C. Aquino, in the exercise
of her legislative powers under the Freedom Constitution,
[19]
enacted Executive Order No. (EO) 227, amending Article 26 of the Family Code to its
present wording, as follows:

Art. 26. All marriages solemnized outside the Philippines, in accordance with the laws in force in the country where they
were solemnized, and valid there as such, shall also be valid in this country, except those prohibited under Articles 35(1), (4), (5)
and (6), 36, 37 and 38.

Where a marriage between a Filipino citizen and a foreigner is validly celebrated and a divorce is thereafter
validly obtained abroad by the alien spouse capacitating him or her to remarry, the Filipino spouse shall likewise have
capacity to remarry under Philippine law.

Through the second paragraph of Article 26 of the Family Code, EO 227 effectively incorporated into the law this Courts holding in Van Dorn v. Romillo,
Jr.
[20]
and Pilapil v. Ibay-Somera.
[21]
In both cases, the Court refused to acknowledge the alien spouses assertion of marital rights after a foreign courts
divorce decree between the alien and the Filipino. The Court, thus, recognized that the foreign divorce had already severed the marital bond between
the spouses. The Court reasoned in Van Dorn v. Romillo that:

To maintain x x x that, under our laws, [the Filipino spouse] has to be considered still married to [the alien spouse]
and still subject to a wife's obligations x x x cannot be just. [The Filipino spouse] should not be obliged to live together with,
observe respect and fidelity, and render support to [the alien spouse]. The latter should not continue to be one of her heirs with
possible rights to conjugal property. She should not be discriminated against in her own country if the ends of justice are to
be served.
[22]



As the RTC correctly stated, the provision was included in the law to avoid the absurd situation where the Filipino spouse remains married to
the alien spouse who, after obtaining a divorce, is no longer married to the Filipino spouse.
[23]
The legislative intent is for the benefit of the Filipino

73
spouse, by clarifying his or her marital status, settling the doubts created by the divorce decree. Essentially, the second paragraph of Article 26 of
the Family Code provided the Filipino spouse a substantive right to have his or her marriage to the alien spouse considered as dissolved,
capacitating him or her to remarry.
[24]
Without the second paragraph of Article 26 of the Family Code, the judicial recognition of the foreign decree of
divorce, whether in a proceeding instituted precisely for that purpose or as a related issue in another proceeding, would be of no significance to the
Filipino spouse since our laws do not recognize divorce as a mode of severing the marital bond;
[25]
Article 17 of the Civil Code provides that the policy
against absolute divorces cannot be subverted by judgments promulgated in a foreign country. The inclusion of the second paragraph in Article 26 of
the Family Code provides the direct exception to this rule and serves as basis for recognizing the dissolution of the marriage between the Filipino spouse
and his or her alien spouse.

Additionally, an action based on the second paragraph of Article 26 of the Family Code is not limited to the recognition of the foreign divorce
decree. If the court finds that the decree capacitated the alien spouse to remarry, the courts can declare that the Filipino spouse is likewise capacitated
to contract another marriage. No court in this jurisdiction, however, can make a similar declaration for the alien spouse (other than that already
established by the decree), whose status and legal capacity are generally governed by his national law.
[26]


Given the rationale and intent behind the enactment, and the purpose of the second paragraph of Article 26 of the Family Code, the RTC was
correct in limiting the applicability of the provision for the benefit of the Filipino spouse. In other words, only the Filipino spouse can invoke the second
paragraph of Article 26 of the Family Code; the alien spouse can claim no right under this provision.


The foreign divorce decree is presumptive evidence of a right that clothes the party
with legal interest to petition for its recognition in this jurisdiction

We qualify our above conclusion i.e., that the second paragraph of Article 26 of the Family Code bestows no rights in favor of aliens with
the complementary statement that this conclusion is not sufficient basis to dismiss Gerberts petition before the RTC. In other words, the unavailability of
the second paragraph of Article 26 of the Family Code to aliens does not necessarily strip Gerbert of legal interest to petition the RTC for the recognition
of his foreign divorce decree. The foreign divorce decree itself, after its authenticity and conformity with the aliens national law have been duly proven
according to our rules of evidence, serves as a presumptive evidence of right in favor of Gerbert, pursuant to Section 48, Rule 39 of the Rules of Court
which provides for the effect of foreign judgments. This Section states:

SEC. 48. Effect of foreign judgments or final orders.The effect of a judgment or final order of a tribunal of a foreign
country, having jurisdiction to render the judgment or final order is as follows:

(a) In case of a judgment or final order upon a specific thing, the judgment or final order is conclusive upon the
title of the thing; and

(b) In case of a judgment or final order against a person, the judgment or final order is presumptive
evidence of a right as between the parties and their successors in interest by a subsequent title.

In either case, the judgment or final order may be repelled by evidence of a want of jurisdiction, want of notice to the party,
collusion, fraud, or clear mistake of law or fact.

To our mind, direct involvement or being the subject of the foreign judgment is sufficient to clothe a party with the requisi te interest to institute an action
before our courts for the recognition of the foreign judgment. In a divorce situation, we have declared, no less, that the divorce obtained by an alien
abroad may be recognized in the Philippines, provided the divorce is valid according to his or her national law.
[27]


The starting point in any recognition of a foreign divorce judgment is the acknowledgment that our courts do not take judicial notice of foreign
judgments and laws.

Justice Herrera explained that, as a rule, no sovereign is bound to give effect within its dominion to a judgment rendered by a
tribunal of another country.
[28]
This means that the foreign judgment and its authenticity must be proven as facts under our rules on evidence, together
with the aliens applicable national law to show the effect of the judgment on the alien himself or herself.
[29]
The recognition may be made in an action
instituted specifically for the purpose or in another action where a party invokes the foreign decree as an integral aspect of his claim or defense.

In Gerberts case, since both the foreign divorce decree and the national law of the alien, recognizing his or her capacity to obtain a divorce,
purport to be official acts of a sovereign authority, Section 24, Rule 132 of the Rules of Court comes into play. This Section requires proof, either by (1)
official publications or (2) copies attested by the officer having legal custody of the documents. If the copies of official records are not kept in
the Philippines, these must be (a) accompanied by a certificate issued by the proper diplomatic or consular officer in the Philippine foreign service
stationed in the foreign country in which the record is kept and (b) authenticated by the seal of his office.

The records show that Gerbert attached to his petition a copy of the divorce decree, as well as the required certificates proving its
authenticity,
[30]
but failed to include a copy of the Canadian law on divorce.
[31]
Under this situation, we can, at this point, simply dismiss the petition for
insufficiency of supporting evidence, unless we deem it more appropriate to remand the case to the RTC to determine whether the divorce decree is
consistent with the Canadian divorce law.

We deem it more appropriate to take this latter course of action, given the Article 26 interests that will be served and the Filipina wifes
(Daisylyns) obvious conformity with the petition. A remand, at the same time, will allow other interested parties to oppose the foreign judgment and
overcome a petitioners presumptive evidence of a right by proving want of jurisdiction, want of notice to a party, collusion, fraud, or clear mistake of law
or fact. Needless to state, every precaution must be taken to ensure conformity with our laws before a recognition is made, as the foreign judgment,
once recognized, shall have the effect of res judicata
[32]
between the parties, as provided in Section 48, Rule 39 of the Rules of Court.
[33]


In fact, more than the principle of comity that is served by the practice of reciprocal recognition of foreign judgments between nations, the res
judicata effect of the foreign judgments of divorce serves as the deeper basis for extending judicial recognition and for considering the alien spouse
bound by its terms. This same effect, as discussed above, will not obtain for the Filipino spouse were it not for the substantive rule that the second
paragraph of Article 26 of the Family Code provides.

Considerations beyond the recognition of the foreign divorce decree

As a matter of housekeeping concern, we note that the Pasig City Civil Registry Office has already recorded the divorce decree on
Gerbert and Daisylyns marriage certificate based on the mere presentation of the decree.
[34]
We consider the recording to be legally improper;
hence, the need to draw attention of the bench and the bar to what had been done.

74

Article 407 of the Civil Code states that [a]cts, events and judicial decrees concerning the civil status of persons shall be recorded in the civil
register. The law requires the entry in the civil registry of judicial decrees that produce legal consequences touching upon a persons legal capacity and
status, i.e., those affecting all his personal qualities and relations, more or less permanent in nature, not ordinarily terminable at his own will, such as his
being legitimate or illegitimate, or his being married or not.
[35]


A judgment of divorce is a judicial decree, although a foreign one, affecting a persons legal capacity and status that must be recorded. In fact,
Act No. 3753 or the Law on Registry of Civil Status specifically requires the registration of divorce decrees in the civil registry:

Sec. 1. Civil Register. A civil register is established for recording the civil status of persons, in which shall be
entered:

(a) births;
(b) deaths;
(c) marriages;
(d) annulments of marriages;
(e) divorces;
(f) legitimations;
(g) adoptions;
(h) acknowledgment of natural children;
(i) naturalization; and
(j) changes of name.

x x x x

Sec. 4. Civil Register Books. The local registrars shall keep and preserve in their offices the following books, in which they
shall, respectively make the proper entries concerning the civil status of persons:

(1) Birth and death register;

(2) Marriage register, in which shall be entered not only the marriages solemnized but also divorces and dissolved
marriages.

(3) Legitimation, acknowledgment, adoption, change of name and naturalization register.


But while the law requires the entry of the divorce decree in the civil registry, the law and the submission of the decree by themselves do not ipso
facto authorize the decreesregistration. The law should be read in relation with the requirement of a judicial recognition of the foreign judgment before
it can be given res judicata effect. In the context of the present case, no judicial order as yet exists recognizing the foreign divorce decree. Thus, the
Pasig City Civil Registry Office acted totally out of turn and without authority of law when it annotated the Canadian divorce decree on Gerbert and
Daisylyns marriage certificate, on the strength alone of the foreign decree presented by Gerbert.

Evidently, the Pasig City Civil Registry Office was aware of the requirement of a court recognition, as it cited NSO Circular No. 4, series of
1982,
[36]
and Department of Justice Opinion No. 181, series of 1982
[37]
both of which required a final order from a competent Philippine court before a
foreign judgment, dissolving a marriage, can be registered in the civil registry, but it, nonetheless, allowed the registration of the decree. For being
contrary to law, the registration of the foreign divorce decree without the requisite judicial recognition is patently void and cannot produce any legal
effect.

Another point we wish to draw attention to is that the recognition that the RTC may extend to the Canadian divorce decree does not, by itself,
authorize the cancellation of the entry in the civil registry. A petition for recognition of a foreign judgment is not the proper proceeding, contemplated
under the Rules of Court, for the cancellation of entries in the civil registry.

Article 412 of the Civil Code declares that no entry in a civil register shall be changed or corrected, without judicial order. The Rules of Court
supplements Article 412 of the Civil Code by specifically providing for a special remedial proceeding by which entries in the civil registry may be judicially
cancelled or corrected. Rule 108 of the Rules of Court sets in detail the jurisdictional and procedural requirements that must be complied with before a
judgment, authorizing the cancellation or correction, may be annotated in the civil registry. It also requires, among others, that the verified petition must
be filed with the RTC of the province where the corresponding civil registry is located;
[38]
that the civil registrar and all persons who have or claim any
interest must be made parties to the proceedings;
[39]
and that the time and place for hearing must be published in a newspaper of general
circulation.
[40]
As these basic jurisdictional requirements have not been met in the present case, we cannot consider the petition Gerbert filed with the
RTC as one filed under Rule 108 of the Rules of Court.

We hasten to point out, however, that this ruling should not be construed as requiring two separate proceedings for the registration of a foreign
divorce decree in the civil registry one for recognition of the foreign decree and another specifically for cancellation of the entry under Rule 108 of the
Rules of Court. The recognition of the foreign divorce decree may be made in a Rule 108 proceeding itself, as the object of special proceedings (such
as that in Rule 108 of the Rules of Court) is precisely to establish the status or right of a party or a particular fact. Moreover, Rule 108 of the Rules of
Court can serve as the appropriate adversarial proceeding
[41]
by which the applicability of the foreign judgment can be measured and tested in terms of
jurisdictional infirmities, want of notice to the party, collusion, fraud, or clear mistake of law or fact.

WHEREFORE, we GRANT the petition for review on certiorari, and REVERSE the October 30, 2008 decision of
the Regional Trial Court of Laoag City, Branch 11, as well as its February 17, 2009 order. We order the REMAND of the case to the trial court for further
proceedings in accordance with our ruling above. Let a copy of this Decision be furnished the Civil Registrar General. No costs.

SO ORDERED.

You might also like